You are on page 1of 159

Krishna's

TEXT BOOK on

M athematical
M ethods
(For B.A. and B.Sc. IVth Semester students of Kumaun University)

Kumaun University Semester Syllabus w.e.f. 2017-18

By

A.R. Vasishtha A.K. Vasishtha


Retired Head, Dept. of Mathematics M.Sc., Ph.D.
Meerut College, Meerut C.C.S. University, Meerut

Kumaun

KRISHNA Prakashan Media (P) Ltd.


KRISHNA HOUSE, 11, Shivaji Road, Meerut-250001 (U.P.), India
Jai Shri Radhey Shyam

Dedicated
to

Lord

Krishna
Authors & Publishers
P reface
This book on Mathematical Methods has been specially written according to the
latest Syllabus to meet the requirements of B.A. and B.Sc. Semester-IV Students of
all colleges affiliated to Kumaun University.
The subject matter has been discussed in such a simple way that the students will find
no difficulty to understand it. The proofs of various theorems and examples have been
given with minute details. Each chapter of this book contains complete theory and a fairly
large number of solved examples. Sufficient problems have also been selected from various
university examination papers. At the end of each chapter an exercise containing objective
questions has been given.
We have tried our best to keep the book free from misprints. The authors shall be
grateful to the readers who point out errors and omissions which, inspite of all care, might
have been there.
The authors, in general, hope that the present book will be warmly received by the
students and teachers. We shall indeed be very thankful to our colleagues for their
recommending this book to their students.
The authors wish to express their thanks to Mr. S.K. Rastogi, M.D., Mr. Sugam Rastogi,
Executive Director, Mrs. Kanupriya Rastogi, Director and entire team of KRISHNA
Prakashan Media (P) Ltd., Meerut for bringing out this book in the present nice form.
The authors will feel amply rewarded if the book serves the purpose for which it is
meant. Suggestions for the improvement of the book are always welcome.

— Authors
Syllabus

M athematical M ethods
B.A./B.Sc. IV Semester
Kumaun University
Fourth Semester – Third Paper
B.A./B.Sc. Paper-III M.M.-60

Integral Transforms: Definition, Kernel.

Laplace Transforms: Definition, Existence theorem, Linearity property, Laplace


transforms of elementary functions, Heaviside Step and Dirac Delta Functions, First Shifting
Theorem, Second Shifting Theorem, Initial-Value Theorem, Final-Value Theorem, The
Laplace Transform of derivatives, integrals and Periodic functions.

Inverse Laplace Transforms: Inverse Laplace transforms of simple functions, Inverse


Laplace transforms using partial fractions, Convolution, Solutions of differential and
integro-differential equations using Laplace transforms. Dirichlet’s condition.

Fourier Transforms: Fourier Complex Transforms, Fourier sine and cosine transforms,
Properties of FourierTransforms, Inverse Fourier transforms.
B rief C ontents
Dedication.........................................................................(v)
Preface............................................................................(vi)
Syllabus (Kumaun University)............................................(vii)
Brief Contents ................................................................(viii)

Chapter 1: The Laplace Transform...............................................M-01—M-48

Chapter 2: The Inverse Laplace Transform....................................M-49—M-82

Chapter 3: Applications of Laplace Transform..............................M-83—M-110

Chapter 4: Fourier Transforms..................................................M-111—M-144

Chapter 5: Finite Fourier Transforms.........................................M-145—M-156


Krishna’s

MATHEMATICAL METHODS
C hapters

1. The Laplace Transform

2. The Inverse Laplace Transform

3. Applications of Laplace Transform

4. Fourier Transforms

5. Finite Fourier Transforms


M-3

1
T he L aplace T ransform

aplace transform or Laplace transformation reduces the problem of solving a


L differential equation to an algebraic problem. It is a method for solving linear
differential equations arising in Physics and Engineering.

1 Integral Transform
Let K ( p, t) be a function of two variables p and t, where p is a parameter (may be real or
complex) independent of t. The function f ( p) defined by the integral (assumed to be
convergent)

f ( p) = ∫− ∞ K ( p, t) F (t) dt

is called the integral transform of the function F (t) and is denoted by T {F (t)}. The
function K ( p, t) is called the kernel of the transformation.
Remark: Some authors use the letter ‘s’ in place of ‘p’.
M-4

2 Laplace Transformation
If the kernel K ( p, t) is defined as
0 for t < 0
K ( p, t) =  − pt
 e for t ≥ 0

then f ( p) = ∫0 e − pt F (t) dt. …(1)

The function f ( p) defined by the integral (1) is called the Laplace transform of the
function F (t) and is also denoted by
L {F (t)} or F ( p).
Thus Laplace transform is a function of a new variable (or parameter) p given by (1).
Note: The Laplace transform of F (t) is said to exist if the integral (1) converges for some
values of p, otherwise it does not exist.

3 Linearity Property of Laplace Transformation


(Bundelkhand 2014)
A transformation T is said to be linear if for every pair of functions F1 (t) and F2 (t) and
for every pair of constants a1 and a2 , we have
T { a1 F1 (t) + a2 F2 (t)} = a1 T {F1 (t)} + a2 T {F2 (t)}.
Theorem: The Laplace transformation is a linear transformation, i.e.
L { a1 F1 (t) + a2 F2 (t)} = a1 L {F1 (t)} + a2 L {F2 (t)}
where a1 , a2 are constants. (Avadh 2010)

Proof: We have

L {F (t)} = ∫0 e − pt F (t) dt .

∞ − pt
∴ L { a1 F1 (t) + a2 F2 (t)} = ∫0 e { a1 F1(t) + a2 F2 (t)} dt

∞ ∞
= a1 ∫ e − pt F1 (t) dt + a2 ∫0 e − pt F2 (t) dt
0

= a1 L {F1 (t)} + a2 L {F2 (t)}.

4 Piecewise (or Sectionally) Continuous Function


A function F (t) is said to be piecewise (or sectionally) continuous on a closed interval
a ≤ t ≤ b, if it is defined on that interval and is such that the interval can be subdivided
into a finite number of intervals, in each of which F (t) is continuous and has finite right
and left hand limits.
M-5

5 Functions of Exponential Order


A function F (t) is said to be of exponential order α as t → ∞ if there exists a positive constant (real)
M, a number α and a finite number t0 such that | F (t)| < Me αt

or | e − α t F (t)| < M , for all t ≥ t0 .


If a function F (t) is of exponential order α, it is also of β, β > α.

6 A Function of Class A
A function which is piecewise (or sectionally) continuous on every finite interval in the
range t ≥ 0 and is of exponential order as t → ∞ is known as ‘a function of class A’.

7 Existence of Laplace Transform


Theorem: If F (t) is a function which is piecewise continuous on every finite interval in the range
t ≥ 0 and satisfies| F (t)| ≤ Me at for all t ≥ 0 and for some constants a and M, then the Laplace
transform of F (t) exists for all p > a.
Proof: We have

L {F (t)} = ∫0 e − pt F (t) dt

t0 ∞
= ∫0 e − pt F (t) dt + ∫ t0 e − pt F (t) dt …(1)

t0 − pt
The integral ∫0 e F (t) dt exists since F (t) is piecewise continuous on every finite

interval 0 ≤ t ≤ t0 .
∞ − pt ∞ ∞ − pt
Now ∫ t0 e F (t) dt ≤ ∫ | e − pt F (t)| dt ≤ ∫ t0 e Me at dt ,
t0

since | F (t)| ≤ Me at
∞ Me − ( p − a) t0
= ∫ t0 e −(p − a) t M dt = , p > a.
p− a
∞ Me −(p − a)t0
∴ ∫ t0 e − pt F (t) dt ≤ , p > a.
p− a

Me − ( p − a) t0
But can be made as small as we please by taking t0 sufficiently large.
p− a

Thus from (1), we conclude that L {F (t)} exists for all p > a.
Note 1: The above theorem of existence of Laplace transform can also be stated
as :
M-6

“If F (t) is a function which is piece-wise continuous on every finite interval in the range
t ≥ 0 and is of exponential order a as t → ∞, the Laplace transform of F (t) exists for all
p > a”.
Or
“If F (t) is a function of class A, the Laplace transform of F (t) exists for p > a”.
Note 2: Conditions in the theorem are sufficient but not necessary for the existence of
Laplace transform. If these conditions are satisfied, the Laplace transform must exist. If
these conditions are not satisfied, the Laplace transform may or may not exist.
We can show this by the following example.
Illustration : Consider the function F (t) = 1 / √ t.

Here F (t) → ∞ as t → 0, from the right.


Thus the function F (t) is not piece-wise continuous on every finite interval in the range
t ≥ 0.
But F (t) is integrable from 0 to any positive value t0 .
Also | F (t)| < Me at for all t > 1 with M = 1 and a = 0.

Now L {F (t)} = ∫0 e − pt ⋅ F (t) dt

∞ 1
= ∫0 e − pt ⋅ dt, which converges for p > 0
√t
2 ∞ 2 dt 2
= ∫0 e − x dx, putting √ ( pt) = x so that = dx
√p √t √ p
2 √π ∞ 2 √π
= ⋅ , since ∫0 e − x dx =
√p 2 2

= √ (π / p), p > 0.
Thus L {1/ √ t} exists for p > 0 even if 1/ √ t is not piecewise continuous in the
range t ≥ 0.

8 Laplace Transforms of Some Elementary Functions


(i) Laplace transform of the function F (t) = 1.

Solution: We have L { F (t)} = ∫0 e − pt F (t) dt.


∞  e − pt  1
∴ L {1} = ∫0 e − pt ⋅ 1 dt = −  = , p> 0 .
 p  p
0

Here the condition p > 0 is necessary, since the integral is convergent for p > 0 and
divergent for p ≤ 0.
M-7

(ii) Laplace transform of the function F (t) = t n, n is any real number greater than –1.

Solution: We have,

L {F (t)} = ∫0 e − pt F (t) dt.

∞ ∞
∴ L { t n} = ∫0 e − pt t ndt = ∫0 e − pt t(n + 1) − 1dt …(1)

Now from Eulerian integral of the second kind known as ‘Gamma function’, we have
∞ − ax m −1 Γ(m)
∫0 e x dx = am , if a > 0 and m > 0.
Γ (n + 1)
∴ from (1), L { t n} = , if p > 0 and n + 1 > 0 i.e., n > − 1.
p n+1
Thus if n is any real number greater than –1, we have
Γ (n + 1)
L { t n} = , p > 0.
p n+1

Here the condition p > 0 is necessary for the convergence of the integral (1).
(iii) Laplace transform of the function F (t) = t n, n is a positive integer. (Gorakhpur 2008)

Solution: We have

L {F (t)} = ∫0 e − pt F (t) dt

∞ ∞
∴ L { t n} = ∫0 e − pt t n dt = ∫0 e − pt t(n + 1) −1 dt

Γ (n + 1)
= , if p > 0
p n+1
 ∞ − ax m −1 Γ(m)
 ∵ ∫ 0 e x dx = m , if a > 0 and m > 0.
a

Here n + 1 > 0, n being a positive integer. 

n!
= , p > 0.
p n +1
[ ∵ Γ (n + 1) = n !, n being a positive integer]
n!
Thus if n is any positive integer, we have L { t n} = n + 1 , p > 0 .
p

Here the condition p > 0 is necessary for the convergence of the integral defining
the Laplace transform of t n.
(iv) Laplace transform of the function F (t) = e at . (Rohilkhand 2009)

Solution: Here
∞ ∞
L { e at} = ∫0 e − pt ⋅ e at dt = ∫0 e − (p − a) t dt
M-8


 e −(p − a) t 
= −  , p≠ a
 p − a 
0
1
= , p > a.
p− a

Here the condition p > a is necessary, since the integral is convergent for p > a and
divergent for p ≤ a.
(v) Laplace transform of the function F (t) = sin at.
(Avadh 2010, 11)

Solution: L {sin at} = ∫0 e − pt sin at dt


 e − pt 
= 2 2
(− p sin at − a cos at)
 p + a 0
 e ax 
 ∵ ∫ e sin bx dx = 2
ax
2
(a sin bx − b cos bx)
 a +b 
a
= , p > 0.
p2 + a2

Here the condition p > 0 is necessary for the convergence of the integral (1).
(vi) Laplace transform of the function F (t) = cos at. (Rohilkhand 2006, 07, 10)

Solution: L {cos at } = ∫0 e− pt cos at dt


 e − pt 
= 2 2
(− p cos at + a sin at)
 p + a 0
 e ax 
∫e
ax
∵ cos bx dx = 2
(a cos bx + b sin bx)
2
 a +b 
p
= , p > 0.
p + a2
2

(vii) Laplace transform of the function F (t) = cosh at.


1
Solution: L {cosh a t} = L { (e at + e − at )}
2
1 1
= L { e at} + L { e − at}
2 2
1 1 1 1
= ⋅ + ⋅ , p > a and p > − a
2 p− a 2 p+ a
p
= , p > | a |.
p −a2
2
M-9

(viii) Laplace transform of the function F (t) = sinh at.


1
Solution: L {sinh at} = L { (e at − e − at )}
2
1
= [ L { e at} − L { e − at}]
2
1 1 1 
=  −  , p > a and p > − a
2  p − a p + a
a
= , p > | a |.
p − a2
2

9 Laplace Transforms of Some Elementary Functions


Laplace Transforms of some elementary functions obtained in 1.8 are given here
in the form of a table.

Laplace Transforms of Some Elementary Functions

F (t) L {F (t)}
1
1. 1 , p> 0
p

tn n!
2. , p> 0
(n is a positive integer) p n +1

ta Γ (a + 1)
3. , p> 0
(a > − 1) p a +1

1
4. e at , p> a
p− a

a
5. sin at , p> 0
p + a2
2

p
6. cos at , p> 0
p + a2
2

a
7. sinh at , p > | a|
p − a2
2

p
8. cosh at , p > | a|
p − a2
2

If we know the Laplace transforms in the above table then nearly all the transforms can
be obtained by using the general theorems which we shall consider later on.
M-10

Example 1: Find L {(t 2 + 1)2} .

Solution: We have
L { (t2 + 1)2} = L { t 4 + 2 t2 + 1} = L { t 4} + 2 L {t2} + L {1}
2 4
4! 2! 1 24 + 4 p + p
= +2⋅ + = , p > 0.
p5 p3 p p5

Example 2: Find the L.T. of the function


F (t) = (sin t − cos t)2 . (Gorakhpur 2006, 10)
2 2 2
Solution: We have L { (sin t − cos t) } = L {sin t + cos t − 2 sin t cos t}
1 2
= L {1} − L {sin 2 t} = − 2 , p> 0
p p + 22
p2 − 2 p + 4
= , p > 0.
p ( p2 + 4)

Example 3: (i) Evaluate L {4 cos2 2 t} .

Solution: We have L {4 cos2 2 t}

= L {2 (1 + cos 4 t)} = 2 [ L{1} + L {cos 4 t}]


1 p 
=2 + 2 2
, p> 0
 p p + 4 
4 ( p2 + 8)
` = , p > 0.
p ( p2 + 16)

Example 3: (ii) Evaluate L { sin2 at}. (Rohilkhand 2014)

Solution: We have
1 1
L {sin2 at} = L { (1 − cos 2 at)} = [ L {1} − L {cos 2 at}]
2 2
1 1 p 
=  − 2 2  , p> 0
2  p p + (2 a) 

2 a2
= , p > 0.
p ( p + 4 a2 )
2

Example 4: Find L {3 t 4 − 2 t3 + 4 e −3 t − 2 sin 5 t + 3 cos 2 t}.

Solution: We have
L {3 t 4 − 2 t 3 + 4 e −3 t − 2 sin 5 t + 3 cos 2 t}
= 3 L { t 4} − 2 L { t 3} + 4 L { e −3 t} − 2 L{sin 5 t} + 3 L {cos 2 t}
M-11

4! 3! 1 5 p
=3⋅ 5
−2⋅ 4
+4⋅ −2⋅ 2 2
+3⋅ 2 ,
p p p+3 p +5 p + 22
p > 0 and p > − 3 i. e. p > 0
72 4 12
10 3p
= 5 − 4 + − + , p > 0.
p p p + 3 p2 + 25 p2 + 4

(t − 1)2 , t > 1


Example 5: Find L { F (t)} if F (t) = 
 0 , 0 < t < 1.

Solution: We have
∞ 1 ∞
L { F (t)} = ∫0 F (t) e − pt dt = ∫ 0 ⋅ e − pt dt + ∫1 (t − 1)2 e − pt dt
0
∞ ∞ 2
= ∫1 (t − 1)2 e − pt dt = ∫0 x e − p ( x +1) dx, putting t − 1 = x ,

so that dt = dx and changing the limits


∞ ∞ Γ(3)
= ∫0 e − p e − px x2 dx = e − p ∫0 e − px x(3 −1)dx = e − p ⋅ 3 , p > 0
p
 ∞ − ax Γ(m) 
∵ ∫0 e x m −1dx = m
, a > 0, m > 0
a 
2! 2e−p
= e−p . = , p > 0.
p3 p3
2
Example 6: Show that the function e t is not of exponential order as t → ∞.
Solution: We have
2
lim { e − at F (t)} = lim { e − at e t } = lim e t(t − a)
t→ ∞ t→ ∞ t→ ∞

= ∞ for all values of a.


Hence whatever be the value of a, we cannot find a number M such that
2
e t < M e at .
∴ the given function is not of exponential order as t → ∞ .
Example 7: Find L { sin √ t }. (Purvanchal 2007; Avadh 12)

Solution: We have
 (√ t)3 (√ t)5 (√ t)7 
L {sin √ t} = L  √ t − + − + ...
 3! 5! 7! 
 t3 / 2
t5 / 2
t7 / 2

= L  t1 /2 − + − + ....
 3! 5! 7! 
1 1 1
= L { t1 /2} − L { t3 /2} + L { t5 /2} − L { t7 /2} + ...
3! 5! 7!
3 5 7 9
Γ( ) Γ( ) Γ( ) Γ( )
1 1 1
= 32/2 − ⋅ 2 + ⋅ 2 − ⋅ 2 + ...
p 3 ! p5 /2 5 ! p7 /2 7 ! p9 /2
M-12

1 3 1 5 3 1
√π ⋅ √π ⋅ ⋅ √π
2 1 2 2 1 2 2 2
= 3 /2 − ⋅ + ⋅ − ...
p 1⋅ 2 ⋅ 3 p5 /2 1⋅ 2 ⋅ 3 ⋅ 4 ⋅ 5 p7 /2

√ π  
2 3
1 1 1  1 1  1
= 3 /2 1 −   +   −   + ...
2p  1!  4 p 2 !  4 p 3 !  4 p 
 
√π
= 3 / 2 ⋅ e −1 / 4 p .
2p

Example 8: Show that the Laplace transform of the function


F (t) = t n, − 1 < n < 0 ,
exists, although it is not a function of class A.
Solution: Here F (t) → ∞ as t → 0 from the right i.e., the function is not piecewise
continuous on every finite interval in the range t ≥ 0 .
 tn 
We have lim { e − at F (t)} = lim  at 
t→ ∞ t→ ∞ e 

1 1
= lim = lim , where 0 < m < 1
t→ ∞ t − n e at t→ ∞ t m e at

= 0 , if a > 0 .
∴ F (t) = t n is of exponential order.
Since F (t) = t n is not sectionally continuous over every finite interval in the range t ≥ 0,
hence it is not a function of class A. But t n is integrable from 0 to any positive number
t0 .
∞ − pt ∞ − pt n ∞ − pt (n+1)−1
Now L { F (t)} = ∫0 e F (t) dt = ∫0 e t dt = ∫
0
e t dt

Γ(n + 1)
= , if p > 0 and n + 1 > 0 i. e., n > − 1.
p(n + 1)
Hence the Laplace transform of t n, − 1 < n < 0, exists, although it is not a function of
class A.

Comprehensive Exercise 1

Find the L.T. of the following functions.


1. (i) 2 t3 − 6 t + 8 (ii) sin t cos t
(iii) cosh2 2t. (iv) 6 sin 2 t − 5 cos 2 t
2. (i) 3 cosh 5 t − 4 sin 5 t
(ii) 7 e2 t + 9 e −2 t + 5 cos t + 7 t3 + 5 sin 3 t + 2 .
(iii) 2 e3 t − e −3 t . (Rohilkhand 2006)
at
(iv) F (t) = (e − 1) / a.
M-13

 e , 0 < t ≤ 1
t
3. (i) Find L {F (t)}, if F (t) = 
 0 , t >1
0 , 0 < t < 2
(ii) Find L {F (t)}, where F (t) = 
 4, t>2
4. (i) Find the Laplace transform of the following function :
 x / a, 0 < x < a
f ( x) = 
 1, x > a.

0 , 0 < t < 1

(ii) Find L {F (t)}, where F (t) =  t, 1 < t < 2 (Lucknow 2010)
0 , t > 2.

5. (i) Find Laplace Transform of the function F (t), where
sin t, 0 < t < π
F (t) =  .
 0, t> π (Lucknow 2007)

 e , 0 < t < 5
t
(ii) Find L {F (t)}, if F (t) = 
3, t > 5.
1, 0 < t < 2
6. (i) Find L {F (t)}, if F (t) = 
 t, t > 2.
 t, 0 < t < 4
(ii) Find L { F (t)}, if F (t) = 
5, t > 4.

2 (1 − e − π p ) sin 2 t, 0< t< π


7. (i) Prove that L {H (t)} = , where H (t) = 
p +42
 0, t > π.

 1  1
(ii) Prove that L  = ⋅
 √ (π t) √ p (Avadh 2014)

 cos √ t   π
8. Show that L   =   e −1 /4 p .
 √ t   p (Purvanchal 2007; Kashi 14)

9. Show that t2 is of exponential order 3.

A nswers 1
1
1. (i) 12 / p4 − 6 / p2 + 8 / p, p > 0 (ii) 2
, p> 0
( p + 4)
p2 − 8 12 − 5 p
(iii) 2
, p> 4 (iv) , p> 0
p ( p − 16) p2 + 4
3 p − 20
2. (i) , p> 5
p2 − 25
M-14

16 p − 4 5p 42 + 2 p3 15
(ii) 2
+ 2
+ 4
+ 2
, p> 2
p −4 p +1 p p +9
p+9
(iii) , p> 3
p2 − 9
1
(iv) , p > 0 if a < 0 and p > a if a > 0
p ( p − a)
1 4 −2 p
3. (i) [1 − e − ( p − 1)], p ≠ 1 (ii) e , p> 0
( p − 1) p
1
4. (i) (1 − e − ap ), p > 0
a p2
2 1 1 1 
(ii) −  + 2  e −2 p +  + 2  e − p , p ≠ 0 .
p p  p p 
e− p π + 1 1 − e −5( p −1) 3 −5 p
5. (i) (ii) + e , p> 0
p2 + 1 p−1 p

1 1 1 + ( p − 1) e −4 p
6. (i) [1 + e −2 p ] + 2 e −2 p , p > 0 (ii) , p> 0
p p p2

10 First Translation or Shifting Theorem


If L {F (t)} = f ( p) when p > α, then L { e at F (t)} = f ( p − a), p > α + a; i.e., if f ( p) is the
Laplace transform of F (t) then f ( p − a) is the Laplace transform of e at F (t).

(Gorakhpur 2008, 11)

Proof: By definition, we have


∞ − pt
f ( p) = L { F (t)} = ∫0 e F (t) dt,
…(1)
the integral (1) being given to be convergent if p > α.
∞ −( p − a) t
∴ f ( p − a) = ∫0 e F (t) dt
…(2)
∞ − pt
∫0
at at
= e ⋅e F (t) dt = L { e F (t)}.

Obviously if the integral (1) converges when p > α, the integral (2) converges when
p − a > α i. e., p > α + a.

11 Second Translation or Shifting Theorem


 F (t − a) , t > a
If L {F (t)} = f ( p) and G (t) =  then L { G (t)} = e − ap f ( p).
0 , t < a,
(Avadh 2007; Rohilkhand 07; Lucknow 09)
M-15

Proof: By definition, we have


∞ a ∞
L { G (t)} = ∫0 e − pt G (t) dt = ∫0 e − pt ⋅ 0 dt + ∫a e − pt ⋅ F (t − a) dt
∞ ∞
= ∫a e − pt F (t − a) dt = ∫0 e − p (a + x) ⋅ F ( x) dx,

putting t − a = x so that dt = dx
− pa ∞ − px − pa ∞ − pt
=e ∫0 e F ( x) dx = e ∫0 e F (t) dt,

by a property of definite integrals


− pa − pa
=e L{F (t)} = e f ( p).

12 Change of Scale Property


1  p
If L {F (t)} = f ( p), then L { F (at)} = f  ⋅
a  a
(Avadh 2006, 11; Kanpur 07; 10; Lucknow 07; Rohilkhand 08, 11)

Proof: By definition, we have



L {F (at)} = ∫0 e − pt F (at) dt

1 ∞
= ∫0 e − p ( x / a) F ( x) dx, putting at = x, so that dt = (1 / a) dx
a
1 ∞  b b 
= ∫0 e −( p / a)t F (t) dt ∵ ∫a f ( x) dx = ∫a f (t) dt
a 
1  p ∞ − pt
= f   , since f ( p) =
a  a ∫0 e F (t) dt.

Example 9: Find (i) L { e − t sin2 t} (Rohilkhand 2004)


t 2
(ii) L { e sin t}. (Kanpur 2012)

1
Solution: We have L {sin2 t} = L { (1 − cos 2 t)}
2
1 1 p  2
=  − 2 2
= 2
= f ( p) , say.
2  p p + 2  p ( p + 4)
∴ From first shifting theorem, we have
2 2
(i) L { e − t sin2 t} = f ( p + 1) = 2
= 2

( p + 1) {( p + 1) + 4} ( p + 1) ( p + 2 p + 5)
2 2
(ii) L { e t sin2 t} = f ( p − 1) = = .
( p − 1) {( p − 1)2 + 4} ( p − 1) ( p2 − 2 p + 5)
M-16

Example 10: Find L { e − t (3 sinh 2 t − 5 cosh 2 t)}.

Solution: We have L {3 sinh 2 t − 5 cosh 2 t}


2 p 6 − 5p
=3⋅ 2 2
−5⋅ 2 2
= 2 = f ( p), say.
p −2 p −2 p −4
∴ From first shifting theorem, we have
L { e − t (3 sinh 2 t − 5 cosh 2 t)} = f ( p + 1)
6 − 5 ( p + 1) 1− 5p
= 2
= 2
.
( p + 1) − 4 p + 2p − 3
Example 11: Using first shifting theorem evaluate
L { e6 t (t + 2)2}.
Solution: We have, L {(t + 2)2} = L { t2 + 4 t + 4}
2! 1! 4 2 + 4 p + 4 p2
= +4⋅ + = = f ( p), say.
p3 p2 p p3
∴ From first shifting theorem, we have
L { e6 t (t + 2)2} = f ( p − 6)
2 + 4 ( p − 6) + 4 ( p − 6)2 4 p2 − 44 p + 122
= = .
( p − 6)3 ( p − 6)3

p2 − p + 1
Example 12: Given L {F (t)} = ,
(2 p + 1)2 ( p − 1)

applying the change of scale property show that


p2 − 2 p + 4
L {F (2 t)} = .
4 ( p + 1)2 ( p − 2)
Solution: We have
p2 − p + 1
L {F (t)} = = f ( p), say.
(2 p + 1)2 ( p − 1)
∴ by the change of scale property, we have
1
L {F (2 t)} = f ( p / 2)
2
1 ( p / 2)2 − ( p / 2) + 1 p2 − 2 p + 4
= ⋅ = ⋅
2 [2 ⋅ ( p / 2) + 1]2 [( p / 2) − 1] 4 ( p + 1)2 ( p − 2)

 2
 cos (t − π), t > 2 π / 3
Example 13:. Find L {F (t)}, where F (t) =  3
 0 , t < 2 π / 3.

Solution: Let φ (t) = cos t.


 φ (t − 2 π / 3), t > 2 π / 3
Then F (t) = 
 0 , t < 2 π / 3.
M-17

p
We have L { φ (t)} = L {cos t} = 2
= f ( p), say.
p +1

∴ From second shifting theorem, we have


p
L {F (t)} = e(−2 π /3)p . f ( p) = e −2 πp /3 ⋅ 2
.
p +1

Aliter: We have L {F (t)} = ∫0 e − pt F (t) dt

2 π /3 ∞  2π 
= ∫0 e − pt ⋅ 0 dt + ∫ 2 π /3 e − pt ⋅ cos  t −  dt
 3
∞  2π 
= ∫ 2 π /3 e − pt ⋅ cos  t −  dt
 3

= ∫0 e − p [ x + (2 π /3)] cos x dx, putting t − 2 π / 3 = x and dt = dx

= e − p (2 π /3) ∫0 e − px cos x dx

= e − p (2 π /3) ∫0 e − pt cos t dt

= e − p (2 π /3) L{cos t} = e −2 p π /3 [ p / ( p2 + 1)], p > 0 .

Comprehensive Exercise 2

Find the Laplace transform of the following functions.


1. (i) t3 e −3 t (Kanpur 2008) (ii) e − at t n−1
/ (n − 1) !
n at
(iii) t e . (Avadh 2010)
3t 3t
2. (i) e sin 4 t. (Lucknow 2010) (ii) e cos 5 t.
3. (i) e t cos2 t. (ii) e − 4 t cosh 2 t.
4. (i) e − t (3 sin 2 t − 5 cosh 2 t). (ii) e − 2 t (3 cos 6 t − 5 sin 6 t).
5. (t + 3)2 . e t .
6. If L { F (t) } = f ( p), find L { F (t) cos ωt }.
7. Applying change of scale property, find
(i) L {sinh 3 t } and (ii) L {cos 5 t }.
t−a
 e ,t> a
8. Find L {G (t)}, where G (t) = 
0 , t < a.
sin (t − π / 3), t > π / 3
9. Find L {F (t)}, where F (t) = 
 0 , t < π / 3.

e −3 /( p + 1)
10. If L {F (t)} = (1 / p) e −1 / p , prove that L { e − t F (3 t)} = .
p+1
M-18

A nswers 2
1. (i) 6 /( p + 3)4 . (ii) 1 / ( p + a)n
(iii) n !/ ( p − a)n +1.
2. (i) 4 /( p2 − 6 p + 25). (ii) ( p − 3) /( p2 − 6 p + 34).
( p2 − 2 p + 3) p+4
3. (i) ⋅ (ii) ⋅
( p − 1) ( p2 − 2 p + 5) p2 + 8 p + 12
6 5 ( p + 1) 3 p − 24
4. (i) 2
− 2
⋅ (ii) 2

p +2 p+5 p +2 p−3 p + 4 p + 40
9 p2 − 12 p + 5 1
5. 3
⋅ 6. [ f ( p − iω) + f ( p + iω)].
( p − 1) 2
3 p
7. (i) 2
. (ii) 2
, p > 0.
p −9 p + 25
e − ap e − π p /3
8. , p > 1. 9. , p > 0.
p−1 p2 + 1

13 Laplace Transform of the Derivative of F ( t )


Theorem: Let F (t) be continuous for all t ≥ 0 and be of exponential order a as t → ∞ and if
F ′ (t) is of class A, then Laplace transform of the derivative F ′ (t) exists when p > a, and
L {F ′ (t)} = pL {F (t)} − F (0 ).
Proof:
Case I: In case F ′ (t) is continuous for all t ≥ 0, then

L {F ′ (t)} = ∫0 e − pt F ′ (t) dt …(1)

∞ ∞
= [e − pt F (t)] 0 + p ∫ e − pt F (t) dt [Integrating by parts]
0
lim − pt
= e F (t) − F (0 ) + pL {F (t)}. …(2)
t→ ∞
Now | F (t)| ≤ Me at for all t ≥ 0 and for some constants a and M.
We have | e − pt F (t)| = e − pt | F (t)| ≤ e − pt Me at
= Me −( p − a)t → 0 as t → ∞ if p > a.
lim − pt
∴ e F (t) = 0 for p > a.
t→ ∞
Therefore from (2) we conclude that L {F ′ (t)} exists and
L {F ′ (t)} = pL {F (t)} − F (0 ).
M-19

Case II: In case F ′ (t) is merely piece-wise continuous, the integral (1) may be broken
as the sum of integrals in different ranges from 0 to ∞ such that F ′ (t) is continuous in
each of such parts.
Then proceeding as in case I, we shall have
L {F ′ (t)} = pL {F (t)} − F (0 ).
Note 1: If F (t) fails to be continuous at t = 0 but
lim
F (t) = F (0 + 0 ) exists, [F (0 + 0 ) is not equal to F (0 ),
t→0
which may or may not exist]
then L {F ′ (t)} = pL {F (t)} − F (0 + 0 ).
Note 2: If F (t) fails to be continuous at t = a, then
L { F ′ (t)} = pL {F (t)} − F (0 ) − e − ap [ F (a + 0 ) − F (a − 0 )]
where F (a + 0 ) and F (a − 0 ) are the limits of F at t = a, as t approaches a from the right
and from the left respectively.
The quantity F (a + 0 ) − F (a − 0 ) is called the jump at the discontinuity t = a.
∞ a ∞
Proof: L {F ′ (t)} = ∫0 e − pt F ′ (t) dt = ∫0 e − pt F ′ (t) dt + ∫a e − pt F ′ (t) dt

a ∞
= [e − pt F (t)] 0 + p ∫ e − pt F (t) dt + [e − pt F (t)] a
a
0

+ p∫ e − pt F (t) dt
a
lim
= e − ap F (a − 0 ) − F (0 ) + e − pt F (t)
t→ ∞

− e − ap F (a + 0 ) + p ∫ e − pt F (t) dt
0

= pL {F (t)} − F (0 ) − e − ap [ F (a + 0 ) − F (a − 0 )].
 lim − pt 
∵ t → ∞ e F (t) = 0 , as shows in the theorem
 
Note 3: For more than one discontinuity of the function F (t), appropriate
modification can be made.

14 Laplace Transform of the nth Order Derivative of


F (t)
n −1
Theorem: Let F (t) and its derivatives F ′ (t), F ′ ′ (t),..., F (t) be continuous functions
n
for all t ≥ 0 and be of exponential orders as t → ∞ and if F (t) is of class A, then Laplace
transform of F n (t) exists when p > a, and is given by

L {F n (t)} = p n L {F (t)} − p n −1 F (0 ) − pn − 2 F ′ (0 ) − ... − F n −1


(0 ).
M-20

Proof: From the theorem of article 13, we have


L {F ′ (t)} = pL {F (t)} − F (0 ) …(1)
Applying the result (1) to the second order derivative F ′ ′ (t), we have
L {F ′ ′ (t)} = pL {F ′ (t)} − F ′ (0 )
= p [ pL {F (t)} − F (0 )] − F ′ (0 )
= p2 L {F (t)} − pF (0 ) − F ′ (0 ). …(2)
(Gorakhpur 2006, 10)

Again applying (1) to the third order derivative F ′ ′ ′ (t), we have


L {F ′ ′ ′ (t)} = pL {F ′ ′ (t)} − F ′ ′ (0 )
= p [ p2 L {F (t)} − pF (0 ) − F ′ (0 )] − F ′ ′ (0 )

= p3 L {F (t)} − p2 F (0 ) − pF ′ (0 ) − F ′ ′ (0 ).

Proceeding similarly, we have


L {F n (t)} = p n L {F (t)} − p n − 1 F (0 ) − p n−2 F ′ (0 ) − ... − F n −1
(0 )
n −1
or L {F n (t)} = p n L {F (t)} − ∑ p n − 1 − r F r (0 ).
r =0

15 Initial-Value Theorem
Let F (t) be continuous for all t ≥ 0 and be of exponential order as t → ∞ and if F ′ (t) is of class A,
lim lim
then F (t) = p L {F (t)}.
t→0 p→ ∞

Proof: By the theorem of article 13, we have



L {F ′ (t)} = ∫0 e − pt F ′ (t) dt = pL{F (t)} − F (0 ) …(1)

Since F ′ (t) is sectionally continuous and of exponential order,


lim ∞ − pt

p→ ∞ 0 ∫ e F ′ (t) dt = 0 .

Taking limit as p → ∞ in (1), we have


lim lim
0= pL {F (t)} − F (0 ) or F (0 ) = p L {F (t)}
p→ ∞ p→ ∞
lim lim
or F (t) = p L {F (t)}.
t→0 p→ ∞
lim
Note: If F (t) fails to be continuous at t = 0, but F (t) exists, the result still holds
t→0
by using Note (1), article 13 theorem.
M-21

16 Final-Value Theorem
Theorem: Let F (t) be continuous for all t ≥ 0 and be of exponential order as t → ∞ and if
F ′ (t) is of class A, then
lim lim
F (t) = p L {F (t)}.
t→ ∞ p→ 0
Proof: By the theorem of article 13, we have

L {F ′ (t)} = ∫0 e − pt F ′ (t) dt

= pL { F (t)} − F (0 ). …(1)
Taking limit as p → 0 in (1), we have
lim ∞ − pt lim
p→ 0 0∫ e F ′ (t) dt =
p→ 0
pL {F (t)} − F (0 )


or ∫0 F ′ (t) dt = lim pL { F (t) − F (0 )}
p→0

lim
or [ F(t)] 0∞ = p → 0 p L {F (t)} − F (0 )

lim lim
or F (t) − F (0 ) = pL {F (t)} − F (0 )
t→ ∞ p→ 0
lim lim
or F (t) = pL {F (t)}.
t→ ∞ p→ 0
lim
Note: If F (t) fails to be continuous at t = 0, but F (t) exists, the result still holds
t→0
by using Note (1), article 13 theorem.

17 Laplace Transform of Integrals


Theorem: If F (t) is piecewise continuous and satisfies| F (t)| ≤ Me at for all t ≥ 0 for some
constants a and M, then
 t  1
L  ∫ F ( x) dx = L { F (t)}, ( p > 0 , p > a) (Lucknow 2010)
 0  p
Proof: Let F (t) be piece-wise continuous such that | F (t)| ≤ Me a t , …(1)
for some constants a and M.
If (1) holds for some negative value of a then it also holds for positive value of a.
Therefore suppose that a is positive.
t
Let G (t) = ∫0 F ( x) dx.

Then G (t) is continuous because the integral of an integrable function is continuous.


M-22
t t
Also | G (t)| ≤ ∫ 0 | F ( x)| dx ≤ ∫ 0 Me ax dx.

M at
∴ | G (t)| ≤ (e − 1), a > 0 . …(2)
a
Further G ′ (t) = F (t), except for points at which F (t) is discontinuous.
Therefore G ′ (t) is piece-wise continuous on each finite interval.
Hence from the theorem of article 13, we have
L { G ′ (t)} = pL { G (t)} − G (0 ) = pL {G (t)} [Since G (0 ) = 0 from (2)]
1
∴ L { G (t)} = L { G ′ (t)}
p
 t  1
L  ∫ F ( x) dx = L { F (t)}
 0  p

18 Multiplication By t
Theorem: If F (t) is a function of class A and if L {F (t)} = f ( p), then
L {t F (t)} = − f ′ ( p).
(Rohilkhand 2009, Gorakhpur 05, 09)
∞ − pt
Proof: We have f ( p) = L {F (t)} = ∫0 e F (t) dt.

d ∞ − pt ∞ ∂ − pt
dp ∫ 0 ∫0
∴ f ′ ( p) = e F (t) dt = {e F (t)} dt,
∂p
by Leibnitz’s rule for differentiating under the sign of integral

=− ∫0 te − pt F (t) dt

=− ∫0 e − pt { t F (t)} dt = − L { t F (t)}.

Thus L { t F ( t)} = − f ′ ( p).

19 Multiplication by t n
Theorem: If F (t) is a function of class A and if L {F (t)} = f ( p) then
n dn
L {t F (t)} = (− 1) n f ( p), where n = 1, 2, 3,......
dp n
(Rohilkhand 2000; Lucknow 06, 09, 11; Kanpur 08;
Avadh 10; Agra 01; Gorakhpur 11)
Proof: We shall prove this theorem by mathematical induction. By article 18, we
have
d
L { t F (t)} = (− 1)1 f ( p)
dp
i.e. the theorem is true for n = 1.
M-23

Now assume that the theorem is true for a particular value of n say m.
Then, we have
m dm
L {t F (t)} = (− 1)m f ( p)
dp m
∞ dm
or ∫0 e − pt t m
F (t) dt = (− 1)m f ( p).
dp m

Now differentiating both sides w. r. t. p, we have


d ∞ − pt d m +1
dp ∫ 0
m
e t F (t) dt = (− 1)m f ( p)
dp m + 1

∞ ∂ − pt d m +1
∫0
m
or {e t F (t)} dt = ( − 1)m f ( p),
∂p dp m + 1

by Leibnitz’s rule for differentiating under the sign of integral


∞ d m +1
or −∫ e − pt ⋅ t m +1
F (t) dt = ( − 1)m f ( p)
0 dp m + 1

∞ d m +1 f ( p )
or ∫0 e − pt ⋅ { t m +1
F (t)} dt = ( − 1) m + 1
dp m +1

m +1
d m + 1 f ( p)
or L {t F (t)} = ( − 1)m + 1
dp m +1

which shows that if the theorem is true for any particular value on n, it is true for the
next value of n. But we have already seen that the theorem is true for n = 1. Hence it is
true for n = 1 + 1 = 2 and n = 2 + 1 = 3, etc.
Therefore the theorem is true for every positive integral value of n.

20 Division by t
1  ∞
Theorem: If L {F (t)} = f ( p), then L  F (t) =
t  ∫p f ( x) dx

1 
provided lim  F (t) exists.
t→0  t 
1
Proof: Let G (t) = F (t) i. e., F (t) = t G (t).
t
∴ L {F (t)} = L { t G (t)}
d
=− L { G (t)}, by the theorem of article 18
dp
d
or f ( p) = − L { G (t)}.
dp
M-24

Now integrating both sides with respect to p from p to ∞, we have


∞ ∞
− [ L { G(t)}]p = ∫p f ( p) dp

lim ∞
or −
p→ ∞
L { G (t)} + L { G (t)} = ∫p f ( p) dp

[Note that L {G (t)} is a function of p]



or 0 + L { G(t)} = ∫p f ( p) dp

 ∞ − pt 
∵ plim
 →∞
L { G (t)} = lim
p→ ∞
∫0 e G(t) dt = 0 

1  ∞
or L  F (t) =
t  ∫p f ( x) dx

Example 14: Evaluate


(i) L { t} (ii) L { e at} (Rohilkhand 2011)

(iii) L {− a sin at }
Solution: We have
L {F ′ (t)} = pL {F (t)} − F (0 ) …(1)
(i) Here let F (t) = t, then F ′ (t) = 1 and F (0 ) = 0 .
∴ from (1), we have L {1} = pL { t } − 0
1 1 1 1
or L { t } = L {1} = ⋅ = 2 , p > 0 .
p p p p

(ii) Here let F (t) = e at , then F ′ (t) = a e at and F (0 ) = 1.


∴ from (1), we have
L { a e at} = pL { e at} − 1
or a L {e at} = pL { e at} − 1
or ( p − a) L { e at} = 1.
1
∴ L { e at} = ⋅
p− a
(iii) Here let F (t) = − a sin at, then F ′ (t) = − a2 cos at and F ′ ′ (t) = a3 sin at
so that F ′ (0 ) = − a2 and F (0 ) = 0 .
2
∴ from L {F ′ ′ (t)} = p L {F (t)} − pF (0 ) − F ′ (0 ),
we have L { a3 sin at} = p2 L { − a sin at} − 0 − (− a2 )
or ( p2 + a2 ) L {− a sin at} = − a2 .
a2
∴ L {− a sin at} = − ⋅
p2 + a2
M-25

2 p ( p2 − 3 a2 )
Example 15: Show that L { t 2 cos at} = , p > 0.
( p2 + a2 )3

(Rohilkhand 2011; Kanpur 07)


2 2
Solution: Since L {cos at} = p /( p + a ), p > 0,

d2  p  d  − p2 + a2 
∴ L { t2 cos at} = (− 1)2   =  
dp2  ( p2 + a2 ) dp  ( p2 + a2 )2 

2 p ( p2 − 3 a2 )
= ⋅
( p2 + a2 )3
Example 16: Find L {(sin at − at cos at)}.
Solution: Let F (t) = sin at − at cos at, then F ′ (t) = a2 t sin at and F (0 ) = 0 .
∴ From L {F ′ (t)} = pL {F (t)} − F (0 ), we have
L { a2 t sin at} = pL{(sin at − at cos at)} − 0 .
1
∴ L {(sin at − at cos at)} = L { a2 t sin at}
p
a2 a2 d
= L { t sin at} = − L {sin at}
p p dp

a2 d  a  2 a3
=− ⋅   =
p dp  p2 + a2  ( p2 + a2 )2

Aliter. L {sin at − at cos at} = L {sin at} − a L { t cos at}


a d
= 2 − a ⋅ ( − 1) [ L {cos at}]
p + a2 dp

a d  p 
= +a  
2
p +a 2 
dp  p + a2 
2

a a (a2 − p2 ) 2 a3
= + = .
p2 + a2 ( p2 + a2 )2 ( p2 + a2 )2
Example 17: Show that
6 p 4 − 18 p 3 + 126 p 2 − 162 p + 432
L {(t2 − 3 t + 2) sin 3 t} = ⋅
( p 2 + 9)3
Solution: We have L {(t2 − 3 t + 2) sin 3 t}
= L { t2 sin 3 t} − 3 L { t sin 3 t} + 2 L {sin 3 t}
d2  d 
= ( − 1)2 L {sin 3 t} − 3 −
2
L {sin 3 t} + 2 L {sin 3 t}
dp  dp 
d2  3  d  3  3
=   +3   +2⋅
2  2   2  2
dp  p + 9  dp  p + 9 p +9
M-26

18 p2 − 54  − 6 p  6
= 2 3
+3⋅ 2 2
+ 2
( p + 9)  ( p + 9)  p + 9
6 p4 − 18 p3 + 126 p2 − 162 p + 432
= .
( p2 + 9)3

Example 18: Find L { t e − t sin t} . (Lucknow 2007)


1
Solution: Since L {sin t} = 2
p +1

d  1  2p
∴ L { t sin t} = −  2 = 2
dp  p + 1 ( p + 1)2
2 ( p + 1) 2p + 2
∴ L { t e − t sin t} = 2 2
= ⋅
[( p + 1) + 1] ( p + 2 p + 2)2
2

 sin t  −1 1  sin at 
Example 19: Prove that L   = tan and hence find L   ⋅ Does the Laplace
 t  p  t 
cos at
transform of exist?
t (Lucknow 2009; Gorakhpur 05; Rohilkhand 10; Avadh 13)

Solution: Let F (t) = sin t.


lim F (t) lim sin t
Now = = 1.
t→0 t t→0 t
1
We have L {sin t} = = f ( p), say.
p2 + 1
 sin t  ∞ ∞ dx π
∴ L = ∫p f ( x) = ∫p 2
= (tan−1 x)p∞ = − tan−1 p
 t  x +1 2
−1 −1
= cot p = tan (1/ p).
 sin at   sin at  1 −1 1
Now L  = aL  = a ⋅ tan ,
 t   at  a ( p / a)
1  p
since L { F (at)} = f   = tan−1(a / p).
p  a
p
Again, since L {cos at} = = f ( p), we have
p + a2
2

∞ ∞
 sin at  x 1 
L
 t 
= ∫p 2
x +a2
dx =  log ( x2 + a2 )
2 p
1 lim 1
= log ( x 2 + a2 ) − log ( p2 + a2 ),
2 x → ∞ 2
lim
which does not exist since log ( x2 + a2 ) is infinite.
x→ ∞
 cos at 
Hence L   does not exist.
 t 
M-27

Example 20: If L {F (t), t → p} = f ( p),


 t F (u)  1 ∞
show that L  ∫ du, t → p  = ∫ f ( y) dy .
 0 u  p p
−1
 t sin u  cot p
Hence show that L  ∫ du, t → p = ⋅
 0 u  p
Solution: From article 17, we have
 t  1
L  ∫ F (u) du = f ( p) …(1)
 0  p
where f ( p) = L {F (t)}.
F (t)
Let G (t) = .
t
 F (t) ∞
Then L { G(t)} = L 
 t 
= ∫p f ( y) dy [from article 20]

= g ( p), say.
∴ From (1), we have
 t  1
L  ∫ G (u) du = g ( p)
 0  p
 t  1 ∞
or L  ∫ F (u) du = ∫ f ( y) dy …(2)
 0  p p
Deduction. Let F (t) = sin t
1
so that f ( p) = L {sin t} = .
p2 + 1
∴ From (2), we have
 t sin u  1 ∞ dy 1
L ∫ du = ∫ 2
= [tan−1 y]∞
p
 0 u  p p y +1 p
1 π −1  1
=  − tan p = cot −1 p.
p 2  p

Comprehensive Exercise 3

Find the Laplace transforms of the following functions.


1. (i) t cos at. (Lucknow 2008) (ii) t cosh 3 t.
2
2. (i) t sin at. (Lucknow 2006) (ii) t 2 e 2 t .
6 p4 − 36 p2 + 6
3. (i) Show that L { t3 cos t} = .
( p2 + 1)4
n!
(ii) Show that L { t n . e at} = , p > a.
( p − a)n +1
M-28

4. (i) Find L { t (3 sin 2 t − 2 cos 2 t)}.


(ii) Find L.T. of f (t) = sin α t + t cos α t.
2 a ( p − a)
5. Prove that L { t e at sin at} = 2 ⋅
( p − 2 ap + 2 a2 )2 (Kanpur 2010)
√π
6. Given L {sin √ t } = e −1 /4 p , show that
2 p3 /2
 cos √ t   π  −1 / 4 p
L =   e .
 √t   p
 sinh t   t sin x 
7. Find L  ⋅ 8. Find L  ∫ dx .
 t   0 x 
9. Prove that if L {F (t)} = f ( p), then
∞ F (t) ∞
∫ 0 t dt = ∫ 0 f ( x) dx provided that the integral converges.
10. Find the Laplace transform of F (t) defined as
 t + 1, 0 ≤ t ≤ 2
F (t) = 
 3, t>2.
Also determine L { F ′ (t)}.

A nswers 3
p2 − a2 p2 + 9
1. (i) , p > 0. (ii) , p > 0.
( p2 + a2 )2 ( p2 − 9)2
2 a (3 p2 − a2 ) 2
2. (i) 2 2 3
, p > 0. (ii) .
(p +a ) ( p − 2)3
8 + 12 p − 2 p2 (α + 1) p2 + (α − 1) α2
4. (i) . (ii) .
( p2 + 4)2 ( p2 + α2 )2
1 p+1
7. log ⋅ 8. (1 / p) cot −1 p.
2 p−1
1 1 − e− 2 p
10. [1 + p − e − 2 p ], ⋅
p2 p

21 Evaluation of Integrals

If L {F (t)} = f ( p) i.e., ∫ e − pt F (t) dt = f ( p), taking limit as p → 0, we have
0

∫0 F (t) dt = f (0 )
assuming that the integral is convergent.
M-29

∞ (e − at − e − bt)
Example 21: Evaluate ∫0 t
dt.

Solution: Let F (t) = e − at − e − bt .


1 1
Then L {F (t)} = L { e − at} − L { e − bt} = − = f ( p), say.
p+ a p+ b

 F (t) ∞ ∞  1 1   ( x + a)
∴ L
 t 
= ∫p f ( x) dx = ∫p  −  dx = log
 x + a x + b 

( x + b) p

lim x+a p+ a
= log − log
x→ ∞ x+b p+ b
lim 1 + (a / x) p+ a
= log − log
x→ ∞ 1 + (b / x) p+ b
p+ a p+ b
= 0 − log = log ⋅
p+ b p+ a

 F (t) ∞ − pt e − at − e − bt p+ b
Thus L
 t 
= ∫0 e ⋅
t
dt = log
p+ a

∴ Taking limit as p → 0, we have

∞ e − at − e − bt b
∫0 t
dt = log ⋅
a

Example 22: Prove that ∫0 t3 e − t sin t dt = 0 .
(Gorakhpur 2006)
3
Solution: Let F (t) = t sin t.
d3
∴ L {F (t)} = L { t3 sin t} = (−1)3 L {sin t}
dp 3

d3  1  d2  2 p  d  2 − 6 p2 
=−   = −  −  =  
dp3  p2 + 1 dp2  ( p2 + 1)2  dp  ( p2 + 1)3 

∞ 24 ( p2 − 1) p
or ∫0 e − pt . t3 sin t dt = ⋅
( p2 + 1)4
Taking limit as p → 1, we have

∫0 t3 e − t sin t dt = 0 .
M-30

Comprehensive Exercise 4

∞ sin t π
1. Show that ∫0 t
dt = ⋅
2 (Purvanchal 2007; Meerut 13, 13B; Rohilkhand 10, 14)
∞ e− t − e− 3 t
2. Evaluate ∫0 t
dt.
(Rohilkhand 2010)
∞ 3
3. Show that ∫0 t e − 3 t sin t dt = ⋅
50
∞ 3
4. Show that ∫0 t e − 2 t cos t dt = ⋅
25
∞ e − x sin x π
5. Prove that ∫ dx = ⋅
0 x 4 (Rohilkhand 2009, 11)

6. Evaluate ∫0 t e −3 t cos 4 t dt.
(Gorakhpur 2007, 09)

A nswers 4
7
2. log 3. 6. ⋅
625

22 Periodic Functions
Fundamental Theorem: Let F (t) be a periodic function with period T > 0, that is
F (u + T ) = F (u), F (u + 2T ) = F (u), etc., then
T
∫0 e − pt F (t) dt
L {F (t)} = .
1 − e − pT
Proof: We have

L {F (t)} = ∫0 e − pt F (t) dt

T 2T 3T
= ∫0 e − pt F (t) dt + ∫T e − pt F (t) dt + ∫ 2T e − pt F (t) dt + ...
T T
= ∫0 e − pt F (t) dt + ∫0 e − p (u + T ) F (u + T ) du
T
+∫ e − p (u + 2T ) F (u + 2T ) du + ... ,
0

putting t = u + T , t = u + 2T , etc. in 2nd, 3rd, … integrals respectively


T T
= ∫0 e − pu F (u) du + e − pT ∫0 e − pu F (u) du

T
+ e −2 pT ∫ e − pu F (u) du + ...
0
M-31

T
= (1 + e − pT + e −2 pT + ...) ∫ e − pu F (u) du
0
T
∫0 e − pt F (t) dt
1 T − pu
=
1 − e − pT ∫0 e F (u) du =
1 − e − pT
.

23 Laplace Transform of Some Special Functions


1. The Sine Integral.
The sine integrals, denoted as Si (t) is defined by
t sin u
Si (t) = ∫ du,
0 u
t  u2 u4 u6 
We have Si(t) = ∫0 1 −

+ −
3! 5! 7!
+ ⋯ du

t3 t5 t7
=t− + − +⋯
3 (3 !) 5 (5 !) 7 (7 !)
1 1 1
∴ L {Si (t)} = L { t} − L {t3 } + L {t5 } − L {t7} + ⋯
3 (3 !) 5 (5 !) 7 (7 !)
1! 1 3! 1 5! 1 7!
= − ⋅ + ⋅ − ⋅ +⋯
p2 3 (3 !) p4 5 (5 !) p6 7 (7 !) p8

1 1 1 1 1 1 1 1 
=  − ⋅ 3 + ⋅ 5 − ⋅ 7 +.....
p  p 3 p 5 p 7 p 
1 1
= tan−1 , by Gregory’s series.
p p
2. The Cosine Integral.
The cosine integrals, denoted by Ci (t) is defined as
∞ cos u
Ci (t) = ∫ du.
t u
 ∞ cos u 
∴ L { Ci(t)} = L  ∫ du ⋅
 t u 
∞ cos u t cos u
Let F (t) = ∫ du = − ∫ du
t u ∞ u
cos t
so that F ′ (t) = − or t F ′ (t) = − cos t.
t
d p
∴ L {t F ′ (t)} = L { − cos t} or − L{F ′ (t)} = − 2
dp p +1
d p
or [ p f ( p) − F (0 )] = 2 , where f ( p) = L {F (t)}
dp p +1
M-32

d p
or [ p f ( p)] = 2 , since F (0 ) is constant.
dp p +1
1
Integrating, we get p f ( p) = log ( p2 + 1) + C (constant) …(1)
2
lim lim
But from the final-value theorem, p f ( p) = F (t) = 0 .
p→ 0 t→ ∞

∴ from (1) as p → 0, we have 0 = 0 + C or C = 0 .


1
∴ from (1), p f (p) = log ( p2 + 1)
2
log ( p2 + 1)
or f ( p) = L {F (t)} = L {Ci (t)} = .
2p
3. The Error Function.
The error function denoted by erf ( t ), is defined by
2 √t 2
erf (√ t) = ∫0 e − u du.
√π
2 √t 2 2 √t  u4 u6 
We have erf (√ t) = ∫0 e − u du = ∫0 1 − u2 + − + ... du
√π √π  2! 3! 
√t
2  u3 u5 u7 
= u − + − + ...
√ π  3 5 (2 !) 7 (3 !)  0

2  1 /2 t3 /2 t5 /2 t7 /2 
= t − + − + ... ⋅
√π  3 5 (2 !) 7 (3 !) 
2  1 /2 1 3 /2 1 1 
∴ L {erf √ t} =  L {t } − 3 L {t }− L { t5 /2 } − L { t7 /2 } + ...
√π 5 (2 !) 7 (3 !) 
  3  5  7  9 
Γ  Γ  Γ  Γ 
2   2   1  2  1  2 1  2 
=  3 /2 −   5 /2 + ⋅ 7 /2 − ⋅ 9 /2 + ⋯
√t  p  3 p 5 (2 !) p 7 (3 !) p 
 

2 √ π 1 √π 1 1
=  ⋅ 3 /2 − ⋅ ⋅
√π  2 p 2 2 p5 /2

√ π 1. 3 1 √ π 1. 3 . 5 1 
+ ⋅ ⋅ 7 /2 − ⋅ ⋅ 9 /2 + …
2 2 .4 p 2 2 .4 .6 p 
1  1 1 1. 3 1 1. 3 . 5 1 
= 3 /2 1 − ⋅ + ⋅ 2 − ⋅ 3 + …
p  2 p 2 . 4 P 2 .4 .6 p 
−1 / 2
1  1 1
= 1 +  = .
p3 /2  p p √ ( p + 1)
M-33

4. The Complementary Error Function.


The complementary error function denoted by erfc (√ t) is defined by
2 √t 2
erfc (√ t) = 1 − erf (√ t) = 1 − ∫0 e − x dx
√π
 2 √t 2  2  √t 2 
∴ L {erfc (√ t)} = L 1 − ∫0 e− x dx = L (1) − L  ∫ e − x dx
 √ π  √ π  0 
1 1 √ ( p + 1) − 1 {√ ( p + 1) − 1}{√ ( p + 1) + 1}
= − = =
p p √ ( p + 1) p √ ( p + 1) p √ ( p + 1) . {√ ( p + 1) + 1}
1
= ⋅
{√ ( p + 1) + 1}. √ ( p + 1)
5. The Unit Step Function (or Heaviside’s Unit function).
The unit step function, denoted by H (t − a), is H (t − a)
defined by
0 , t < a 1
H (t − a) = 
1, t ≥ a.
∞ − pt
∴ L { H (t − a)} = ∫0 e H (t − a) dt

∞ − pt  e − pt 
= ∫a
e ⋅ 1 dt = 
 − p 
 O a t
a

 lim e − pt  e − ap e − ap
= + =0 + , p> 0
t → ∞ − p  p p

e − ap
= , p > 0.
p

Note 1: If L {F (t)} = f ( p), then L {F (t − a). H (t − a)} = e − ap f ( p).

Note 2: If L {F (t)} = f ( p), then L {F ( t ). H (t − a)} = e − ap L {F (t + a)}.

6. The Exponential Integral Function.


The exponential integral function, denoted by Ei (t) is defined as
∞ e− x
Ei (t) = ∫t x
dx.

∞ e− x
Let F (t) = ∫t x
dx.

Now L{t F ′ (t)} = L{− e − t } = − L {e − t }


d 1
or − { p f ( p) − F (0 )} = −
ds p+1
d 1
or {p f ( p)} = [F (0 ) is constant]
dp p+1
or p f ( p) = log ( p + 1) + c , c is a constant.
M-34

By final value theorem, we have


lim lim
p f ( p) = F (t).
p→ 0 t→ ∞
lim lim
But [ p f ( p)] = [log ( p + 1)] + c = 0 + c = c
p→ 0 p→ 0
lim lim ∞
(e − x / x)dx = 0 .
t → ∞ ∫t
and F (t) =
t→ ∞
∴ c = 0 and thus p f ( p) = log ( p + 1)
1 1
∴ L{F (t)} = f ( p) = log ( p + 1) i. e., L {Ei (t)} = log ( p + 1).
p p
7. The Bessel Function.
Bessel function of order n is defined by
t n t2 t4 
Jn(t) = n 1 − + − ⋯
2 Γ (n + 1)  2.(2 n + 2) 2.4. (2 n + 2) (2 n + 4) 
∞ n + 2r
(− 1)r  t
= ∑ ⋅ 
r ! Γ (n + r + 1)  2 

r =0

The Bessel function of order zero i.e., J0 (t) is given by


t2 t4 t6
J0 (t) = 1 − 2
+ 2
+ ⋯.
2

22 .4 2 .42 .62 2

1 1 1
∴ L{ J0 (t)} = L {1} − 2 L {t 2 } + 2 2 L{t 4 } − 2 2 2 L {t 6 } + ⋯
2 2 .4 2 .4 .6
1 1 2! 1 4! 1 6!
= − 2 ⋅ 3 + 2 2 ⋅ 5 − 2 2 2 ⋅ 7 +⋯
p 2 p 2 .4 p 2 .4 .6 p
 2 3 
1  1  1  1. 3  1 1. 3 . 5  1
= 1−   +   −   + ⋯
p  2  p2  2 . 4  p2 
  2 .4 .6  p2 
  
 
−1 / 2
1 1 1
= 1 +  = .

p p2  √ (1 + p2 )

Example 23: Show that


a ( p2 − 2 a2 )
(i) L { sinh at cos at} =
p4 + 4 a4

2 a2 p
and (ii) L { sinh at sin at } = ⋅
p4 + 4 a4
M-35
a
Solution: We have L {sinh at} = = f ( p), say.
p2 − a2
∴ L { e iat sinh at} = f ( p − ia)
a a
= 2 2
= 2
( p − ia) − a ( p − 2 a2 ) − 2 iap
a {( p2 − 2 a2 ) + 2 iap}
=
( p2 − 2 a2 )2 − (2 ipa)2
a ( p2 − 2 a2 ) + 2 ia2 p
or L {sinh at (cos at + i sin at)} =
p4 + 4 a4
a ( p2 − 2 a2 ) 2 a2 p
or L{sinh at cos at} + i L{sinh at sin at} = +i ⋅
p4 + 4 a4 p4 + 4 a4
a ( p2 − 2 a2 )
Hence L {sinh at cos at} =
p4 + 4 a4
2 a2 p
and L {sinh at sin at} = .
p4 + 4 a4
1 3 6 6
Example 24: Show that L {(1 + t e − t )3} = + + + ⋅
p ( p + 1)2 ( p + 2)3 ( p + 3)4
Solution: We have
L {(1 + t e − t )3} = L {1 + 3 t e − t + 3 t2 e −2 t + t3 e −3 t}
d d2 d3
= L {1} + 3 (−1) L { e − t} + 3 (−1)2 2 L { e −2 t} + (−1)3 3 L {e −3 t}
dp dp dp
1 1! 2! 3!
= + 3. + 3. +
p ( p + 1)2 ( p + 2)3 ( p + 3)4
1 3 6 6
= + + + .
p ( p + 1)2 ( p + 2)3 ( p + 3)4

Aliter: L {(1 + t e − t )3} = ∫0 (1 + t e − t )3 . e − pt dt


= ∫0 [e − pt + 3 t e − ( p + 1)t + 3 t2 e − ( p + 2)t + t3 e − ( p + 3)t ] dt

∞ ∞ ∞
= ∫0 e − pt dt + 3 ∫ te − ( p + 1)t dt + 3 ∫ t2 e − ( p + 2)t dt
0 0

+ ∫0 t 3 e − ( p +3)t dt

∞ ∞
= ∫0 e − pt t1−1 dt + 3 ∫ e − ( p + 1)t t2 − 1 dt
0
∞ ∞
+3∫ e − ( p + 2)t t3 − 1 dt + ∫0 e − ( p + 3)t t 4 − 1 dt
0
1 3 6 6
= + + + , p > 0.
p ( p + 1)2 ( p + 2)3 ( p + 3)4
 ∞ − at n−1 Γ(n) 
∵ ∫ 0 e t dt = n , if a > 0 and n > 0 
a 
M-36

1
Example 25: Prove that L { J0 (t)} = and hence deduce that
√ (1 + p2 ) (Avadh 2007)
1
(i) L { J0 (at)} =
√ ( p2 + a2 )
p
(ii) L { t J0 (at)} =
( p + a2 )3 /2
2 (Rohilkhand 2002)

1
(iii) L { e − at J0 (at)} =
√ ( p2 + 2 ap + 2 a2 )

(iv) ∫0 J0 (t) dt = 1.

Solution: We know that


t2 t4 t6
J0 (t) = 1 − 2
+ 2
+ ...
2

2 2 .4 2 .42 .62 2

1 1 1
∴ L { J0 (t)} = L {1} − 2 L { t 2} + 2 2 L { t 4} − 2 2 2 L { t 6} + ...
2 2 .4 2 .4 .6
1 1 2! 1 4! 1 6!
= − 2 ⋅ 3 + 2 2 ⋅ 5 − 2 2 2 7 + ...
p 2 p 2 .4 p 2 .4 .6 p
 2 3 
1  1  1  1⋅ 3  1  1⋅ 3 ⋅ 5  1 
= 1 −  2  +   −   + ....
p  2  p  2 ⋅ 4  p2  2 ⋅ 4 ⋅ 6  p2  
 
−1 / 2
1 1 1
= 1 + 2  = ⋅
p p  √ (1 + p2 )

Deductions:
1  p
(i) Since L { F (at)} = f   where f ( p) = L { F (t)},
a  a
1 1 1
∴ L { J0 (at)} = ⋅ = .
a √ [1 + ( p / a)2 ] √ ( p2 + a2 )

d d  1  p
(ii) L { t J0 (at)} = − L{ J0 (at)} = −  = 2
dp dp  √ ( p + a ) ( p + a2 )3 /2
2 2

(iii) Since L { e − at F (t)} = f ( p + a) where f ( p) = L {F (t)},

1  1 
∴ L { e − at J0 (at)} = 2 2 ∵ L { J0 (at)} = 2 2 
√ [( p + a) + a ]  √ ( p + a )
1
= .
√ ( p + 2 ap + 2 a2 )
2

∞ 1
(iv) We have L { J0 (t)} = ∫0 e − pt J0 (t) dt = .
√ (1 + p2 )

∴ Putting p = 0, we have ∫0 J0 (t) dt = 1.
M-37

p
Example 26: Prove that L { J1 (t)} = 1 − where J1 (t) is the Bessel function of order
√ ( p 2 + 1)
1
one and hence deduce that L {t J1 (t)} = .
( p + 1)3 /2
2
(Kanpur 2009)
Solution: We know that J0 ′ (t) = − J1 (t).
∴ From L {F ′ (t)} = pL {F (t)} − F (0 ), we have
L { J1 (t)} = L { − J0 ′ (t)} = − L { J0 ′ (t)} = − [ pL { J0 (t)} − J0 (0 )]
 1 
= −p ⋅ 2 
 √ ( p + 1) − 1
 1 
∵ from Ex . 25, L { J0 (t)} = 2
and J0 (0 ) = 1
 √ (1 + p ) 
p
= 1− ⋅
√ ( p2 + 1)
Aliter: We have
∞ n +2 r
(− 1)r  t
Jn (t) = ∑ ⋅ 
r ! Γ (n + r + 1)  2 
.
r =0
∞ 2 r +1
(− 1)r 1 t3 t5
∑ r !(r + 1) ! ⋅  2
t t 1
∴ J1(t) = = − 2 + 2 2 ⋅ − ...
r =0
2 2 4 2 .4 6
1 1 1
∴ L { J1 (t)} = ⋅ L { t } − 2 L { t3} + 2 2 L { t5} − ...
2 2 .4 2 .4 .6
1 1! 1 3! 1 5!
= ⋅ 2 − 2 ⋅ 4 + 2 2 ⋅ 6 − ...
2 p 2 .4 p 2 .4 .6 p
 2 3 
1  1  1⋅ 3  1  1⋅ 3 ⋅ 5  1 
= 1 − 1 −  2  +   −
 
  + ...
 
 2  p  2 ⋅4  p 
2 2
2 ⋅4 ⋅6  p  
 
−1 / 2
 1 p
= 1 − 1 + 2  = 1− 2
 p  √ ( p + 1)
d
Deduction: L { t J1 (t)} = − L { J1 (t)}
dp

d  p  1
=− 1 −  = .

dp  2 
√ ( p + 1) ( p + 1)3 /2
2

Example 27: Find L { erf √ t } and hence prove that


3p + 8
L { t ⋅ erf (2 √ t)} = 2 .
p ( p + 4)3 /2 (Avadh 2012)
Solution: We know that
2 √t 2
erf (√ t) = ∫0 e − u du
√π
M-38

2 √t  u4 u6 
=
√π ∫0 1 − u2 +
 2 !

3 !
+ … du

√t
2  u3 u5 u7 
= u − + − + ...
√π  3 5 (2 !) 7 (3 !) 0

2  1 /2 t3 /2 t5 /2 t7 /2 
= t − + − + ...
√π  3 5 (2 !) 7 (3 !) 

2  1 /2 1 3 /2 1 1 
∴ L {erf √ t} =  L {t } − L {t } − L { t5 /2 } − L { t7 /2 } + ...
√π  3 5 (2 !) 7 (3 !) 
2  Γ( 3 )  1 Γ( 5 ) 1 Γ( 7 ) 1 Γ( 9 ) 
=  32/2 −   52/2 + ⋅ 72/2 − ⋅ 92/2 + ...
√π  p  3 p 5 (2 !) p 7 (3 !) p 

2 √ π 1 √π 1 1 √ π 1⋅ 3 1 √ π 1⋅ 3 ⋅ 5 1 
=  ⋅ 3 /2 − + ⋅ − ⋅ + ...
√π  2 p 2 2 p5 /2 2 2 ⋅ 4 p7 /2 2 2 ⋅ 4 ⋅ 6 p9 /2 
−1 / 2
1  1 1 1⋅ 3 1 1⋅ 3 ⋅ 5 1  1  1
= 3 /2 
1− ⋅ + 2
− 3
+ ... = 3 /2 1 + 
p  2 p 2 ⋅ 4 p 2 ⋅4 ⋅6 p  p  p
1
= ⋅ (Rohilkhand 2004)
p √ ( p + 1)
1  p
Since L { F (at)} = f   , where f ( p) = L { F (t)},
a  a
∴ L {erf (2 √ t)} = L {erf √ (4 t)}
1 1 2
= ⋅ = ⋅
4 p p  p √ ( p + 4)
 + 1
4 4 
d
Hence L { t. erf (2 √ t)} = − L {erf (2 √ t)}
dp
d  2  3p + 8
=−  = ⋅
dp  p √ ( p + 4) p2 ( p + 4)3 /2

Example 28: Find the Laplace transform of Si (t). (Rohilkhand 2009)

Solution: We know that


t sin u t  u2 u4 u6 
Si (t) = ∫0 u
du = ∫0 1 −

+ −
3! 5! 7!
+ ... du

t3 t5 t7
=t− + − + ...
3 (3 !) 5 (5 !) 7 (7 !)
1 1 1
∴ L { Si (t)} = L { t } − L { t 3} + L { t 5} − ⋅ L { t7} + ...
3 (3 !) 5 (5 !) 7 (7 !)
M-39

1! 1 3! 1 5! 1 7!
= 2
− ⋅ 4 + ⋅ 6 − ⋅ 8 + ...
p 3 (3 !) p 5 (5 !) p 7 (7 !) p

1 1 1 1 1 1 1 1 
=  − ⋅ 3 + ⋅ 5 − ⋅ 7 + ...
p  p 3 p 5 p 7 p 
1 1
= tan−1 , by Gregory’s series.
p p

Example 29: Find L { Ci (t)}. (Rohilkhand 2003, 08)

 ∞ cos u 
Solution: L { Ci (t)} = L  ∫ du ⋅
 t u 
∞ cos u t cos u
Let F (t) = ∫ du = − ∫ du
t u ∞ u
cos t
so that F ′ (t) = − or t F ′ (t) = − cos t.
t
∴ L { t F ′ (t)} = L { − cos t}
d p
or − L{F ′ (t)} = − 2
dp p +1

d p
or [ p f ( p) − F (0 )] = 2 where f ( p) = L {F (t)}
dp p +1

d p
or [ p f ( p)] = 2 , since F (0 ) is constant.
dp p +1
1
Integrating, p f ( p) = log ( p2 + 1) + C (constant) …(1)
2
But from the final-value theorem article 16,
lim lim
p f ( p) = F (t) = 0 .
p→ 0 t→ ∞
∴ from (1) as p → 0 we have
0 = 0 + C or C = 0 .
1
∴ from (1), p f ( p) = log ( p2 + 1)
2
or f ( p) = L {F (t)} = L { Ci (t)}
log ( p2 + 1)
= .
2p
3 t, 0 < t < 2
Example 30: If F (t) = 
6, 2 < t < 4,
find L {F (t)} where F (t) has period 4.
Solution: Here F (t) is a periodic function with period T = 4.
∴ from article 22 theorem, we have
M-40

T
∫0 e − pt F (t) dt
L {F (t)} =
1 − e − pT
2 4
∫0 3 t e − pt dt + ∫2 6 . e − pt dt
=
1 − e− 4 p
2 4
1  e − pt   e − pt   1 6 e − pt 
=   ⋅ 3 t −  2  ⋅ 3 +  
1 − e −4 p  − p   p  0 1 − e −4 p  − p 2

1  6 −2 p 3 e −2 p 3 6 6 
= −4 p − e − 2
+ 2 − e −4 p + e −2 p 
1− e  p p p p p 
3 − 3 e− 2 p − 6 p e− 4 p
= .
p2 (1 − e − 4 p )

24 The Dirac-delta Function or Unit-impulse Function


In applied mechanics we frequently come across
incidences when a very large force acts for a very short
1
time. We deal with such and similar ideas by a new ε
function called the unit impulse function or the Dirac-delta δε (t−a)
function.
The unit impulse function is the limiting form of the
function O t=a t=a+ε
1 / ε, if a ≤ t ≤ a + ε
δ ε (t − a) = 
 0 , otherwise.
From the figure it is clear that the height of the strip value of the function increases
indefinitely and the width decreases as ε → 0. In this case the area of the rectangle is
always unity.
Thus, the unit impulse function denoted by δ(t − a) is defined as :
 ∞, if t = a
δ(t − a) = 
 0 , if t ≠ a.

such that the area of the strip = ∫0 δ(t − a) dt = 1. [For a ≥ 0]

Illustration: The case of a load ω0 acting at the point x = a of a beam may be considered
as the limiting case of uniform loading ω0 / ε per unit length over the portion of the
beam between x = a and x = a + ε. Thus, we have
ω0 / ε, a < x < a + ε
ω( x) = 
 0 , otherwise

= ω0 δ ( x − a).
M-41

Laplace Transform of Unit Impulse Function: If F (t) be a function of t continuous at


t = a, then we have
∞ a+ε  1
∫0 F(t) δ ε (t − a) dt =∫a F(t) .  ε dt
 1
= (a + ε − a) F (η) .   , where a < η < a + ε.
 ε
[∵ By Mean value theorem for integrals
b
∫a f (t) dt = (b − a) f (η),where a < η < b]

= F(η). ...(1)

Theorem 1: ∫0 F (t) δ (t − a) dt = F (a).

Proof: From (1), as ε → 0, we get ∫ F (t) δ(t − a) dt = F (a).
0

Particular Case: If F (t) = e − pt , then L{δ (t − a)} = e − ap . (Note)



Theorem 2: ∫−∞ F (t) δ′ (t − a) dt = − F ′ (a).

Proof: We have

[ ]
∞ ∞ ∞
∫−∞ F (t) δ ′ (t − a)dt = F (t) . δ(t − a)
−∞
− ∫−∞ F ′ (t) δ(t − a) dt

= 0 − 0 − F ′ (a) = − F ′ (a).

Example 31: Find the Laplace transform of t3 δ(t − 4).


∞ − pt 3
Solution: We have L{t3 δ(t − 4)} = ∫0 e t δ(t − 4) dt = 43 e − 4 p


Example 32: Evaluate ∫− ∞ e − 5 t δ(t − 2) dt.


Solution: We have ∫− ∞ e −5 t δ(t − 2) dt = e −5 × 2 = e −10 .

Comprehensive Exercise 5

  t 1 1  1 
1. Given L 2    = 3 /2 , show that 1 /2 = L  ⋅
  
π  p p  √ (πt)
2. Find (i) L {F (t)} and (ii) L {F ′ (t)}, for the function given by
2 t , 0 ≤ t ≤ 1
F (t) = 
 t, t > 1.
M-42

25 30 9
3. Show that L {(5 e2 t − 3)2} = − + , p > 4.
p−4 p−2 p
 sin2 t  1  p2 + 4 
4. Show that L   = log  2  ⋅ (Rohilkhand 2010; Kashi 14)
 t  4  p 
∞ π sin2 t
5. Show that ∫ dt = .
t2 0 2 (Rohilkhand 2003)
∞ cos 6 t − cos 4 t  2
6. Show that ∫ dt = log   ⋅
0 t  3
2
7. Prove that L { J0 (a √ t)} = (1 / p) e − (a /4 p)
.
2
8. If F (t) = t , 0 < t < 2 and F (t + 2) = F (t), find L {F (t)}.
[Hint. Here F (t) is a periodic function with period T = 2]
sin t, 0 < t < π
9. Compute L {F (t)}, if F (t) =  where F (t) has period 2π.
0 , π < t < 2 π,
10. Find the Laplace transform of the Heaviside’s unit step function H (t − a).
t 1 − e −2 x 
11. Find Laplace transform of ∫   dx.
0 x 
∞ e− u log ( p + 1)
12. If E (t) = ∫t u
du, show that L { E (t)} =
p


13. Evaluate ∫0 e − t erf √ t dt.

 δ( t − 4)
14. Evaluate (i) L  (ii) L{ e − 4 t δ( t − 3)}.
 t 
15. Evaluate (i) L{cos t log t δ( t − π)} (ii) L{ t3 δ( t − 5)}.
∞ ∞
16. Evaluate (i) ∫−∞ e − 3 t δ( t − 4)dt (ii) ∫−∞ e − 3 t δ ′( t − 2)dt.

A nswers 5
2 e −p
2. (i) 2 / p2 − (1/ p + 1/ p2 ) e − p (ii) −
p p
− (4 p2 + 4 p + 2) e −2 p + 2 1
8. −2 p
9.
3
p (1 − e ) ( p + 1) . (1 − e − pπ )
2

e − ap 1  2
10. 11. log 1 + 
p p  p
13. 1/ √ 2
e − 4p −3( p +4 )
14. (i) (ii) e
4
15. (i) − log πe − pπ (ii) 125 e −5 t
16. (i) e −12 (ii) 3 e −6
M-43

25 Table of Laplace Transform Theorems

Laplace Transform Theorems

No. Operation F (t ) L { F (t )} = f ( p)

1. Linearity property a1 F1 (t) + a2 F2 (t) a1 L{F1(t)} + a2 L{F2 (t)}

2. First translation e at F (t) f ( p − a)


or shifting theorem

3. Second translation  F (t − a), t > a e − ap f ( p)


G(t) = 
or shifting theorem  0 , t< a

4. Change of scale 1  p
F (at) f  
property a  a

F ′ (t) pf ( p) − F (0 )
5. Differentiation
theorems n−1
F n (t) pn f ( p) − ∑ p n −1− r F r (0 )
r =0

t F (t) − f ′ ( p)
6. Multiplication
theorems n
t F (t) dn
(− 1)n n
f ( p)
dp

1 ∞
7. Division theorem
t
F (t) ∫p f ( x) dx

t 1
8. Integral theorem ∫0 F ( x) dx
p
f ( p)

lim lim
9. Initial value theorem F (t) = p L {F (t)}
t→0 p→ ∞

lim lim
10. Final-value theorem F (t) = p L {F (t)}
t→ ∞ p→ 0
T
∫0 e − pt F (t) dt
11. Fundamental L {F (t)} = ,
1 − e − pT
theorem for
F (t) is periodic function of period T
Periodic Functions
M-44

Laplace Transforms of Some Special Functions

Function F (t) L { F (t)}

t sin u 1 1
1. Sine integral Si (t) = ∫0 du tan−1
u p p

2. Cosine integral ∞ cos u 1


Ci (t) = ∫t du log ( p2 + 1)
u 2p

3. Error function 2 √t 2 1
erf (√ t) = ∫0 e − u du
√π p √ ( p + 1)

4. Complementary error erfc (√ t) = 1 − erf (√ t) 1


function 2 √ t − x2 {√ ( p + 1) + 1} . √ ( p + 1)
√ π ∫0
= 1− e dx

5. Heaviside’s unit step 0 , t< a e − ap


H (t − a) =  , p> 0
function 1, t ≥ a. p

6. Dirac-delta function 1 / ε, 0 ≤ t ≤ ε 1
(1 − e − pε )
Fε (t) = 
 0, t > ε pε

7. Exponential integral ∞ e− x 1
log ( p + 1)
function
Ei (t) = ∫t x
dx
p

8. Bessel function of t2 t4
J0 (t) = 1 − 2
+
order zero 2 2 . 42
2 1
t6 √ (1 + p 2 )
− +…
2 . 42 . 62
2

Objective Type Questions

Multiple Choice Questions


Indicate the correct answer for each question by writing the corresponding letter from (a),
(b), (c) and (d).
1. The Laplace transform of F (t) exists for all p > a, if
(a) F (t) is a continuous function
(b) F (t) is a differentiable function
(c) F (t) is a function of class A
(d) None of these. (Rohilkhand 2002)
M-45

2. The Laplace transform of F (t) = 1 is


(a) p (b) 1
1 1
(c) , p> 0 (d) , p > 1.
p p−1 (Rohilkhand 2008)

 F (t − a), t > a
3. If L { F (t)} = f ( p) and G (t) =  then L { G (t)} is
 0 , t< a
(a) f ( p − a) (b) e − ap f ( p)
1  p
(c) f   (d) f (ap). (Rohilkhand 2003)
a  a
4. If L { F (t)} = f ( p) and F ′ (t) is of class A, then L { F ′ (t)} is
(a) pf ( p) − F (0 ) (b) p2 f ( p)
(c) f ( p) − F (0 ) (d) None of these.
(Rohilkhand 2003)
5. The Laplace transform of cosh at is
p p
(a) (b)
p − a2
2
p + a2
2

1 1
(c) (d) ⋅ (Rohilkhand 2003)
p − a2
2
p2 + a2
6. If L { F (t)} = f ( p) then L { t F (t)} is
(a) f ′ ( p) (b) − f ′ ( p)
∞ 1
(c) ∫p f ( x) dx (d)
p
f ( p). (Rohilkhand 2002)

sin t
7. The value of Laplace transform of is
t
1 2
(a) tan−1 (b) tan−1
p p
(c) tan−1 p (d) None of these.

8. If L { F (t)} = f ( p) then L { F (at)} is


 p 1
(a) f   (b) f ( p)
 a a
1  p
(c) f   (d) None of these.
a  a
2
9. The Laplace transform of e t is
1 1
(a) (b)
p−2 ( p − 2)2
1
(c) (d) not existing.
p2
(Rohilkhand 2003)
M-46

 sin t  −1 1  sin 3 t 
10. If L   = tan , then L   is
 t  p  t 
1  3  3
(a) tan−1   (b) tan−1  
3  p  p
(c) tan−1 (3 p) (d) 3 tan−1 (3 p).
11. The value of L { t sin at} is
a 2 ap
(a) 2 2
(b)
(p +a ) ( p + a2 )
2

2 ap
(c) (d) None of these.
( p + a2 )2
2


12. The value of ∫ J0 (t) dt is
0

(a) 1 (b) 0
(c) −1 (d) None of these.
∞ sin t
13. The value of ∫ dt is
0 t
π
(a) 0 (b)
2
(c) 1 (d) None of these.
1
14. If L {erf √ t} = then L { e3 t erf √ t} is
p √ ( p + 1)
1 1
(a) (b)
( p − 3) √ ( p − 2) ( p − 3) √ ( p + 2)
3
(c) (d) None of these.
( p − 3) √ ( p − 2)

Fill in the Blank(s)


Fill in the blanks “……” so that the following statements are complete and correct.

1. If the integral f ( p) = ∫−∞ K ( p, t) F (t) dt is convergent, then f ( p) is called

the …… of the function F (t) and the function K ( p, t) is called the …… of the
transformation.
∞ − pt
2. The integral ∫0 e F (t) dt is called the …… transform of the function F (t).

3. The Laplace transform of the function F (t) = cos at is …… .


4. If f ( p) is the Laplace transform of F (t) then f ( p − a) is the Laplace transform of
…… .
n
5. If F (t) is a function of class A and if L { F (t)} = f ( p), then L { t F (t)} = ……
where n = 1, 2 , 3, …… .
M-47

6. The value of L { J0 (t)} is …… .


∞ sin2 t
7. The value of integral ∫ dt is …… .
0 t2
8. If F (t) is a periodic function with period T > 0 then F (u + T ) = …… .

True or False
Write ‘T’ for true and ‘F’ for false statement.
1. The Laplace transformation is a linear transformation.
2
2. The function e t is not of exponential order as t → ∞.
3. If the Laplace transform of a function F (t) exists then it must be of class A.

4. The Laplace transform of the function F (t) = t n, n being any real number greater
Γn
than −1 is n + 1 , p > 0 .
p

5. If F (t) is a function of class A and if L { F (t)} = f ( p), then


L { t F (t)} = f ′ ( p).
1
6. The value of L { e at} = ⋅
p+ a
λ
7. The value of L {sin λt . cos λt} = , p > 0.
p + 4 λ2
2

6
8. The value of L { t3 e −3 t } = ⋅
( p + 3)4

A nswers
Multiple Choice Questions
1. (c) 2. (c) 3. (b)
4. (a) 5. (a) 6. (b)
7. (a) 8. (c) 9. (d)
10. (b) 11. (c) 12. (a)
13. (b) 14. (a)

Fill in the Blank(s)


1. integral transform ; kernel. 2. Laplace.
p
3. , p> 0 4. e at F (t)
p + a2
2

dn 1
5. (− 1)n f ( p) 6.
n
dp √ (1 + p2 )
M-48

π
7. 8. F (u)
2

True or False
1. T 2. T
3. F 4. F
5. F 6. F
7. T 8. T

¨
M-49

2
T he I nverse L aplace T ransform

1 Inverse Laplace Transform


f f ( p) is the Laplace Transform of a function F (t), i. e. L {F (t)} = f ( p), then F (t) is
I called the inverse Laplace transform of the function f ( p) and is written as
F (t) = L−1 { f ( p)}.

L−1 is called the inverse Laplace transformation operator.

2 Null Function
t
If N (t) is a function of t such that ∫ N (t) dt = 0 for all t > 0 then N (t) is called a null
0

function.

3 Lerch’s Theorem
If F1 (t) and F2 (t) are two functions having the same Laplace transform f ( p), then
F1 (t) − F2 (t) = N (t) where N (t) is a null function for all t > 0.
From this it follows that, an inverse Laplace transform is unique except for the addition of a null
function.
M-50

If we restrict ourselves to functions F (t) which are sectionally continuous in every finite
interval 0 ≤ t ≤ t0 and of exponential order for t > t0 , then the inverse Laplace transform of
f ( p) i.e., L−1 { f ( p)} = F (t) is unique.

4 Linearity Property
Theorem: Let f1 ( p) and f2 ( p) be the Laplace transforms of functions F1 (t) and F2 (t)
respectively and c1, c2 be two constants, then L−1 { c1 f1 ( p) + c2 f2 ( p)}
= c1 L−1 { f1 ( p)} + c2 L−1 { f2 ( p)} = c1 F1 (t) + c2 F2 (t).(Avadh 2014)

Proof: We have
L { c1 F1 (t) + c2 F2 (t)} = c1 L {F1 (t)} + c2 L {F2 (t)}
= c1 f1( p) + c2 f2 ( p).
−1
∴ L { c1 f1 ( p) + c2 f2 ( p)} = c1 F1 (t) + c2 F2 (t)
= c1 L−1 { f1 ( p)} + c2 L−1 { f2 ( p)}.

5 Inverse L.T. of Some Special Functions


(Rohilkhand 2006)
(i) Inverse L.T. of 1/p, p > 0.
∵ L {1} = 1/ p, p > 0 , ∴ L−1 {1/ p} = 1, p > 0 .
(ii) Inverse L.T. of 1 / pn + 1, p > 0, n > − 1.
Γ (n + 1)
∵ L { t n} = , p > 0, n > − 1
p n+1
∴ L−1 {1/ p n+1} = t n / Γ (n + 1), n > − 1.
If n is a positive integer, then Γ (n + 1) = n !.
∴ L−1 {1/ pn+1} = t n / n !, p > 0 , n is a positive integer.
(iii) Inverse L.T. of 1/( p − a), p > a.
∵ L { e at} = 1/( p − a), p > a, ∴ L−1{1/( p − a)} = e at , p > a.
(iv) Inverse L.T. of 1 /( p2 + a2 ), p > 0.
a  p  1
∵ L {sin at} = 2 2
, p > 0, ∴ L−1  2 2
= sin at
p +a  p + a  a
(v) Inverse L.T. of p /( p + a2 ), p > 0.
2

p  p 
∵ L {cos at} = 2 2
, ∴ L−1  2  = cos at.
p +a  p + a2 
(vi) Inverse L.T. of 1 /( p − a2 ), p > | a | .
2

a  1  1
∵ L {sinh at} = 2 2
, ∴ L−1  2 2
= sinh at.
p −a  p − a  a
M-51

(vii) Inverse L.T. of p /( p2 − a2 ), p > | a | .


p  p 
∵ L {cosh at} = , ∴ L−1  2 2
= cosh at .
p2 − a2  p − a 

6 Table of Some Inverse Laplace Transforms


f ( p) L−1 { f ( p)} = F(t )

1
1. 1
p

1 tn
2. n +1
, n is a positive integer
p n!

1
3. e at
p− a

1 1
4. sin at
p2 + a2 a

p
5. cos at
p + a2
2

1 1
6. 2 2
sinh at
p −a a

p
7. cosh at
p − a2
2

1 tn
8. n +1
,n> −1
p Γ (n + 1)

If we know the inverse L.T. given in the above table then nearly all the inverse Laplace
transforms can be obtained by using the general theorems, which we shall give later on.

 1   1 
Example 1: Find (i) L−1  4  (ii) L−1  2 .
 p   p + 4  (Meerut 2013B)
4 −1
 1  t t3
Solution: (i) L−1  4  = =
 p  3! 6

 1   1  1
(ii) L−1  2 =L
−1
 2 2
= sin 2 t
 p + 4   p + 2  2
M-52

Example 2: Prove that


 2 
−1  5  √ p − 1  7   t  7 −2 t / 3
L +
 2  −  = 1 + 6 t − 4   − e .
p p  3 p + 2 π 3
   
 
 2 
−1  5  √ p − 1  7 
Solution: We have L  2 + − 
 p  p  3 p + 2
 
 5 p− 2 √ p+1 7 1 
= L−1  2 + 2
− . 
3 p + (2 / 3)
 p p 
 1   1  1  7 −1  1 
= 6 L−1  2  + L−1   − 2 L−1  3 /2  − L  
 p   p  p  3  p + (2 / 3)
t 2 −1 t 3 /2 −1 7
=6 + 1− 2 . − e −2 t /3
1! Γ (3 / 2) 3

= 6 t + 1 − 4 √ (t / π) − (7 / 3) e −2 t /3 .

Example 3: Show that


1 1 t3 t5 t7
L−1  sin  = t − 2
+ 2
− + ...
 p p (3 !) (5 !) (7 !)2 (Avadh 2014)

1 1  1  1 (1/ p)3 (1/ p)5 (1/ p)7  


Solution: L−1  sin  = L−1   − + − + ... 
 p p  p  p 3! 5! 7!  

 1  1 −1  1  1 −1  1  1 −1  1 
= L−1  2  − L  4 + L  6 − L  8  + ...
 p  3 !  p  5 !  p  7 !  p 

t3 t5 t7
=t− 2
+ 2
− + ... .
(3 !) (5 !) (7 !)2

Comprehensive Exercise 1

Find the inverse Laplace transforms of each of the following functions.


p 6p 3
1. 4 /( p − 2). 2. 2 + 2 + ⋅
p + 2 p − 16 p − 3
2p + 1
3. ⋅
p ( p + 1) (Purvanchal 2014; Kashi 14, 14B)
2p − 5
4. ⋅
p2 − 9 (Meerut 2013)
3p − 2 7 6 3 + 4p 8 − 6p
5. − ⋅ 6. − + ⋅
p5 /2 3p + 2 2 p − 3 9 p2 − 16 16 p2 + 9
M-53

3 3p + 2 3 p − 27 6 − 30 √ p
7. + − + ⋅
p2 − 3 p3 p2 + 9 p4
3 ( p2 − 1)2 4 p − 18 ( p + 1) (2 − √ p)
8. 5
+ 2
+ ⋅
2p 9− p p5 /2
1 1 t2 t4 t6
9. Show that L−1  cos  = 1− + − + ...
 p p (2 !)2 (4 !)2 (6 !)2

A nswers 1
1. 4 e 2 t. 2. cos √ 2 t + 6 cosh 4 t + 3 e 3 t .

3. e − t + 1. 4. 2 cosh 3 t − (5 / 3) sinh 3 t.

 t 8  t 7
5. 6   − t   − e −2 t / 3 .
 π 3  π 3
1 4t 4 4t 2 3t 3 3t
6. 3 e 3 t /2 − sinh − cosh + sin − cos ⋅
4 3 9 3 3 4 8 4
7. √ 3 sinh (√ 3 t) + 3 t + t 2 − 3 cos 3 t + 9 sin 3 t + t 3 − 16 t 2 √ (t / π).
1 3 2 1 4 8
8. − t + t − 4 cosh 3 t + 6 sinh 3 t + 4 √ (t / π) + t √ (t / π) − t.
2 2 16 3

7 First Translation or Shifting Theorem


If L−1 { f ( p)} = F (t), then L−1 { f ( p − a)} = e at F (t) = e at L−1 { f ( p)}.
∞ − pt
Proof: We have f ( p) = ∫0 e F (t) dt.

∞ ∞ − pt
∴ f ( p − a) = ∫0 e −(p − a)t F (t) dt = ∫0 e .{ e at ⋅ F (t)} dt = L { e at F (t)}.

∴ L−1 { f ( p − a)} = e at F (t) = e at L−1 { f ( p)}.

8 Second Translation or Shifting Theorem


 F (t − a), t> a
If L−1 { f ( p)} = F (t), then L−1 { e − ap f ( p)} = G (t), where G (t) = 
 0, t < a.

Proof: We have f ( p) = ∫0 e − pt F (t) dt .

∞ − p (t + a)
∴ e − ap f ( p) = ∫0 e F (t) dt
M-54

∞ − px
= ∫0 e F ( x − a) dx, putting t + a = x, so that dt = dx

a − px ∞ − px
= ∫0 e ⋅ 0 dx + ∫a e F ( x − a) dx

a − pt ∞ − pt
= ∫0 e ⋅ 0 dt + ∫a e F (t − a) dt

∞ − pt
= ∫0 e G (t) dt = L { G (t)} ,

 F (t − a), t > a
where G (t) = 
 0, t < a.
Note: In terms of Heaviside’s unit step function H (t − a) this theorem can be stated as :
If L−1 { f ( p)} = F (t),
then L−1 { e − ap f ( p)} = F (t − a). H (t − a).

9 Change of Scale Property


Theorem: If L−1 { f ( p)} = F (t), then L−1 { f (ap)} = (1 / a) F (t / a).
∞ − pt
Proof: We have, f ( p) = ∫0 e F (t) dt .

∞ − apt 1 ∞ − px  x 
∴ f (ap) = ∫0 e F (t) dt =
a ∫0
e F   dx .
 a
putting at = x, so that dt = (1 / a) dx
1 ∞ − pt  t 1   t  1  t 
a∫
= e F   dt = L  F    = L F    .
0  a a   a   a  a 
Hence L−1 { f (ap)} = (1 / a) F (t / a).

10 Use of Partial Fractions


g ( p)
If f ( p) is of the form , where g and h are polynomials in p, then break f ( p) into
h ( p)
partial fractions and manipulate term by term.

 3 p + 7 
Example 4: Evaluate L−1  2 .
 p − 2 p − 3  (Bundelkhand 2013)

 3 p + 7   3 ( p − 1) + 10 
Solution: L−1  2 =L
−1
 
 p − 2 p − 3   ( p − 1)2 − 4 
M-55

 3 ( p − 1) 10 
= L−1  2
+ 2 
 ( p − 1) − 4 ( p − 1) − 4 
 p−1   1 
= 3 L−1  2  + 10 L
−1
 2 
 ( p − 1) − 4   ( p − 1) − 4 
 p   1 
= 3 e t L−1  2 2
+ 10 e t L−1  2 
 p − 2   p − 22 

= 3 e t cosh 2 t + 5 e t sinh 2 t = 4 e 3 t − e − t .
−1 / p −a / p 
 e  cos 2 √ t  e 
Example 5: If L−1  1 /2  = , find L−1  1 /2  , where a > 0.
 p  √ (πt)  p 
−1 / p 
 e  cos 2 √ t
Solution: Since L−1  1 /2  = ,
 p  √ (πt)

 e −1 / pk  1 cos 2 √ (t / k )  e −1 / pk  cos 2 √ (t / k )
∴ L−1  1 /2 
= or L−1  1 /2  = .
 ( pk )  k √ (πt / k )  p  √ (πt)

 e − a / p  cos 2 √ (at)
Taking k = 1 / a, we have L−1  1 /2  = ⋅
 p  √ (πt)

Example 6: Find a function F(t) for which


 3 4 e − p 4 e −3 p 
F (t) = L−1  − 2 + ⋅
 p p p2 
 3 4 e − p 4 e −3 p 
Solution: F (t) = L−1  − 2 + 
 p p p2 
 1  e − p   e −3 p 
= 3 L−1   − 4 L−1  2  + 4 L−1  2 ⋅ …(1)
 p  p   p 
Now L−1 {1 / p} = 1, L−1 {1 / p2 } = t.
∴ L−1 { e − p / p2 } = (t − 1) H (t − 1),
L−1 { e −3 p / p2 } = (t − 3) H (t − 3).
Putting in (1), we have
F (t) = 3 − 4 (t − 1) H (t − 1) + 4 (t − 3) H (t − 3).
Example 7: Find the inverse Laplace transform of the following function.
p+1
2
.
p ( p + 4 p + 8)
 p+1   1 p−4 
Solution: L−1  2 =L
−1
 − 2 ,
 p ( p + 4 p + 8)  8 p 8 ( p + 4 p + 8)
resolving into partial fractions
M-56

1 −1  1 1 −1  ( p + 2) − 6 
= L  − L  
8  p 8  ( p + 2)2 + 4 

1 1 −2 t −1  p − 6 
= − e L  2 
8 8  p + 4 

1 1 −2 t  −1  p   1  
= − e L  2 2
−6 L−1  2 2 
8 8   p + 2   p + 2  
1
= [1 − e −2 t (cos 2 t − 3 sin 2 t)].
8
 p  2 t 1
Example 8: Prove that L−1  4 2 = sinh . sin . √ 3 t.
 p + p + 1 √ 3 2 2
(Kanpur 2010; Meerut 13B; Kashi 14; Rohilkhand 14)

 p  
−1 
p 
Solution: We have L−1  4 2 =L  2 2 2
 p + p + 1  ( p + 1) − p 

 p 
= L−1  2 2 
 ( p − p + 1) ( p + p + 1)

 1 ( p2 + p + 1) − ( p2 − p + 1)
= L−1  
 2 ( p2 − p + 1) ( p2 + p + 1) 

 1 1 
= L−1  2
− 2 
 2 ( p − p + 1) 2 ( p + p + 1)

   
1 −1  1  1 −1  1 
= L  − L
2 1 2 3 2  1 2 3
( p − ) +  ( p + ) + 
 2 4  2 4
   
1 t / 2 −1  1  1 − t / 2 −1  1 
= e L  − e L  2 
2 1 1
 p + ( √ 3)  2
2 2 2
 p + ( √ 3) 
 2   2 
1 t /2 2  t 1 2  t
= e sin  √ 3  − e − t /2 . sin  √ 3 
2 √3  2 2 √3  2
1  t 2 t  t
= (e t /2 − e − t /2 ) sin  √ 3  = sinh sin  √ 3  ⋅
√3  2 √3 2  2
 1 
Example 9: Evaluate (i) L−1  2 2
,
 ( p + 4) ( p + 1)  (Meerut 2013)

 3 p3 − 3 p2 − 40 p + 36 
(ii) L−1  ⋅
 ( p2 − 4)2 
M-57

 1 
Solution: (i) We have L−1  2 2
 ( p + 4) ( p + 1) 

 2 1 2 p + 3 
= L−1  + 2
− 
 25 ( p + 1) 5 ( p + 1) 25 ( p2 + 4)

1  −1  1   1  
−1 
= 2 L   +5L  2 
25   p + 1  ( p + 1)  
 p   1  
− 2 L−1  2  −3 L
−1
 2 
 p + 4   p + 4  
1
= [2 e − t + 5 e − t L−1 {1/ p2 } − 2 cos 2 t − (3 / 2) sin 2 t]
25
1 −t
= [e (2 + 5 t) − 2 cos 2 t − (3 / 2) sin 2 t ].
25
 3 p3 − 3 p2 − 40 p + 36 
(ii) L−1  
 ( p2 − 4)2 
 3 p3 − 3 p2 − 40 p + 36 
= L−1  2 2 
 ( p − 2) ( p + 2) 

 2 3 5 
= L−1  − 2
+ + 2 
 ( p − 2) p + 2 ( p + 2) 

 1   1   1 
= − 2 L−1  2
+ 3 L−1   + 5L
−1
 2
 ( p − 2)   p + 2  ( p + 2) 
= − 2 e2 t L−1 {1/ p2 } + 3 e −2 t + 5 e −2 t L−1 {1/ p2 }
= − 2 e2 t . t + 3 e −2 t + 5 e −2 t . t
= (5 t + 3) e −2 t − 2 t e2 t .
 p2 
  1
Example 10: Show that L−1  4 4
= (cosh at sin at + sinh at cos at).
 p + 4 a  2a

Solution: We have
 p2   p2 
L−1  4 4
= L−1  2 2 2 2 2
 p + 4 a   ( p + 2 a ) − 4 a p 
 p2 
= L−1  2 2 2 2 
 ( p − 2 ap + 2 a ) ( p + 2 ap + 2 a )
 p ( p2 + 2 ap + 2 a2 ) − p ( p2 − 2 ap + 2 a2 ) 
−1  1
=L  ⋅ 
 4 a ( p2 + 2 ap + 2 a2 ) ( p2 − 2 ap + 2 a2 ) 
M-58

 p p 
= L−1  2 2
− 2 2 
 4 a ( p − 2 ap + 2 a ) 4 a ( p + 2 ap + 2 a )
1 −1  ( p − a) + a  1 −1  ( p + a) − a 
= L  2 2
− L  
4a  ( p − a) + a  4 a  ( p + a)2 + a2 
1 at −1  p + a  1 − at −1  p − a 
= e L  2 2 − e L  2 
4a  p + a  4 a  p + a2 

1 at  −1  p  
−1  1  
= e L  2 2 +aL  2 2 
4a   p + a   p + a  
1 − at   p   1  
− e  L−1  2 2
− a L−1  2 2 
4a   p + a   p + a  
1 at 1 − at
= e (cos at + sin at) − e (cos at − sin at)
4a 4a
1   e at + e − at   e at − e − at  
=    sin at +   cos at
2a  2   2 
     
1
= (cosh at sin at + sinh at cos at).
2a
 ( p + 1) e − πp 
Example 11: Find L−1  2 .
 p + p + 1 
 1 1
 p + 1  ( p+ )+ 
−1 −1  2 2
Solution: We have L  2 =L  1 3
 p + p + 1  ( p + )2 + 
 2 4
 1
 p+ 2 
= e − t / /2 L−1 
3
 p2 + 
 4

 p  1 − t /2 −1  1 
= e − t /2 L−1  2 2
+ e L  2 2 
 p + (√ 3 / 2)  2  p + (√ 3 / 2) 
1 − t /2
= e − t /2 cos (√ 3 t / 2) + e . (2 / √ 3) sin (√ 3 t / 2)
2
e − t /2
= [√ 3 cos (√ 3 t / 2) + sin (√ 3 t / 2)].
√3
 − (t − π ) / 2  √3 √3 
 ( p + 1 ) e − πp   e
−1 √ 3 cos (t − π) + sin (t − π) , t > π
∴ L  2 = √3  2 2 
 p + p + 1   0 ,t< π

e − (t − π ) / 2  √3 √3 
= √ 3 cos (t − π) + sin (t − π) H (t − π).
√3  2 2 
M-59

Comprehensive Exercise 2

Evaluate the following :


 1   p−1 
1. (i) L−1  2 ⋅ (ii) L−1  2 ⋅
 p − 6 p + 10   ( p + 3) ( p + 2 p + 2)
 1   p 
2. (i) L−1   (ii) L−1  5 /2 

 ( p + a)n   ( p + 1) 
 p   3 p + 2 
3. (i) L−1  5
(Lucknow 2011) (ii) L−1  2 
 ( p + 1)   4 p + 12 p + 9 
 1 
(iii) L−1  .
 √ (2 p + 3)
 p2 − 2 p + 3   1 
4. (i) L−1  2  (ii) L−1  2

 ( p − 1) ( p + 1)  ( p + 2) ( p − 1) 

(Gorakhpur 2005)

 p  1  32 p 
5. If L−1  2 2
= t sin t, find L−1  2 2

 ( p + 1)  2  (16 p + 1) 
 e −5 p   e − 4 p 
6. Find (i) L−1   (ii) L−1  ⋅
 ( p − 2)4   ( p − 3)4 

 e4 − 3 p 
7. Find L−1  5 /2 

 ( p + 4) 
8. Find the inverse L.T. of e − 3 p / p3 . (Lucknow 2009)

9. Prove the following :


 e − pπ 
(i) L−1  2  = − sin t. H (t − π)
 p + 1 (Kanpur 2008)
−2 pπ /3
 p e 
(ii) L−1  2  = cos 3 (t − 2 π / 3) . H (t − 2 π / 3).
 p + 9 

10. Prove the following :


 pe − ap 
(i) L−1  2 2
= cos h ω (t − a). H (t − a), a > 0
 p − ω 
 3 (1 + e − pπ )
(ii) L−1   = − sin 3 t. H (t − π) + sin 3 t.
 p2 + 9 
M-60

11. Evaluate
 p+2   p+8 
(i) L−1  2  (ii) L−1  2 ⋅
 p − 2 p + 5   p + 8 p + 5 
(Kanpur 2008; Lucknow 10; Avadh 13)

 4p + 5  1 t 1 −2 t
12. Prove that L−1  2
t
 = 3t e + e − e .
 ( p − 1) ( p + 2) 3 3 (Purvanchal 2007)

 4p + 5   5 p2 − 15 p − 11
13. Evaluate (i) L−1  2  (ii) L−1  3
.
 ( p − 4) ( p + 3)  ( p + 1) ( p − 2) 
(Gorakhpur 2009)
 2p + 1  1
14. Prove that L−1  2 2
= t (e t − e −2 t ).
 ( p + 2) ( p − 1)  3 (Gorakhpur 2007)

 p  1
15. Prove that L−1  2 2  = sin t sinh t.
 ( p − 2 p + 2) ( p + 2 p + 2) 2

A nswers 2
1. (i) e 3 t sin t
4 − 3t 1 − t
(ii) − e + e (4 cos t − 3 sin t)
5 5
t n−1 2 −t  t
2. (i) e − at ⋅ (ii) e   (3 − 2 t)
(n − 1) ! 3  π
1 −t 1 −3 t /2
3. (i) e (4 t 3 − t 4 ) (ii) e (6 − 5 t)
24 8
1
(iii) 3 t/2
e √ 2π t
1 3 1
4. (i) (t − ) e t + e − t (ii) [(3 t − 1) e t + e − 2 t ]
2 2 9
1 1
5. t sin ( t)
4 4
1 1
6. (i) (t − 5)3 e 2 (t − 5) . H (t − 5) (ii) (t − 4)3 e 3 (t − 4) . H (t − 4)
6 6
4 1
7. (1/ √ π) . (t − 3)3 /2 e − 4 (t − 4) . H (t − 3) 8. (t − 3)3 . H (t − 3)
3 2
3
11. (i) e t [cos 2 t + sin 2 t]
2
(ii) e − 4 t [cosh (√ 11t) + (4 / √ 11) sinh (√ 11t)]
1 − 3t 1 4t 1 7
13. (i) − e + e + 3 te 4 t (ii) (− e − t + e2 t ) + 4 te2 t − t2 e2 t
7 7 3 2
M-61

11 Inverse Laplace Transform of Derivatives


Theorem: If L−1 { f ( p)} = F (t), then
dn 
L−1 { f n
( p)} = L−1  n f ( p)
 dp 
= (− 1)n t n
F (t)
= (− 1)n t n
L−1 { f ( p)}, n = 1, 2, 3,...

Proof: Since, we have


dn
L { t n F (t)} = (− 1)n f ( p)
dpn
= (− 1)n f n ( p),
 dn 
∴ L−1 { f n ( p)} = L−1  n f ( p) = (−1)n t n
F (t)
 dp 
= (−1)n t n L−1 { f ( p)}.

12 Inverse Laplace Transform of Integrals


 ∞  F (t)
Theorem: If L−1 { f ( p)} = F (t), then L−1 ∫ f ( x) dx = ⋅
 p  t
Proof: Since, we have
1  ∞  F (t)
L  F (t) =
 t  ∫p f ( x) dx, provided lim 
t→0  t 
exists.

 ∞  F (t)
∴ L−1  ∫p f ( x) dx = ⋅
  t

13 Multiplication by Powers of p
Theorem: If L−1 { f ( p)} = F (t) and F (0 ) = 0 , then L−1 { p f ( p)} = F ′ (t).

Proof: We have
L {F ′ (t)} = p L {F (t)} − F (0 ) = p f ( p). [ ∵ F (0 ) = 0 ]
∴ L−1 { p f ( p)} = F ′ (t).

Note 1: If F (0 ) ≠ 0 , then
L−1 { p f ( p) − F (0 )} = F ′ (t)

or L−1 { p f ( p)} = F ′ (t) + F (0 ) δ(t)

where δ (t) is the Dirac delta function or unit impulse function.


M-62

Note 2: Generalizations to L−1 { p n f ( p)} are possible, for n = 2, 3, ... i. e.,


in general L−1 { p n f ( p)} = F n (t) if F (0 ) = 0 .

14 Division by Powers of p
Theorem I: If F(t) is sectionally continuous and of exponential order a and such that
F (t)  f ( p) t
lim exists then for p > a, L−1   = ∫ 0 F ( x) dx.
t→0 t  p 
t
Proof: Let G (t) = ∫0 F ( x) dx.

Then G ′ (t) = F (t) and G (0 ) = 0 .


∴ L { G ′ (t)} = p L { G (t)} − G (0 ) = p L { G (t)}
or L { F (t)} = f ( p) = p L { G (t)}.
f ( p)  f ( p) t
∴ L { G (t)} = . Hence L−1   = G (t) = ∫ 0 F ( x) dx.
p  p 
 f ( p) t y
Theorem II: L−1  2  = ∫0 ∫0 F ( x) dx dy.
 p 
t y
Proof: Let G (t) = ∫0 ∫0 F ( x) dx dy .

t
Then G ′ (t) = ∫ 0 F ( x) dx, G ′ ′ (t) = F (t). Also G (0 ) = G ′ (0 ) = 0 .

Now L { G ′ ′ (t)} = p2 L { G (t)} − p G (0 ) − G ′ (0 ) = p2 L { G (t)}


f ( p)
or L { F (t)} = f ( p) = p2 L { G (t)}. ∴ L { G (t)} =
p2
 f ( p) t y
or L−1  2  = G (t) = ∫0 ∫0 F ( x) dx dy .
 p 

 f ( p) t t
This may also be written as L−1  2  = ∫ ∫ F (t) dt2 .
 p  0 0

 f ( p) t t t
In general L−1  n  = ∫ ∫ ... ∫ F (t) dt n.
 p  0 0 0

 p 
Example 12: Find L−1  2 2 2

 ( p + a )  (Purvanchal 2010)

 p   1 d  1   1 −1  d  1  
Solution: L−1  2 2 2
= L−1 − ⋅ 
 2

2 = − L   2 
2 
 ( p + a )   2 dp  p + a   2  dp  p + a  
M-63

1  1  t
=− t . (− 1)1 L−1  2 2
= sin at.
2  p + a  2 a
 p
2 
Example 13: Find L−1  2 2

 ( p + 4) 
p
Solution: Let f ( p) = .
( p + 4)2
2

 p  t
∴ L−1 { f ( p)} = L−1  2 2
= sin 2 t = F (t)
 ( p + 4)  4
[See Ex. 12, here a = 2]
and F (0 ) = 0 .
∴ L−1 { p f ( p)} = F ′ (t)
 p2  d 1 1
or L−1  2 2
= ( t sin 2 t) = (sin 2 t + 2 t cos 2t).
 ( p + 4)  dt 4 4

 p + 2 
Example 14: Find L−1  2 ⋅
 p ( p + 3)
 p + 2   ( p + 3) − 1 
−1 
Solution: We have L−1  2 =L  2 
 p ( p + 3)  p ( p + 3) 

 1 1  
−1  1 
 
−1  1 
= L−1  2 − 2  = L  2 − L  2  …(1)
 p p ( p + 3)  p   p ( p + 3)
Now L−1 {1/ p2 } = t /1 ! = t.
 1 
Also L−1  =e
−3 t
= F (t), say.
 p + 3
 1  t
∴ we have L−1   = ∫ 0 F ( x) dx.
 p ( p + 3) (by theorem I of article 14)
t −3 x 1 −3 t
=∫ e dx = (1 − e ) = F1 (t) say,
0 3

 1 
 t 1 t
L−1  2  = ∫ 0 F1 ( x) dx = (1 − e −3 x ) dx
3 ∫0

 p ( p + 3)
1 1
= t + (e −3 t − 1).
3 9
∴ From (1), we have
 p + 2  1 1 −3 t 2 1 1
L−1  2  = t − t − (e − 1) = t − e −3 t + ⋅
 p ( p + 3) 3 9 3 9 9

  1  
Example 15: Find (i) L−1  log 1 + 2   (Agra 2001)
  p  
M-64

 1  1  
(ii) L−1  log 1 +

 ⋅
 p  p2  

Solution: (i) Let


 1  p2 
f ( p) = log 1 + 2  = − log  2 

 p   p + 1
= − 2 log p + log ( p2 + 1).
2 2p
∴ f ′ ( p) = − + 2 ⋅
p p +1

∴ L−1 { f ′ ( p)} = − 2 + 2 cos t


or − t L−1 { f ( p)} = − 2 (1 − cos t)
or L−1 { log (1 + 1/ p2 )} = 2 (1 − cos t) / t.
(ii) From part (i), we have
F (t) = L−1 { log (1 + 1/ p2 )} = 2 (1 − cos t) / t.
 1  1   1 
∴ L−1  log 1 +

 = L−1  f ( p)
2 
 p  p   p 
t
= ∫0 F ( x) dx

t 2
=∫ (1 − cos x) dx .
0 x

Comprehensive Exercise 3

Evaluate the following :


 1   1 
1. (i) L−1  3
(ii) L−1  3

 ( p − a)   ( p + a) 
(Rohilkhand 2010)

 p   p+1 
2. (i) L−1  2 2 2
(ii) L−1  2 2

 ( p − a )   ( p + 2 p + 2) 
  1    1 
3. L−1  log 1 − 2   4. L−1  3 ⋅
  p    p ( p + 1)

 1   1 
5. L−1  3 2 ⋅ 6. L−1  3

 p ( p + 1)  p ( p + 1) 
 p+2  1 p+2 
7. (i) L−1  log  (ii) L−1  log ⋅
 p+1  p p+1
M-65

 p+3 
8. (i) L−1  log  (Kanpur 2007)
 p+2

1 p+3 
(ii) L−1  log ⋅
 p p+2

 p  1  1 
9. If L−1  2 2
= t sin t , find L−1  2 2 ⋅
 ( p + 1)  2  ( p + 1)  (Kanpur 2011)

 1
10. L−1  tan−1  ⋅
 p

A nswers 3
1 1 2 − at
1. (i) t2 e at (ii) t e .
2 2
1  t  1
2. (i)   sinh at (ii) te − t sin t.
2  a 2
1 2
3. 2 (1 − cosh t) / t. 4. 1 − t + t − e− t .
2
1 2 1
5. t + cos t − 1. 6. 1 − e − t (1 + t + t2 ).
2 2
t 1 −x
7. (i) (e − t − e − 2 t ) / t (ii) ∫ (e − e −2 x
) dx .
0 x
t 1 −2 x
8. (i) (e − 2 t − e − 3 t ) / t (ii) ∫ (e − e −3 x ) dx .
0 x
1 sin t
9. (sin t − t cos t). 10. ⋅
2 t

15 Convolution
Let F (t) and G (t) be two functions of class A, then the convolution of the two functions F (t) and
G (t) denoted by F * G is defined by the relation
t
F*G= ∫0 F ( x) G (t − x) dx.

This relation F * G is also called the resultant or falting of F and G.


Properties of Convolution:
(i) F * G is commutative i. e., F * G = G * F .
t
Proof: F*G= ∫0 F ( x) G (t − x) dx
M-66

t
= −∫ F (t − y) G ( y) dy, putting t − x = y so that dx = − dy
0
t
=∫ G ( y) F (t − y) dy = G * F .
0

(ii) F * G is associative i. e., ( F * G) * H = F * (G * H).


(iii) F * G is distributive w.r.t. addition, i.e., F * (G + H) = F * G + F * H .
t
Proof: F * (G + H) = ∫0 F ( x) . [G (t − x) + H (t − x)] dx

t t
=∫ F ( x) G (t − x) dx + ∫0 F ( x) H (t − x) dx
0

= F*G+ F* H.

16 Convolution Theorem (Convolution Property)


Let F (t) and G (t) be two functions of class A and let L−1 { f ( p) } = F (t) and
L−1 { g ( p)} = G (t), then
t
L−1 { f ( p) g ( p)} = ∫ F ( x) G (t − x) dx = F * G.
0

(Rohilkhand 2003, 07; Avadh 06, 09, 14; Lucknow 06, 10, 11; Kanpur 09)
Proof: Here we shall prove that the Laplace transform of the convolution of two
functions is equal to the product of their Laplace transforms
 t 
i. e., L ∫ F ( x) G (t − x) dx = f ( p) g ( p)
 0 
from which the required result follows immediately.
 t  ∞  t 
We have, L  ∫ F ( x) G (t − x) dx = ∫ t =0 e − pt ∫ F ( x) G (t − x) dx dt
 0   0 
∞  t 
= ∫ t =0 ∫ x =0 e − pt F ( x) G (t − x) dx dt ,
 …(1)

the integration being performed first with respect to x


P
x and then with respect to t.
In the double integral (1), the region of integration is t
t=0 x= t
the area in the tx- plane lying below the infinite line OP
whose equation is x = t and above the line Ot i.e., t-axis
whose equation is x = 0. Here t is taken along the line x
O t
x=0
Ot and x is taken along the perpendicular line Ox. The
integration is first performed with respect to x
regarding t as constant and so the strip is taken parallel to Ox.
M-67

If we change the order of integration, then the strip is taken parallel to Ot so that the
limits of t are from x to ∞ and those of x are from 0 to ∞.
∴ changing the order of integration in the double integral (1), we have
 t  ∞  ∞ − pt 
L  ∫ F ( x) G (t − x) dx = ∫ ∫ t = x e F ( x) G (t − x) dt dx
 0  x = 0 
∞  ∞ 
= ∫ x =0 e − px F ( x) ∫ e px e − pt G (t − x) dt dx
 t=x 
∞  ∞ − p ( t − x) 
= ∫ x =0 e − px F ( x) ∫ e G (t − x) dt dx
 t=x 
∞  ∞ 
= ∫ x =0 e − px F ( x) ∫ e − pz G (z ) dz  dx,
 z = 0 
putting t − x = z , so that dt = dz ; when t = x,
we have z = 0 and when t → ∞, z → ∞
 ∞  ∞ 
= ∫ e − px F ( x) dx ∫ e − pz G (z ) dz 
 x =0   z =0 
 ∞  ∞ 
= ∫ e − pt F (t) dt ∫ e − pt G (t) dt
 t = 0  t = 0 
 b b b 
∵ ∫a f ( x) dx = ∫a f (t) dt = ∫a f (z ) dz 

= L{ F (t)}. L { G (t)} = f ( p) g ( p).
 t 
Thus L  ∫ F ( x) G (t − x) dx = f ( p) g ( p) .
 0 
t
Hence L−1{ f ( p) g ( p)} = ∫0 F ( x) G (t − x) dx = F (t) * G (t) .

17 Heaviside’s Expansion Theorem or Formula


Let F (p) and G (p) be two polynomials in p where F (p) has degree less than that of G (p). If G (p)
has n distinct zeros α r , r = 1, 2, ..., n, i.e. G ( p) = ( p − α1) ( p − α 2 )...( p − α n), then
 F ( p)  n F (α r ) α r t
L−1  = Σ e .
 G ( p)  r =1 G ′ (α r )

Proof: Since F (p) is a polynomial of degree less than that of G ( p) and G ( p) has n
distinct zeros α r , r = 1, 2, ..., n
F ( p) F ( p)
∴ =
G ( p) ( p − α1) ( p − α 2 )...( p − α n)
A1 A2 Ar An
= + + ... + + ... + .
p − α1 p − α2 p − αr p − αn
Multiplying both sides by ( p − α r ) and taking limits as p → α r , we have
M-68

F ( p) ⋅ ( p − α r )
Ar = lim
p→ α r G ( p)
(p − αr)  0 
= F (α r ) ⋅ lim Form 0 
p→ α r G ( p)
1
= F (α r ) ⋅ lim [by L’ Hospital’s rule]
p → αr G ′ ( p)
1
= F (α r ) ⋅ ⋅
G ′ (α r )
F ( p) F (α1) 1 F (α2 ) 1
∴ = ⋅ + . + ...
G ( p) G ′ (α1) ( p − α1) G ′ (α2 ) ( p − α2 )
F (α r ) 1 F (α n) 1
+ ⋅ + ... + ⋅ + ... .
G ′ (α r ) ( p − α r ) G ′ (α n) p − α n
 F ( p)  F (α1) −1  1  F (α2 ) −1  1 
Hence L−1  = ⋅L  + L   + ...
 G ( p )  G ′ (α1 )  p − α1  G ′ (α 2 )  ( p − α )
2 

F (α r ) −1  1  F (α n) −1  1 
+ L   + ... + .L  
G ′ (α r )  p − αr  G ′ (α n)  p − αn 
F (α1) α 1 t F (α r ) α r t F (α n) α n t
= ⋅e + ... + ⋅e + ... + ⋅e
G ′ (α1) G ′ (α r ) G ′ (α n)
n F (α r ) α r t
= Σ e .
r =1 G ′ (α r )

18 The Beta Function


If m > 0, n > 0, the Beta function is defined as
1
B (m, n) = ∫0 x m −1 (1 − x)n−1 dx.

 p2 
Example 16: Use the convolution theorem to find L−1  ⋅
 ( p2 + a2 )2 
 p 
Solution: We have L−1  2 2
= cos at.
 p + a 
∴ By the convolution theorem, we have
 p2   p p 
L−1  2 2 2
= L−1  2 2
⋅ 2 2
 ( p + a )   p + a p + a 
M-69
t
= ∫ 0 cos ax cos a (t − x) dx
t
= ∫0 cos ax (cos at cos ax + sin at sin ax) dx
t t
∫ 0 cos ax dx + sin at ∫ cos ax sin ax dx
2
= cos at
0
1 t 1 t
= cos at ∫ (1 + cos 2 ax) dx + sin at ∫ sin 2 ax dx
2 0 2 0
t t
1  1  1  1 
= cos at  x + sin 2 ax  + sin at. − cos 2 ax
2  2a 0 2  2a 0
1  1  1
= cos at t + sin 2 at  + sin at (1 − cos 2 at)
2  2a  4a
1 1 1
= t cos at + sin at + (sin 2 at cos at − sin at cos 2 at)
2 4a 4a
1 1 1
= t cos at + [sin at + sin (2 at − at)] = [at cos at + sin at].
2 4a 2a
 1 
Example 17: Find L−1   , by the convolution integral and deduce the value of
 √ p . ( p − a)
 1 
L−1  ⋅
 p √ ( p + a)
 1 
−1  1 
 t1 /2 − 1 1
Solution: We have L−1   = L  1 /2  = 1
= = F1 (t), say
 √ p  p  Γ (2
) √π √t
 1 
and L−1  at
 = e = F2 (t), say.
 p− a
∴ By the convolution theorem , we have
 1  t 1
L−1   = F1 (t) * F2 (t) = ∫0 ⋅ e a(t − x)dx
 √ p ⋅ ( p − a) √π √ x

e at √(at) √ a − u2 2 u 2 u du
=
√π ∫0 u
e ⋅
a
du, putting ax = u2 , so that dx =
a
e at 2 √(at) − u2 e at
√ a √ π ∫0
= ⋅ e du = erf (√ (at)).
√a
 1  −1  1 
Deduction. L−1  =L  
 p √ ( p + a)  ( p + a − a) √ ( p + a)
 1  − at e
at
1
= e − at L−1  =e ⋅ erf (√ (at)) = erf (√ (at)).
 ( p − a) √ p √a √a

Example 18: Apply convolution theorem to prove that


1 Γ (m) ⋅ Γ(n)
B (m, n) = ∫ x m −1 (1 − x)n−1 dx = , m > 0, n > 0 .
0 Γ(m + n)
(Gorakhpur 2008; Agra 02)
M-70

Hence deduce that


π /2 1 Γ (m) ⋅ Γ (n)
∫0 sin2 m −1 θ ⋅ cos2 n−1 θ dθ = B (m, n) = ⋅
2 2 Γ (m + n)
Solution: Consider the function
t
F (t) = ∫0 x m −1 ⋅ (t − x)n−1 dx.

t
We have, F (t) = ∫0 F1 ( x) ⋅ F2 (t − x) dx, where F1 (t) = t m −1 and F2 (t) = t n−1

= F1 * F2 .
∴ L {F (t)} = L {F1 * F2 }
= L {F1 (t)}. L {F2 (t)}, by convolution theorem
Γ (m) Γ (n) Γ (m) Γ (n)
= L { t m − 1} . L {t n − 1} = ⋅ n = ⋅
pm p pm+n

t m −1  Γ (m) Γ (n)
∴ F (t) = ∫0 x ⋅ (t − x)n − 1 dx = L−1  
 pm + n 
 1  Γ (m) Γ (n) m + n − 1
= Γ (m) ⋅ Γ (n) ⋅ L−1  m + n  = t .
 p  Γ (m + n)

Taking t = 1, we have
1
B (m, n) = ∫0 x m − 1 (1 − x)n − 1 dx

Γ (m) Γ (n)
= ⋅ …(1)
Γ (m + n)
Deduction: Taking x = sin2 θ, so that dx = 2 sin θ cos θ dθ. From (1), we have
π /2 Γ (m) Γ (n)
2 ∫0 sin2 m − 1 θ cos2 n − 1 θ dθ = ⋅
Γ (m + n)
π /2 Γ (m) Γ (n) 1
Hence ∫0 sin2 m − 1 θ ⋅ cos2 n − 1 θ dθ = = B (m, n)
2Γ (m + n) 2
 3p + 1 
Example 19: Using Heaviside’s expansion formula find L−1  2 
 ( p − 1) ( p + 1)

(Gorakhpur 2008)
Solution: Here F ( p) = 3 p + 1
and G ( p) = ( p − 1) ( p2 + 1) = ( p − 1) ( p − i) ( p + i).

∴ G ( p) has 3 distinct zeros α1 = 1, α2 = i and α3 = − i.


2
Also G ′ ( p) = 3 p − 2 p + 1.

∴ By the Heaviside’s expansion formula, we have


 3p + 1  F (1) t F (i) i t F (− i) − it
L−1  2 = e + e + e
 ( p − 1) ( p + 1) G ′ (1) G ′ (i) G ′ (− i)
M-71

4et (3 i + 1) (− 3 i + 1) − it
= + e it + e
2 − (2 + 2 i) (− 2 + 2 i)
(3 i + 1) (1 − i) it (3 i − 1) (1 + i) − it
= 2e t − e + e
2 (1 + i)(1 − i) 2 (1 − i) (1 + i)
1 1
= 2et − (i + 2) e it + (i − 2) e − it
2 2
1 −it
t
= 2 e − i (e − eit
) − (e i t + e − i t )
2
1
= 2 e t − i . 2 i sin t − 2 cos t = 2 e t + sin t − 2 cos t.
2
Example 20: Using Heaviside’s expansion formula find
 2 p2 + 5 p − 4 
L−1  3 2 ⋅
 p + p − 2 p  (Avadh 2010)

Solution: Here F ( p) = 2 p2 + 5 p − 4
and G ( p) = p3 + p2 − 2 p = p ( p − 1) ( p + 2)
G ′ ( p) = 3 p2 + 2 p − 2.
G ( p) has 3 distinct zeros α1 = 0 , α2 = 1 and α3 = − 2.
∴ By the Heaviside’s expansion formula, we have
 2 p2 + 5 p − 4  F (0 ) 0 t F (1) t F (− 2) −2 t
L−1  3 2 = e + e + e
G ′ (0 ) G ′ (1) G ′ (− 2)
 p + p − 2 p 
= 2 + e t − e −2 t .

Comprehensive Exercise 4

Use the convolution theorem to find:


 1   1 
1. (i) L−1   (ii) L−1  
 ( p − 1) ( p + 2)  ( p + 1) ( p − 2)
 1 
(iii) L−1  ⋅
 ( p − 1) ( p + 3) (Kanpur 2012)
 p   1 
2. (i) L−1  2 2 2
(ii) L−1  2 2

 ( p + a )   p ( p + 4) 
(Lucknow 2009, 11)
  1   1 p 
Hint. (ii) L−1  2 2
= L−1  2 ⋅ 2 2

  p ( p + 4)   p ( p + 4) 

1 p 
Take f1 ( p) = 2
, f2 ( p) = 2 2

p ( p + 4) 
M-72

 1   1 
3. (i) L−1  2  (ii) L−1  2 ⋅
 ( p − 2) ( p + 1)  ( p + 4) ( p + 2)

 1 
4. (i) L−1  2 2 (Gorakhpur 2006)
 p ( p + 1) 
 1 
(ii) L−1  2 ⋅ (Kanpur 2011)
 ( p + 2) ( p − 2)
 p2 
5. Find L−1  2 2

 ( p + 4)  (Kanpur 2010)

[Hint. Proceed as in Ex. 16. Here a = 2].


 1  1
6. Show that L−1   = erf (2 √ t).
 p √ ( p + 4) 2
Using Heaviside’s expansion formula, find
 2 p2 − 6 p + 5   p2 − 6 
7. L−1  3 2 ⋅ 8. L−1  3 2 ⋅
 p − 6 p + 11p − 6   p + 4 p + 3 p
 19 p + 37   p+5 
9. (i) L−1   (ii) L−1  2 ⋅
 ( p + 1) ( p − 2) ( p + 3)  ( p + 1) ( p + 1)

A nswers 4
1 t 1 2t
1. (i) (e − e − 2 t ) (ii) (e − e − t)
3 3
1 t
(iii) (e − e − 3 t )
4
1
2. (i) (1/ 2 a) t sin at (ii) (1 − t sin 2 t − cos 2 t)
16
1 2t 1 − 2t
3. (i) (e − 2 sin t − cos t) (ii) (e + sin 2 t − cos 2 t)
5 8
1 2t
4. (i) (t + 2) e − t + t − 2 (ii) [e − (4 t + 1) e 2 t ]
16
1 1 t 5
5. (2 t cos 2 t + sin 2 t). 7. e − e2t + e3t
4 2 2
5 − t 1 − 3t
8. −2+ e + e
2 2
9. (i) − 3 e − t + 5 e2 t − 2 e −3 t (ii) 2 e − t − 2 cos t + 3 sin t
M-73

 e −√ p   − x √ p   x 
−1  e
Example 21: Find L−1   and hence deduce that L   = erfc   ⋅
 p   p   2 √ t

Solution: Let f ( p) = e − √ p.

∴ F (t) = L−1 {e − √ p}
 p p3 /2 p2 p5 /2 
= L−1 1 − √ ( p) + − + − + …
 2! 3! 4! 5! 
1 −1 1 −1 3 / 2
= L−1 {1} − L−1 { p1 /2} + L { p} − L {p }
2! 3!
1 −1 2 1 −1 5 / 2
+ L {p }− L { p } + ... …(1)
4! 5!
 1 
Now L−1{ pn + (1 /2)} = L−1  − n−(1 /2) 
 p 
− n − (3 /2)
t
= , for n = 0 , 1, 2, ...
1
Γ (− n − )
2
(− 1)n + 1  1  3   5   2 n + 1 − n − (3 /2)
=       ...   t .
√ π  2  2  2  2 
 1 n+1  2   2  2   2  
∵ Γ (− n − ) = (− 1)       ... √ π
 2  1  3  5   2 n + 1 
Also L−1 { p n} = 0 , if n is a +ve integer.
∴ from (1), we have
(− 1) t −3 /2 1 1 (− 1)2  1  3  −5 /2
F (t) = − ⋅ − ⋅ ⋅    t
√π 2 3 ! √ π  2  2
1 (− 1)3  1  3  5  −7 / 2
−       t + ...
5! √ π  2  2  2
  1
2
 1
3 
     
1 1 − 1 +  4t   
= − 4t + ...
2 √ (π) t3 /2  4t 2! 3! 
 
 
1
= e −1 / 4 t ⋅
2 √ (π) t3 /2
 f ( p) t
Since L−1  = ∫0 F ( x) dx , where F (t) = L−1 { f ( p)}, we have
 p 
M-74

 e −√ p  t 1
L−1   = ∫0 3 /2
e −1 /(4 x)dx
 p  2 √ (π) x

2 1 /(2 √ t) 2 1 dy
=− ∫∞ e − y dy, putting x = so that dx = −
√π 4 y2 2 y3

2 ∞ − y2  1 
=
π ∫1 / (2 √t) e dy = erfc  ⋅
 2 √ t
 2 t − x2
∵ Complementary Error Function erfc (t) = 1 − erf (t) = 1 − √ π ∫0 e dx

2  ∞ − x2 0 2  2 ∞ − x2 
e − x dx =
√ π  ∫0 ∫t ∫t
=  e dx + e dx
 √π 
 e −√ p   1 
Deduction: We have L−1   = erfc   ⋅
 p   2 √ t
 −√( x2 p)   
−1  e  1 1
∴ L  2  = 2 erfc   , by change of scale property
2 

x p  x
  2 √ (t / x )
 e − x √ p   x 
or L−1   = erfc  ⋅

p
 2 √ t
 1  t2 t5 t8 t11
Example 22: (i) Prove that L−1  3 = − + − + ...
 p + 1 2 ! 5 ! 8 ! 11!
(ii) Applying Heaviside’s Expansion formula , prove that

 1  1 − t 1 1
L−1  3 t /2
 = [e − e { cos ( √ 3 t) − √ 3 sin ( √ 3t)}].
 p + 1 3 2 2
1 1
Solution: (i) We have, 3 = (1 + 1 / p3 )−1
p +1 p3

1  1 1 1 1 
= 3 
1 − 3 + 6 − 9 + 12 − ...
p  p p p p 
1 1 1 1
= 3
− 6
+ 9
− 12
+ ...
p p p p

 1  t2 t5 t8 t11
∴ L−1  3 = − + − +…
 p + 1 2 ! 5 ! 8 ! 11 !

(ii) Here F ( p) = 1 and G ( p) = p 3 + 1 = ( p + 1) ( p2 − p + 1), G ′ ( p) = 3 p2 .


1 1
G ( p) has 3 distinct zeros − 1, (1 + √ 3 i) and (1 − √ 3 i).
2 2
∴ By the Heaviside’s expansion formula , we have
M-75
1
 1  F (− 1) − t F { 12 (1 + √ 3 i )} { (1 + √3 i)} t
L−1  3  = e + e 2
 p + 1 G ′ (− 1) G ′ { 12 (1 + √ 3 i )}
1
F { 12 (1 − √ 3 i)} { (1 − √3 i)} t
+ e2
G ′ { 12 (1 − √ 3 i)}
1 1
1 −t 1 { (1 + √3 i )}t 1 { (1 − √3 i)}t
= e + e2 + e2
3 3
4
(1 + √ 3 i )2 3
4
(1 − √ 3 i)2
1
1 −t 2 (√ 3 i + 1) { (1 + √3 i )}t
= e + e 2
3 3 (√ 3 i − 1) (√ 3 i + 1)
1
2 (√ 3 i − 1) { (1 − √3 i )}t
+ e2
3 (√ 3 i + 1) (√ 3 i − 1)
1 − t 1 t /2
=
e − e [(√ 3 i + 1) e √3 it /2 − (√ 3 i − 1) e −√3 it /2 ]
3 6
1 1
= e − t − e t /2 [√ 3 i (e √3 it /2 − e − √3 it /2 ) + (e √3 it /2 + e −√3 it /2 )]
3 6
1 1 1
= [e − t − e t /2 {− √ 3 sin ( √ 3 t) + cos ( √ 3 t)}].
3 2 2
∞ 1
Example 23: Show that ∫ cos x2 dx = √ (π / 2) (Avadh 2008)
0 2

Solution: Let F (t) = ∫0 cos tx2 dx.

∞ − pt ∞ − pt  ∞ 
∫0 ∫0  ∫ 0 cos tx dx dt
2
∴ L { F (t)} = e F (t) dt = e

∞ ∞ − pt  ∞ ∞ p
=∫  ∫ 0 e cos tx dt dx = ∫ 0 L {cos tx } dx =
2 2
∫0 dx
0 p2 + x4
1 π /2 dθ
=
2√p ∫ 0 √ (tan θ)
, putting x = √ ( p tan θ) so that

p sec2θ dθ
dx =
2 √ ( p tan θ)
1 π /2
= ∫ sin−1 /2 θ cos 1 /2 θ dθ
2√ p 0
1 3 1 1
1 Γ (4 ) Γ(4 ) 1 Γ (4 ) Γ (1 − 4 ) 1 π π
= = = ⋅ = ⋅
2 √ p 2 Γ (1) 2√ p 2 4 √ p sin 14 π 2 √ (2 p)
 π 
∵ Γ ( p) Γ (1 − p) = , 0 < p < 1
 sin pπ 
π  1  π t(1 /2) −1 1  π 
∴ F (t) = L−1  1 /2  = =  
2√2  p  2 2 Γ (2 )
1 2  2t 
∞ 1  π
or ∫0 cos tx2 dx =   .
2  2t 
Now taking t = 1 , we have
M-76
∞ 1
∫0 cos x2 dx =
2
√ (π / 2)

∞ 2 1
Example 24: Prove that ∫0 e − x dx = √ π.
2 (Meerut 2013B)
∞ 2
Solution: Let F (t) = ∫0 e − tx dx.

∴ Proceeding as in Ex. 23,



∞ − tx2 ∞ dx  1 x  π
L { F (t)} = ∫ L {e } dx = ∫0 2
=  tan−1  =

0 p+ x √ p √ p 0 2 √ p

π −1  1 π 1 1  π
or F (t) = L   = ⋅ =  
2  √ p 2 √ (πt) 2  t 
∞ 2 1  π
or ∫0 e − t x dx =   .
2  t
∞ 2 1
Taking t = 1 , we have ∫0 e − x dx = √ π.
2
 8 
Example 25: Prove that L−1  2 3
= (3 − t2 ) sint − 3 t cos t.
 ( p + 1) 
 1 
Solution: We have L−1  2  = sin t .
 p + 1
∴ by the convolution theorem, we have
 1 1  t
L−1  2 ⋅ 2  = ∫ sin x sin (t − x) dx
( p + 1) (( p + 1) 0
t
=∫ sin x (sin t cos x − cos t sin x) dx
0
t t
= sin t ∫ sin x cos x dx − cos t ∫ sin2 x dx
0 0
1 t 1 t
= sin t ∫ sin 2 x dx − cos t ⋅ ∫ (1 − cos 2 x) dx
2 0 2 0
1 1 1 1
= sin t . (1 − cos 2 t) − cos t . (t − sin 2 t)
2 2 2 2
1 1 1
= sin t.sin2 t − t cos t + cos t sin t cos t
2 2 2
 1  1 t
or L−1  2 2
= sin t − cos t.
( p + 1)  2 2
 1   1 1 
∴ L−1  2 3
= 8 L−1  2 2
⋅ 2 
( p + 1)  ( p + 1) ( p + 1)
t 1 x 
= 8 ∫  sin x − cos x sin (t − x) dx, by the convolution theorem
0 2 2 
t
= 4 ∫ (sin x − x cos x) (sin t cos x − cos t sin x) dx
0
t t
= 4 sin t ∫ (sin x cos x − x cos2 x) dx − 4 cos t ∫0 (sin2 x − x sin x cos x) dx
0
M-77

t t
= 2 sin t ∫ {sin 2 x − x (1 + cos 2 x) } dx − 2 cos t ∫ {(1 − cos 2 x) − x sin 2 x} dx
0 0
 t2 1 − cos 2 t t 1 − cos 2 t 
= 2 sin t − + − sin 2 t + 
 2 2 2 4 
 1 t cos 2 t sin 2 t 
−2 cos t t − sin 2 t + −
 2 2 4 
3 3
= − t 2 sin t + sin t − sin t cos 2 t − t sin t sin 2 t
2 2
3
− 2 t cos t + sin 2 t cos t − t cos t cos 2 t
2
3 3
= − t 2 sin t + sin t + (− sin t cos 2 t + sin 2 t cos t)
2 2
− t (cos 2 t cos t + sin t sin 2 t) − 2 t cos t
2
= (3 − t ) sin t − 3 t cos t.

Comprehensive Exercise 5

 1 
1. Find L−1  5 .
 ( p − 1) ( p + 2) (Kanpur 2009, 11)

 1  t
2. Prove that L−1  2 2 
= ∫0 J0 (ax) dx .
 p √ ( p + a )

3. Apply Heaviside’s expansion formula, to prove that


 1  1 t 1 1
L−1  3  = [e − e
−1 / 2
{cos ( √ 3 t) + √ 3 sin ( √ 3 t)}]
 p − 1 3 2 2
∞ 1
4. Show that ∫ sin x2 dx = √ (π /2).
0 2
5. Apply the convolution theorem to show that
t t
∫ 0 sin u cos (t − u) du = 2 sin t . (Gorakhpur 2007, 11)
2
∞ sin x π
6. Use Laplace transform to prove that ∫0 2
x
dx =
2

 2  2
7. Prove that L−1  tan−1 2  = sin t sinh t. (Agra 2002)
 p  t

8. Show that 1 * 1 * 1 * … * 1 (n times ) = t n−1 / (n − 1)! , where n = 1, 2, 3,…


(Rohilkhand 2002)
M-78

 1 
9. Find L−1  2 2 3 /2 

( p + a ) 
t t t t (t − u)n−1
10. Prove that ∫ 0 ∫ 0 ...... ∫ 0 F (t) dt n = ∫0 (n − 1) !
F (u) du.

A nswers 5
1 t 4 4 3 4 2 8 8 1 −2 t
1. e t − t + t − t +  − e
72  3 3 9 27  243
t
9. J1 (at)
a

19 Table of Inverse Laplace Transform Theorems

Inverse Laplace Transform Theorems

No. Operation f (p) L−1 { f ( p)} = F(t )

1. Linearity property a1 f1 ( p ) + a2 f2 ( p ) a1 L−1 { f1 ( p )} +


a2 L−1 { f2 ( p )}

2. First translation or f ( p − a) e at L−1 { f ( p )}


shifting theorem

3. Second translation or e − ap f ( p )  F (t − a), t > a


shifting theorem G (t) = 
 0, t< a

4. Change of scale f (ap ) 1  t


property F 
a  a

5. Differentiation f n
( p) (− 1)n t n
F (t)
theorem

∞ 1
6. Integral theorem ∫p f ( x) dx
t
F (t)

7. Multiplication p f ( p) F ′ (t)
theorems
pn f ( p ) F n (t)
M-79

f ( p) t

p
∫0 F ( x) dx
8. Division theorems t t t
f ( p)
n
∫0 ∫0 …∫0 F (t) (dt)n
p

9. Convolution t
theorem
f ( p). g ( p ) F*G= ∫0 F ( x) ⋅ G (t − x) dx

F ( p) n F (α r ) α r t
10. Heaviside’s , Σ e
G ( p) r =1 G ′ (α r )
expansion
theorem degree F ( p ) < degree where α r , r = 1, 2, ... n, are
G ( p) roots of
G ( p ) = 0 and are all
distinct.

Objective Type Questions

Muliple Choice Questions


Indicate the correct answer for each question by writing the corresponding letter from (a),
(b), (c) and (d).
 1
1. The value of L−1  4  is
p 
t3 t4
(a) (b)
3! 4!
t5
(c) (d) None of these.
5!
 1
2. The value of L−1   is
 √ p
 π  t
(a)   (b)  
 t  π
1
(c) (d) √ (πt).
√ (πt)
 1 
3. The value of L−1  5 /2  is
 p 
4 3 /2 4 5 /2
(a) t (b) t
3 3
4  t 4  π
(c) t   (d) t   ⋅
3  π 3  t
M-80

4. If L−1 { f ( p)} = F (t) then L−1 { f ( p − a)} is


1  t
(a) e at F (t) (b) F 
a  a
t
(c) (−1)n t ∫0
n
F (t) (d) F (u) du
(Rohilkhand 2002)
−1 −1
5. If L { f ( p)} = F (t) then L { f (ap)} is
(a) aF (at) (b) F (at)
 t 1  t
(c) F   (d) F   . (Rohilkhand 2003, 10)
 a a  a
 f ( p)
6. If L−1 { f ( p)} = F (t) then L−1   is
 p 
(a) (−1)n t n F (t ) (b) F ′ (t)
t F (t)
(c) ∫0 F (u) du (d)
t
⋅ (Rohilkhand 2002)

1
7. Inverse Laplace transform of is
p + a2
2

(a) sin at (b) cos at


1
(c) sin at (d) sinh at. (Rohilkhand 2003)
a
 p 
8. The value of L−1  2  is
 2 p − 8 

(a) cosh 2t (b) sinh 2t


1 1
(c) sinh 2 t (d) cosh 2 t.
2 2
9. If L−1 { f ( p)} = F (t) and F (0 ) = 0 , then L−1 { p f ( p )} is
(a) (−1)n t n
F (t) (b) F ′ (t)
F (t) t
(c) (d) ∫ F (u) du.
t 0
∞ 2
10. The value of ∫ e − x dx is
0
1 1  π
(a) √π (b)  
2 2  2
(c) √π (d) None of these.
 1 
11. The value of L−1  3
is
 ( p − 1) 
t2 e t
(a) t2 e t (b)
2
te t
(c) (d) None of these. (Rohilkhand 2004)
2
M-81

Fill In The Blank(s)


Fill in the blanks “……” so that the following statements are complete and correct.
1. If f ( p ) is the Laplace transform of a function F (t), then F (t) is called the ……
Laplace transform of the function f ( p ).
1
2. Inverse Laplace transform of , p > a is …… .
p− a
3. If L−1 { f ( p )} = F (t) then L−1 {e − ap f ( p )} = G (t), where G (t) = …… .
4. Let F (t) and G (t) be two functions of class A and let
L−1 { f ( p )} = F (t)

and L−1 { g ( p )} = G (t), then

L−1 { f ( p ) g ( p )} = …… .

5. The value of ∫ cos x2 dx = ......
0

True or False
Write ‘T’ for true and ‘F’ for false statement.
 4 
1. The value of L−1  − 4t
 is 3 e .
 p + 4 
 1 
2. The value of L−1  2
is t2 e − t .
 ( p + 1) 
 1 
3. The value of L−1  2 3t
 is e sin t.
 p − 6 p + 10 

4. The convolution of two functions F and G obeys the commutative law.


 1  1 −t
5. The value of L−1  2t
 is (e − e ).
 ( p − 1) ( p + 2) 3

A nswers

Multiple Choice Questions


1. (a) 2. (c) 3. (c)
4. (a) 5. (d) 6. (c)
7. (c) 8. (d) 9. (b)
10. (a) 11. (b)
M-82

Fill in the Blank(s)


1. inverse 2. e a t 3. F (t − a), t > a and 0 , t < a
t 1  π
4. ∫0 F ( x) G (t − x) dx 5.  
2  2

True or False
1. T 2. F 3. T
4 T 5. F

¨
M-83

3
A pplications of L aplace
T ransform
(To Solutions of Differential Equations and Integral Equations)

1 Solution of an Ordinary Differential Equation with


Constant Coefficients
he Laplace transform is very useful in solving ordinary linear differential equations
T with constant coefficients.
Let us consider a linear differential equation with constant coefficients
dn y d n−1 y dy
+ C1 + ... + Cn−1 + Cn y = F (t) …(1)
dt n n−1 dt
dt
where F (t) is a function of the independent variable t.
Let y (0 ) = A0 , y ′ (0 ) = A1,......, y n−1(0 ) = An−1 …(2)

be the given initial or boundary conditions where A0 , A1, A2 ,..., An−1 are constants.
On taking the Laplace transform of both sides of equation (1) and using conditions (2)
we obtain an algebraic equation known as “subsidiary equation” from which
y ( p ) = L { y(t)} is determined. The required solution is then obtained by finding the
inverse Laplace transform of y ( p ).
M-84

Example 1: Solve y ′ ′ (t) + y(t) = t with y ′ (0 ) = 1, y (π) = 0 .


(Kanpur 08; Lucknow 10)
Solution: Taking Laplace transform of both sides of the given equation, we have
L{ y ′ ′ } + L{ y} = L{ t}
or p2 L{ y } − p y(0 ) − y ′ (0 ) + L{ y } = 1 / p2
or ( p2 + 1) L{ y} = Ap + 1 + 1 / p2 = Ap + ( p2 + 1) / p2 , where y (0 ) = A
p 1
or L{ y } = A ⋅ + ⋅
2
p +1 p2
 p   
−1  1 
∴ y = y (t) = AL−1   + L  2  = A cos t + t.
2
 p + 1  p 
But y (π) = 0, therefore 0 = A cos π + π or A = π.
∴ y = π cos t + t, which is the required solution.
Example 2: Solve ( D2 + m2 ) x = a cos nt, t > 0 , x, Dx equal to x0 and x1, when t = 0, n ≠ m.

Solution: Taking Laplace transform of both sides of the given equation, we have
L { x ′ ′ } + m2 L { x } = a L {cos nt}
ap
or p2 L { x} − px (0 ) − x ′ (0 ) + m2 L { x} =
p + n2
2

ap
or ( p2 + m2 ) L {x} = px0 + x1 +
p2 + n2
p 1 a. p
or L { x} = x0 ⋅ + x1 ⋅ +
2 2 2 2
p +m p +m ( p + m2 ) ( p2 + n2 )
2

p 1 ap ( p2 + m2 ) − ( p2 + n2 ) 
= x0 . + x1. +  
p2 + m2 p2 + m2 m2 − n2  ( p2 + m2 ) ( p2 + n2 ) 
 
p 1 a  p p 
= x0 . + x1 . + ⋅ − ⋅
p2 + m2 p2 + m2 (m2 − n2 )  p2 + n2 p2 + m2 
Taking the inverse transform, we have
 p   1 
x = x0 . L−1  + x1 . L−1 
2 2 2 2
 p + m   p + m 
a   p   p  
+ ⋅  L−1  − L−1  
2 2 2 2 2 2
(m − n )   p + n   p + m  
x a
or x = x0 cos mt + 1 ⋅ sin mt + (cos nt − cos mt),
m (m − n2 )
2

which is the required solution.


M-85

Example 3: Solve ( D2 + 9) y = cos 2 t if y (0 ) = 1, y (π / 2) = − 1.

(Rohilkhand 2000, 02, 04; Agra 01; Gorakhpur 07, 10, 11; Kashi 14)

Solution: Taking the Laplace transform of both sides of the given equation, we have
L { y ′ ′ } + 9 L { y } = L {cos 2 t}
or p2 L { y } − p y(0 ) − y ′ (0 ) + 9 L { y } = p /( p2 + 4)
or ( p2 + 9) L { y } − p − A = p / ( p2 + 4), where y ′ (0 ) = A
p+ A p
or L{ y} = +
2
p +9 ( p + 9) ( p2 + 4)
2

p A p p
= + + − ⋅
2 2 2
p +9 p +9 5 ( p + 4) 5 ( p2 + 9)
 p   1  1 −1  p  1 −1  p 
∴ y = L−1   + AL−1  + L  2 − L  2 
 p2 + 9   p2 + 9  5  p + 4  5  p + 9 
1 1 1
= cos 3 t + A sin 3 t + cos 2 t − cos 3 t
3 5 5
4 1 1
= cos 3 t + A sin 3 t + cos 2 t.
5 3 5
But y (π / 2) = − 1.
4 3 1 3 1
∴ − 1 = cos π + A sin π + cos π
5 2 3 2 5
1 1
or − 1= − A − or A = 12 / 5.
3 5
4 4 1
Hence the required solution is y = cos 3 t + sin 3 t + cos 2 t.
5 5 5
Example 4: Solve ( D + 1)2 y = t given that y = − 3, when t = 0 and y = − 1, when t = 1.
(Rohilkhand 2006)
2
Solution: The given equation can be written as ( D + 2 D + 1) y = t.
∴ L { y ′ ′ } + 2 L { y ′ } + L { y} = L { t}
or p2 L { y} − py (0 ) − y ′ (0 ) + 2 [ pL { y} − y (0 )] + L { y} = 1/ p2
or ( p2 + 2 p + 1) L { y } − p (− 3) − A − 2 (− 3) = 1/ p2 where y ′ (0 ) = A
or ( p + 1)2 L { y} = (1 / p2 ) − 3 p − 6 + A
1 3p + 6 A
or L{ y} = − +
2 2 2
p ( p + 1) ( p + 1) ( p + 1)2
1 3 ( p + 1) + 3 A
= − +
2 2 2
p ( p + 1) ( p + 1) ( p + 1)2
2 1 2 1 3 3 A
=− + + + − − +
p p2 p + 1 ( p + 1)2 ( p + 1) ( p + 1)2 ( p + 1)2
2 1 1 A−2
=− + − + ⋅
p p2 ( p + 1) ( p + 1)2
M-86

 1  1   1   1 
−1 
∴ y = − 2 L−1   + L−1   − L−1   + ( A − 2) L  
 p + 1
2
 p  p   ( p + 1)2 

. + t − e − t + ( A − 2) e − t L−1 {1/ p2} = − 2 + t − e − t + ( A − 2) te − t .


= − 21
Now, since y = − 1, when t = 1,
∴ − 1 = − 2 + 1 − e − 1 + ( A − 2) e − 1 or A = 3.
Hence the complete solution is y = − 2 + t − e − t + te − t .
dy
Example 5: Solve ( D2 + 1) y = t cos 2 t, y = 0 , = 0 when t = 0.
dt (Avadh 2006, 07; Meerut 13B)

Solution: Taking the Laplace transform of both the sides of the given equation, we
have L { y ′ ′ } + L { y } = L { t cos 2 t}
d
or p2 L { y } − p y (0 ) − y ′ (0 ) + L { y } = − ( L {cos 2 t })
dp

d  p  1 2 p2
or ( p2 + 1) L { y} = − =− +
dp  p2 + 4  p2 + 4 ( p2 + 4)2

p2 − 4 5 5 8
or L{ y} = =− + +
2 2 2 2 2
( p + 1) ( p + 4) 9 ( p + 1) 9 ( p + 4) 3 ( p + 4)2
2

5 −1  1  5 −1  1  8 −1  1 
∴ y=− L   + L   + L 
2 2
9  p + 1 9  p2 + 4  3 2
 ( p + 4) 
5 5 8 t1 1
= − sin t + sin 2 t + ∫ sin 2 x ⋅ sin 2 (t − x) dx ,
9 18 3 02 2
  1  1 
By the convolution theorem since L−1   = sin 2 t
2
  p + 4  2 
5 5 1 t
3 ∫0
= − sin t + sin 2 t + {cos (2 t − 4 x) − cos 2 t} dx
9 18
t
5 5 1  1 
=− sin t + sin 2 t + − 4 sin (2 t − 4 x) − x cos 2 t 
9 18 3 0
5 5 1 1 1
=− sin t + sin 2 t + sin 2 t − t cos 2 t + sin 2 t
9 18 12 3 12
5 4 1
or y = − sin t + sin 2 t − t cos 2 t , which is the required solution.
9 9 3
Example 6: Solve ( D3 − D2 − D + 1) y = 8 t e − t if y = D2 y = 0 , Dy = 1 when t = 0.

Solution: Taking the Laplace transform of both sides of the given equation, we have
L { y ′ ′ ′ } − L { y ′ ′ } − L { y ′ } + L { y } = 8 L { te − t}
or p3 L { y } − p2 y (0 ) − py ′ (0 ) − y ′ ′ (0 ) − [ p2 L { y } − p y (0 ) − y ′ (0 )]
d
− [ pL { y } − y (0 )] + L { y } = − 8 [ L {e − t}]
dp
M-87

d  1 
or ( p3 − p2 − p + 1) L { y } − p + 1 = − 8  
dp  p + 1
8
or ( p − 1)2 ( p + 1) L { y } = p − 1 +
( p + 1)2
1 8
or L{ y} = +
( p − 1) ( p + 1) ( p − 1)2 ( p + 1)3

1 1 1  3 1 3
=  −  − + +
2  p − 1 p + 1 2 ( p − 1) ( p − 1)2 2 ( p + 1)
2 2
+ +
2
( p + 1) ( p + 1)3
1 1 1 2 2
=− + + + + ⋅
p − 1 p + 1 ( p − 1)2 ( p + 1)2 ( p + 1)3

 1  −1  1 

−1  1 
∴ y = − L−1  +L  +L  
 p − 1  p + 1  ( p − 1)2 
 1   1 
+ 2 L−1  + 2 L−1 
2 
 ( p + 1)   ( p + 1)3 
 1   1   1 
= − e t + e − t + e t L−1   + 2 e − t L−1   + 2 e − t L−1  
2 2
 p   p   p3 

= − e t + e − t + e t t + 2 e − t t + 2 e − t (t 2 / 2 !)
= (1 + 2 t + t 2 ) e − t − (1 − t) e t , which is the required solution.

Example 7: Solve ( D 4 + 2 D2 + 1) y = 0 , where y (0 ) = 0 , y ′ (0 ) = 1, y ′ ′ (0 ) = 2 and

y ′ ′ ′ (0 ) = − 3.
Solution: Taking the Laplace transform of both sides of the given equation, we have
L { y iv} + 2 L { y ′ ′ } + L { y } = 0

or p4 L { y } − p3 y (0 ) − p2 y ′ (0 ) − p y ′ ′ (0 ) − y ′ ′ ′ (0 )
+ 2 [ p2 L { y } − p y (0 ) − y ′ (0 )] + L { y } = 0

or ( p4 + 2 p2 + 1) L { y } − p2 − 2 p + 3 + 2 (− 1) = 0

or ( p2 + 1)2 L { y } = p2 + 2 p − 1
p2 + 2 p − 1 1 2p − 2 1 2p 2
or L{ y} = = + = + − ⋅
2 2 2 2 2 2 2 2
( p + 1) p +1 ( p + 1) p +1 ( p + 1) ( p + 1)2
2

 1   2 p   1 
∴ y = L−1   + L−1   − 2 L−1  ⋅ …(1)
 p2 + 1  ( p2 + 1)2   ( p2 + 1)2 
M-88

 2 p   d  1    
Now L−1   = − L−1     = − (− t) L−1  1  = t sin t
2
 ( p + 1)  2  2  2
 dp  p + 1   p + 1
 1   1 1 
and L−1   = L−1  ⋅  = F (t) * F (t),
 ( p2 + 1)2   p2 + 1 p2 + 1
 1 
where F (t) = L−1   = sin t
2
 p + 1
t 1 t
= ∫0 sin x ⋅ sin (t − x) dx =
2 ∫0
[cos (2 x − t) − cos t] dx
t
1 1  1
= 2 sin (2 x − t) − x cos t  = (sin t − t cos t).
2 0 2

Putting in (1), the required solution is


y = sin t + t sin t − (sin t − t cos t) or y = t (sin t + cos t).
Example 8: Find the general solution of the differential equation
x ′ ′ (t) + k 2 x (t) = F (t); x(0 ) = A, x ′ (0 ) = B.
Solution: Taking the Laplace transform of both sides of the given equation, we have
L { x ′ ′ } + k 2 L { x } = L {F (t)}
or p2 L { x } − px(0 ) − x ′ (0 ) + k 2 L { x} = L {F (t)}
or ( p2 + k 2 ) L { x } = pA + B + f ( p), where L {F (t)} = f ( p)
p 1 1
or L { x } = A. + B. + f ( p).
2 2 2 2
p +k p +k p + k2
2

 p   1   1 
∴ x = AL−1   + BL−1   + L−1  ⋅ f ( p) ⋅
 p2 + k 2   p2 + k 2   p2 + k 2 
= A cos kt + ( B / k ) sin kt + {(1/ k ) sin kt} * F (t)
t
x (t) = A cos kt + ( B / k ) sin kt + (1/ k ) ∫0 sin k (t − x) ⋅ F ( x) dx.

[By the convolution theorem]


This is the required general solution.
d2 x
Example 9: Solve + x = F (t), if x = x ′ = 0 for t = 0.
dt2
Solution: Taking the Laplace transform of both sides of the given equation, we have
L{ x ′ ′ } + L{ x } = L {F (t)}
or p2 L{ x } − px(0 ) − x ′ (0 ) + L{ x } = f ( p), where L{F (t)} = f ( p)
1
or L { x} = f ( p).
2
p +1
M-89

 1 
∴ x = L−1  f ( p) = (sin t) * F (t)
2
 p + 1 
t
or x= ∫0 sin (t − x) ⋅ F ( x) dx , (by the convolution theorem)

which is the required solution.

Comprehensive Exercise 1

Solve the following problems by means of the Laplace transform :


dy
1. (i) + y = 1, given that y = 2 when t = 0.
dt
d2 y dy
(ii) + y = 0 , under the conditions that y = 1, = 0 when t = 0.
2 dt
dt (Rohilkhand 2002)

2. (i) ( D2 + 1) y = 6 cos 2 t, if y = 3, D y = 1, when t = 0.


(Gorakhpur 2006, 09; Rohilkhand 07; Purvanchal 07)
2
(ii) ( D + D) x = 2, when x (0 ) = 3, x ′ (0 ) = 1. (Rohilkhand 2011; Kanpur 07)

3. (i) ( D2 + m2 ) x = a sin nt, t > 0 , where x, Dx equal to x0 and x1 , when


t = 0, n ≠ m.
(ii) ( D + 2)2 y = 4 e −2 t , y (0 ) = − 1 and y ′ (0 ) = 4.

4. (i) ( D2 − D − 6) y = 2, t > 0 , if y = 1, Dy = 0 , when t = 0.

(ii) ( D2 + 6 D + 9) y = sin x, where y (0 ) = 1, y ′ (0 ) = 0.


5. ( D2 + 9) y = 18 t, if y (0 ) = 0 , y (π / 2) = 0 .
6. ( D2 + 6 D + 25) y = 208 e3 t , t > 0 , if y = 1, D y = 0 when t = 0.
7. ( D2 − 4 D + 5) y = 125 t2 , if y = 0 = D y , when t = 0.
8. ( D2 + 2 D + 1) y = 3 t e − t , t > 0 , subject to the conditions, y = 4, D y = 2 when
t = 0.
9. ( D2 − 3 D + 2) y = 1 − e 2 t , y = 1, D y = 0 when t = 0.
(Avadh 08; Gorakhpur 05)
2
10. ( D + 1) y = sin t sin 2 t, t > 0 , if y = 1, Dy = 0 when t = 0.
11. ( D3 − D) y = 2 cos t, y = 3, D y = 2, D2 y = 1, when t = 0.

12. ( D3 + D) y = e2 t , y (0 ) = y ′ (0 ) = y ′ ′ (0 ) = 0 .

13. ( D3 − 2 D2 + 5 D) y = 0 , if y (0 ) = 0 , y ′ (0 ) = 1, y (π / 8) = 1.

14. ( D2 + D) y = t2 + 2 t, where y (0 ) = 4, y′ (0 ) = − 2.
M-90

A nswers 1
1. (i) y = e − t + 1 (ii) y = cos t
2. (i) y = 5 cos t + sin t − 2 cos 2 t (ii) y = 2 + 2 t + e − t
x a  n 
3. (i) x = x0 cos mt + 1 sin mt + sin nt − sin mt
m m2 − n2  m 

(ii) y = e −2 t (2 t2 + 2 t − 1)
1 4 8 3t
4. (i) y = − + e −2 t + e
3 5 15
1
(ii) y = [(53 + 155 x) e −3 x − (3 cos x − 4 sin x)]
50
5. y = π sin 3 t + 2 t
3
6. y = 4 e3 t − e −3 t (4 cos 4 t + 7 sin 4 t)
4
7. y = 25 t2 + 40 t + 22 + 2 e2 t (2 sin t − 11 cos t)
1 1 1 2t
8. y = e − t . t3 + 4 e − t + 6 te − t 9. y= + e − te2 t
2 2 2
15 1 1
10. y= cos t + t sin t + cos 3 t
16 4 16
11. y = 3 sinh t − sin t + cosh t + 2
1 1 2t 2 1
12. y=− + e + cos t − sin t
2 10 5 5
1 3
13. y = 1 + e t (sin 2 t − cos 2 t) 14. y = t + 2 (1 + e − t )
3

2 Solution of Ordinary Differential Equations


with Variable Coefficients
The Laplace transform can be used in solving some ordinary differential equations with
variable coefficients. The method is found useful in case of the equations having the
dm
terms of the form t m y n (t) whose Laplace transform is (− 1)m L { y n (t)}.
dp m

Example 10: Solve ty ′ ′ + y ′ + 4 t y = 0 if y (0 ) = 3, y ′ (0 ) = 0 . (Purvanchal 2010)

Solution: Taking the Laplace transform of both sides of the given equation, we have
L { t y ′ ′ } + L { y ′ } + 4 L { t y} = 0
M-91

d d
or − L{ y ′ ′ } + L{ y ′ } + 4. (− 1) L{ y } = 0
dp dp
d 2 d
or − [ p y − py (0 ) − y ′ (0 )] + [ p y − y(0 )] − 4 y = 0,
dp dp
where y = L{ y }
d d y
or − ( p2 y − 3 p) + ( p y − 3) − 4 =0
dp dp
d y d y p
or − ( p2 + 4) − p y = 0 or + dp = 0.
2
dp y p +4
1 C1
Integrating, log y + log ( p2 + 4) = log C1 or y= ⋅
2 √ ( p2 + 4)
 1 
∴ y = L−1 { y } = C1 L−1  
2
 √ ( p + 4)
or y = C1 J0 (2 t) (See Ex. 25 of Chapter 1)
Since y(0 ) = 3, therefore 3 = C1 J0 (0 ) = C1. [ ∵ J0 (0 ) = 1]
Hence y = 3 J0 (2 t), which is the required solution.
Example 11: Solve [t D2 + (1 − 2 t) D − 2] y = 0 if y(0 ) = 1, y ′ (0 ) = 2.

Solution: The given equation can be written as ty ′ ′ + y ′ − 2 t y ′ − 2 y = 0.


∴ L {t y ′ ′} + L { y ′} − 2 L {t y ′} − 2 L { y } = 0
d d
or − L { y ′ ′} + L { y ′} + 2 L { y ′} − 2 L { y } = 0
dp dp
d
or − [ p2 y − p y(0 ) − y ′ (0 )] + [ p y − y (0 )]
dp
d
+2 [ p y − y (0 )] − 2 y = 0 , where y = L { y }
dp
d d
or − ( p2 y − p − 2) + ( p y − 1) + 2 ( p y − 1) − 2 y = 0
dp dp
d y d y 1
or − ( p2 − 2 p) − p y =0 or + dp = 0.
dp y p−2
Integrating, log y + log ( p − 2) = log C1 or y = C1 / ( p − 2).
−1 −1
∴ y=L { y } = C1 L {1/( p − 2)} = C1 e 2 t .
But y (0 ) = 1.
∴ 1 = C1.
2t
Hence y = e , which is the required solution.

Example 12: Solve y ′ ′ − ty ′ + y = 1, if y (0 ) = 1, y ′ (0 ) = 2.


Solution: Taking the Laplace transform of both sides of the given equation, we have
L{ y ′ ′ } − L { t y ′ } + L { y} = L {1}
M-92

d 1
or p2 y − py (0 ) − y ′ (0 ) + [ L { y ′ }] + y =
dp p
d 1
or p2 y − p − 2 + [ p y − y(0 )] + y =
dp p
d 1
or p2 y − p − 2 + ( p y − 1) + y =
dp p
d y 1
or p + ( p2 + 2) y = p + 2 +
dp p
d y  2 2 1
or +  p +  y = 1+ +
dp  p p p2 …(1)

which is a linear differential equation in y.


2 2
∴ I. F. = e ∫ ( p + 2 / p) dp = e p /2 + 2 log p = p2 e p /2.
∴ solution of (1) is given by
 
1 + 2 + 2  p2 e p /2 dp = c1 +
2 2 2
p2 e p /2 y = c1 + ∫ ∫ ( p2 + 2 p + 1) e p /2 dp
 2
p p  

2 2
= c1 + ∫ ( p2 + 1) e p /2 dp + 2 ∫ pe p /2 dp
dv dv
∫ (2 v + 1) e v ⋅ ∫ √ (2v) e
v
= c1 + +2 ,
√ (2 v) √ (2 v)
p2
putting = v so that p dp = dv or dp = dv / √ (2 v)
2
ev
∫ √ (2v) e
v
∫ ∫e
v
= c1 + dv + dv + 2 dv
√ (2 v)
2 2
= c1 + √ (2 v) e v + 2 e v = c1 + pe p /2 + 2 e p /2
c 2 1 2
or y = L { y } = 1 e − p /2 + +
2
p p p2

c  p2 p4  1 2
= 1 1 − + −... + +
2  2 4 ⋅2 !  p p2
p  
(2 + c1) c c 1
= − 1 + 1 p2 − ... + ⋅
2 2 8 p
p
1 1
∴ y = (2 + c1) L−1 {1/ p2} − c1 L−1 {1} + c1 L−1 { p2} + ... + L−1 {1/ p}
2 8
= (2 + c1) t + 1, since L−1 { pn} = 0 , n = 0 , 1, 2, ...
But given y ′ (0 ) = 2. Therefore,
2 = 2 + c1 or c1 = 0 .
Hence y = 2 t + 1, which is the required solution.
M-93

Comprehensive Exercise 2

Solve the following problems by means of the Laplace transform:


1. y ′ ′ + t y ′ − y = 0, if y (0 ) = 0 , y ′ (0 ) = 1. (Agra 2002; Purvanchal 11)
2
2. [t D + (t − 1) D − 1] y = 0 , if y (0 ) = 5, y (∞) = 0 .
3. y ′ ′ (t) + at y ′ (t) − 2 a y (t) = 1, y (0 ) = y ′ (0 ) = 0 , a > 0 .
4. ty ′ ′ + 2 y ′ + t y = 0 , y (0 +) = 1, y (π) = 0 .
5. Solve ty ′ ′ (t) + (2 t + 3) y ′ (t) + (t + 3) y (t) = ae − t if y (t) and its derivatives
have transforms.

A nswers 2
1. y=t 2. y = 5e− t
1 sin t
3. y = t2 4. y =
2 t
 at 
5. y =  A +  e− t
 3

3 Solution of Simultaneous Ordinary Differential


Equations
The Laplace transform can also be used in solving two or more simultaneous ordinary
differential equations.

Example 13: Solve Dx + D y = t ; D 2 x − y = e − t if x (0 ) = 3, x ′ (0 ) = − 2, y (0 ) = 0 .

(Rohilkhand 2010)

Solution: Taking the Laplace transform of both sides of the two equations, we have
L { x ′ } + L { y ′ } = L { t} and L { x ′ ′ } − L { y} = L { e − t}
or p x − x (0 ) + p y − y (0 ) = 1 / p2
and p2 x − p x (0 ) − x ′ (0 ) − y = 1 / ( p + 1)
or p x + p y = 3 + 1/ p2 and p2 x − y = 3 p − 2 + 1/( p + 1).
Solving for x and y, we have
3 p2 + 1 3p 2 1
x= + − −
3 2
p (1 + p ) 1 + p2 1 + p2 ( p + 1) ( p2 + 1)
M-94

1  2 p4 
 +
3p 2 1 p 1
= 1 + 2 p2 − − + − +
3  2 2 2 2 ( p + 1) 2 ( p + 1) 2 ( p2 + 1)
2
p  1+ p  1+ p 1+ p
2 1 1 p 3
= + + + −
p p3 2 ( p + 1) 2 ( 1 + p2 ) 2 ( p2 + 1)
1 2
and y= +
p ( p + 1) ( p2 + 1) p2 + 1
1 1 p 1 2
= − − − +
p 2 ( p + 1) 2 ( p2 + 1) 2 ( p2 + 1) p2 + 1

1 1 p 3
= − − + ⋅
p 2 ( p + 1) 2 ( p2 + 1) 2 ( p2 + 1)

 1  1  1 −1  1 
∴ x = 2 L−1   + L−1  + L 
3 
 p  p  2  p + 1

1 −1  p  3 −1  1 
+ L  − L  2 
2
2  p + 1 2  p + 1
1 2 1 −t 1 3
=2+ t + e + cos t − sin t
2 2 2 2
 1 1  1  1 −1  p  3 −1  1 
and y = L−1   − L−1  − L  2 + L  2 
 p 2  p + 1 2  p + 1 2  p + 1
1 1 3
= 1 − e − t − cos t + sin t.
2 2 2
Example 14: Solve Dx + 2 D 2 y = e − t , ( D + 2) x − y = 1 if x (0 ) = y (0 ) = y ′ (0 ) = 0 .

Solution: Taking the Laplace transform of both sides of the given equations, we have
L { x ′ } + 2 L { y ′ ′ } = L { e− t }
and L { x ′ } + 2 L { x} − L { y} = L {1}
1
or p x − x (0 ) + 2 [ p2 y − py (0 ) − y ′ (0 )] = ,
p+1
where x = L { x } and y = L { y}
1
and p x − x (0 ) + 2 x − y =
p
2 1 1
or p x + 2p y = and ( p + 2) x − y = ⋅
p+1 p
Solving for x and y, we have
1 2
x= +
2 2
p ( p + 1) (2 p + 4 p + 1) (2 p + 4 p + 1)
1 1
= +
 2 − √ 2  2 + √ 2  2 − √ 2  2 + √ 2
2 p ( p + 1)  p +   p +   p +   p + 
 2   2   2   2 
M-95

1 1 1 1
= + − −
p p+1  2 − √ 2  2 + √ 2
(2 − √ 2)  p +  (2 + √ 2)  p + 
 2   2 
1 1
+ −
 2 − √ 2  2 + √ 2
√ 2  p +  √ 2  p + 
 2   2 
1 1 1 1
= + − −
p p+1  2 − √ 2  2 + √ 2
 p +  p+ 
 2   2 
1
and y=
p ( p + 1) (2 p2 + 4 p + 1)
1 1 1 1
= + − −
p p+1  2 − √ 2  2 + √ 2
(2 − √ 2)  p +  (2 + √ 2)  p + 
 2   2 
1 1  2 + √ 2 1  2 − √ 2 1
= + −   −  
p p+1  2   2 − √ 2  2   2 + √ 2
 p +   p + 
 2   2 
 1  1  −1  1  −1  1 
∴ x = L−1   + L−1  −L  −L  
 p  p + 1  p + a  p + b
= 1 + e − t − e − at − e − b t
 1  1  −1  1  −1  1 
and y = L−1   + L−1  −bL   − aL  
 p  p + 1  p + a  p + b
2 − √2 2 + √2
= 1 + e − t − be − a t − ae − b t , where a= and b = ⋅
2 2
Example 15: Solve ( D2 − 3) x − 4 y = 0 , x + ( D2 + 1) y = 0, t > 0

if x = y = D y = 0 , D x = 2 when t = 0.
Solution: Taking the Laplace transform of both sides of the two equations, we have
L { x ′ ′ } − 3 L { x } − 4 L { y } = 0 and L { x } + L { y ′ ′ } + L { y } = 0
or p2 x − px (0 ) − x ′ (0 ) − 3 x − 4 y = 0 , where x = L { x }
and y = L{ y}
2
and x + p y − py (0 ) − y ′ (0 ) + y = 0
or ( p2 − 3) x − 4 y = 2 and x + ( p2 + 1) y = 0 .
Solving for x and y, we have
2 ( p2 + 1) 1 1
x= = +
2 2 2
( p − 1) ( p − 1) ( p + 1)2
M-96

−2 1  1 1 1 1 
and y= = − + − − ⋅
 p + 1 p − 1 ( p + 1)
2 2 2
( p + 1) ( p − 1) 2 ( p − 1)2 

 1   1 
∴ x = L−1  + L−1 
2 
 ( p − 1)   ( p + 1)2 

= e t L−1 {1/ p2} + e − t L−1 {1/ p2} = (e t + e − t ) t


  1      
1
− L−1  −1  1  −1  1 1
and y= +L  −L   − L−1  


2  p + 1  p − 1  ( p + 1)2   ( p − 1)2  
1 1 1
= [− e − t + e t − t e − t − te t ] = (1 − t) e t − (1 + t) e − t .
2 2 2

Comprehensive Exercise 3

Solve the following problems by means of the Laplace transform:


1. ( D − 2) x + 3 y = 0 , 2 x + ( D − 1) y = 0 , if x (0 ) = 8 and y (0 ) = 3.
2. ( D2 + 2) x − Dy = 1, Dx + ( D2 + 2) y = 0 , if x = 0 = Dx = y = Dy, when t = 0.

3. ( D2 − 1) x + 5 Dy = t, − 2 Dx + ( D2 − 4) y = − 2 ,

if x = 0 = Dx = y = D y when t = 0.
4. Applying Laplace transform solve the equations :
dx dy d2 x
+ = t and − y = e− t,
dt dt 2
dt
dx
given that x (0 ) = 0 = y (0 ) and = 0 when t = 0. (Rohilkhand 2007)
dt
5. Solve ( D − 2) x − ( D + 1) y = 6 e3 t , (2 D − 3) x + ( D − 3) y = 6 e3 t , if x = 3, y = 0
when t = 0.
6. Solve ( D − 2) x − ( D − 2) y = sin t, ( D2 + 1) x + 2 D y = 0 ,
if x = 0 = x ′ (0 ) = y (0 ).

A nswers 3
1. x = 5 e − t + 3 e4 t , y = 5 e − t − 2 e4 t .
1 1 1
2. x= − (2 cos t + cos 2 t), y = (sin 2 t − 2 sin t).
2 6 6
3. x = − t + 5 sin t − 2 sin 2 t, y = 1 − 2 cos t + cos 2 t.
1 1 1 1 1 1 1
4. x = t 2 − 1 + e − t + cos t + sin t, y = 1 − e − t − cos t − sin t.
2 2 2 2 2 2 2
M-97

5. x = e t + 2 t e t + 2 e 3 t, y = e t − t e t − 2 e 3 t.
1 4 2t 1 1
6. x= (1 + 3 t) e − t + e − (cos t + 2 sin t), y = [(1 + 3 t) e − t − e2 t ].
9 45 5 9

4 Solution of Partial Differential Equations


Laplace transform is also useful in solving partial differential equations when the
boundary conditions are given.
Laplace transforms of some partial derivatives:
Theorem: If y ( x, t) is a function of x and t, then
 ∂y 
(a) L   = p y ( x, p) − y ( x, 0 )
 ∂t 
 ∂2 y 
  2
(b) L 
2  = p y ( x, p) − p y ( x, 0 ) − yt ( x, 0 )
 ∂t 

 ∂y  d y  ∂2 y  d 2 y
(c) L  = and (d) L  = where L { y ( x, t)} = y ( x, p).
 ∂x  dx  ∂x2  dx2
 ∂y  ∞ ∂y s ∂y
Proof: (a) L  = ∫0 e − pt dt = lim ∫0 e − pt dt
 ∂t  ∂t s→ ∞ ∂t
 
{ }
s s
= lim  e − pt y ( x, t) + p ∫ e − pt y ( x, t) dt
s→ ∞ 0 0 

=p ∫0 e − pt y ( x, t) dt − y ( x, 0 ) = py ( x, p) − y ( x, 0 )

∂y
(b) Let V = ⋅
∂t
 ∂2 y 
   ∂V 
∴ L  = L  = p . L {V } − V ( x, 0 )
∂t2  ∂t 
 
= p [ pL { y} − y ( x, 0 )] − yt ( x, 0 )  ∂y 
∵ V = = yt 
 ∂t 

= p2 y ( x, p) − py ( x, 0 ) − yt ( x, 0 ).

 ∂y  ∞ ∂y d ∞ − pt d y
∫0 e − pt
dx ∫ 0
(c) L  = dt = e y dt = ⋅
 ∂x  ∂x dx

 ∂2 y  ∂y
   ∂U 
(d) L  = L  , where U =
∂x2  ∂x  ∂x
 
M-98

2
d d  ∂y  d d y  d y
= L {U } = L  =   = ⋅
dx dx  ∂x  dx  dx  dx2

∂y ∂2 y
Example 16: Solve =2 where y (0 , t) = 0 = y (5, t) and y ( x, 0 ) = 10 sin 4 πx.
∂t ∂x 2
(Meerut 2008)
Solution: Taking the Laplace transform of both sides of given equation, we have
 ∂y   ∂2 y  d2 y
L   = 2L   or p y − y ( x, 0 ) = 2
 ∂t   ∂x2  dx2

d2 y p
or − y = − 5 sin 4 πx whose general solution is
2 2
dx
5 sin 4 πx
y = C1e √( p /2 ) x + C2 e − √( p /2 ) x −
− (4 π)2 − p / 2
10
or y = C1e √( p /2 ) x + C2 e − √( p /2 ) x + sin 4 πx …(1)
32 π2 + p

But y (0 , t) = 0 = y (5, t). Therefore, y (0 , p) = 0 , y (5, p) = 0 .


∴ from (1), we have 0 = C1 + C2
10
and 0 = C1e5 √( p /2 ) + C2 e −5 √( p /2 ) + sin 20 π
32 π2 + p

= C1e5 √( p /2 ) + C2 e −5 √( p /2 ) + 0 .
Solving C1 = 0 = C2 .
10
∴ from (1), we have y= sin 4 πx.
32 π2 + p
 10  2
∴ y = L−1  sin 4 πx  or y = 10 e −32 π t sin 4 πx,
 32 π2 + p 
which is the required solution.
∂y ∂2 y
Example 17: Find the bounded solution of = , x > 0, t > 0,
∂t ∂x2
where y (0 , t) = 1, y ( x, 0 ) = 0 . (Meerut 2007)

Solution: Taking the Laplace transform of both the sides of the given equation, we
have
 ∂y   ∂2 y  d2 y
L  = L p y ( x, p) − y ( x, 0 ) =
2
or
 ∂t   ∂x  dx2

d2 y
or − p y =0
dx2
M-99

whose general solution is y = c1 e √ px + c2 e −√ px .


But y ( x, t) must be bounded as x → ∞.
∴ y ( x, p) = L { y ( x, t)} must also be bounded as x → ∞
∴ c1 = 0 so that y = c2 e −√ px , if √ p > 0. …(1)
But y (0 , t) = 1. Therefore,
L { y (0 , t)} = L {1} or y (0 , p) = 1/ p …(2)
Thus, from (1) and (2) we have 1/ p = c2 .
∴ y = (1/ p) e − √ px
 1   x 
or y = L−1   = erfc   , which is the required solution.
 pe √ px   2 √ t
[See Ex. 21 of Chapter 2]
∂2 y ∂2 y ∂y
Example 18: Solve − = xt where y = 0 = at t = 0 and y (0 , t) = 0 .
∂x2 ∂t2 ∂t
Solution: Taking the Laplace transform of both the sides of the given equation, we
have
 ∂2 y   ∂2 y 
L  2  − L  2  = L {xt}
 ∂x   ∂t 
2
d y
or − [ p2 y ( x, p) − p y ( x, 0 ) − yt ( x, 0 )] = x L { t}
dx2
d2 y
or − p2 y = x / p2 whose general solution is
dx2
y = c1 e px + c2 e − px − x / p4 .
Since y = 0 for all values of x, therefore, c1 = 0 , otherwise
y = ∞, as x → ∞.
∴ y = c2 e − px − x / p4 .
Again y = 0 when x = 0; ∴ c2 = 0 .
1 3
y = − x / p4 or y = − L−1 { x / p4} = − xt ,
6
which is the required solution.

Comprehensive Exercise 4

Solve the following problems by means of the Laplace transform:


∂y ∂y
1. =2 + y, y ( x, 0 ) = 6 e −3 x which is bounded for x > 0 , t > 0 .
∂x ∂t
∂y ∂2 y π   ∂y 
2. = 3 2 , where y  , t  = 0 ,   = 0 and y ( x, 0 ) = 30 cos 5 x.
∂t ∂x 2   ∂x  x = 0

(Meerut 2006)
M-100

∂y ∂2 y
3. = 2 , y ( x, 0 ) = 3 sin 2 π x , y (0 , t) = 0 = y (1, t), 0 < x < 1, t > 0 .
∂t ∂x
(Meerut 2006)
2
∂y ∂ y
4. = 2 2 , y (0 , t) = 0 , y (5, t) = 0 , y ( x, 0 ) = 10 sin 4 πx − 5 sin 6 πx.
∂t ∂x
(Meerut 2008)
∂u ∂u
5. − = 1 − e − t , 0 < x < 1, t > 0 , u ( x, 0 ) = x.
∂x ∂t (Meerut 2007)

A nswers 4
1. y ( x, t) = 6 e − 2 t − 3 x . 2. y = 30 e − 75 t cos 5 x.
2
3. y ( x, t) = 3 e − 4 π t
sin 2 πx.
2 2
4. y ( x, t) = − 5 e − 72 π t
sin 6 πx + 10 e − 32 π t
sin 4 πx.
−t
5. u ( x, t) = ( x + 1) − e .

5 Integral Equations
1. Integral equation: An equation of the form
b
F (t) = y (t) + ∫a K (u, t) F (u) du

is called an “Integral equation” where y (t) and K (u, t) are known, a and b are either
constants or functions of t.
Here the function F (t) which appears under the sign of integration is to be determined.
2. Abel’s integral equation: An equation of the form
t F (u) du
G (t) = ∫
0 (t − u)n

is called Abel’s integral equation where F (u) is unknown and G (t) is known and n is a
constant between 0 and 1, i. e., 0 < n < 1.
3. Integral equation of Convolution type: An integral equation of the form
t
F (t) = y (t) + ∫ 0 K (t − u) F (u) du
which may also be expressed as
F (t) = y (t) + K (t) * F (t)
is called an integral equation of convolution type.
4. Integro-differential equation: An integral equation in which various derivatives of
the unknown function F (t) can also be present is called an Integro-differential equation.
t
For example, F ′ (t) = F (t) + y (t) + ∫ 0 sin (t − u) F (u) du.
M-101

6 Application of Laplace Transform to


Integral Equations
Laplace transform may be used to solve various integral equations.

Example 19: Solve the integral equation


t
F (t) = e − t − 2 ∫ cos (t − u) F (u) du.
0 (Rohilkhand 2011)

Solution: The given integral equation may be expressed as


F (t) = e − t − 2 F (t) * cos t.
Taking the Laplace transform of both the sides, we have
L { F (t)} = L { e − t} − 2 L { F (t) * cos t}
1
= − 2 . L { F (t)} . L {cos t}
p+1
1 2p
= − 2 ⋅ L { F (t)}
p+1 p +1
 2p  1
or 1 +  L { F (t)} =
 p 2 + 1 p+1

( p 2 + 1)
or L { F (t)} = ⋅
( p + 1)3
 p 2 + 1   ( p + 1 − 1)2 + 1
∴ F (t) = L−1  3
= L−1  3 
 ( p + 1)   ( p + 1) 
 ( p − 1)2 + 1  1 2 2 
= e − t L− 1  =e L
− t −1
 − 2 + 3
 p3   p p p 
 t t 3 
= e − t 1 − 2 ⋅ +2⋅ 
 Γ (2) Γ (3)
= e − t (1 − 2 t + t 2 ) = e − t (1 − t)2 .
Example 20: Solve the integral equation
t
F (t) = 1 + ∫0 F (u) ⋅ sin (t − u) du

and verify your solution .


Solution: The given integral equation may be expressed as
F (t) = 1 + F (t) * sin t.
Taking the Laplace transform of both the sides, we have
L { F (t)} = L {1} + L { F (t) * sin t}
= L {1} + L { F (t)} . L {sin t}
1 1
= + L { F (t)} ⋅ 2
p p +1
M-102

 1  1 p2 +1
or 1 − 2  L { F (t)} = or L { F (t)} = ⋅
 p + 1 p p3

 p + 1  1 
2  1
∴ F (t) = L−1  3  = L−1   + L−1  3 
 p   p  p 
t2 t2
= 1+ = 1+ ⋅
Γ (3) 2
t2
Verification: We have F (t) = 1 + ⋅
2
Putting in the R.H.S. of the given equation, we have
t  u2 
R.H.S. = 1 + ∫ 1 +  ⋅ sin (t − u) du
0  2 
t
 u2   t
= 1 + 1 +
 2
 cos (t − u) −
0
∫0 u ⋅ cos (t − u) du

t2 t
= 1+ 1+
2
− cos t − [− u ⋅ sin (t − u)]0t − ∫0 sin (t − u) du

t2 t2
− cos t − [cos (t − u)]0 = 2 +
t
=2+ − cos t − (1 − cos t)
2 2
t2
= 1+ = F (t) = L.H. S.
2
Example 21: Solve the following equation for F (t) with the condition that F (0 ) = 0 ,
t
F ′ (t) = sin t + ∫0 F (t − u) cos u du .

Solution: The given equation may be expressed as


F ′ (t) = sin t + F (t) * cos (t).
Taking the Laplace transform of both the sides, we have
L { F ′ (t)} = L {sin t} + L { F (t) * cos t}
1
or pL { F (t)} − F (0 ) = 2 + L { F (t)} . L {cos t}
p +1
1 p
or pL { F (t)} = 2 + L { F (t)} . 2
p +1 p +1
 p  1 1
or p−  L { F (t)} = or L { F (t)} = ⋅
 2
+  2
+1 p3
 p 1 p

 1  t2 t2
∴ F (t) = L−1  3  = = ⋅
 p  Γ (3) 2
t
Example 22: Solve the integral equation ∫0 F (u) F (t − u) du = 16 sin 4 t.

Solution: The given integral equation may be expressed as


F (t) * F (t) = 16 sin 4 t.
M-103

Taking the Laplace transform of both the sides, we have


L { F (t) * F (t)} = 16 L {sin 4 t}
4
or L { F (t)} . L { F (t)} = 16 ⋅ 2
p + 42
8
or L { F (t)} = ± ⋅
√ ( p 2 + 42 )
 1 
∴ F (t) = ± 8 L−1  2 2 
= ± 8 J0 (4 t).
 √ ( p + 4 )

(See Q. 2 of Comprehensive Exercise 5, Chapter 2)


t F (u) du
Example 23: Solve the integral equation ∫0 (t − u)1 /3
= t (1 + t).

Solution: The given integral equation may be expressed as


t
∫0 F (u) ⋅ (t − u)−1 /3 du = t + t2
− 1 /3
or F (t) * t = t + t 2.

Taking the Laplace transform of both the sides, we have


L { F (t) * t − 1 /3} = L { t + t 2}
− 1 /3
or L { F (t)} . L { t } = L { t} + L { t 2}
1
Γ (− + 1) Γ (2) Γ (3)
or L { F (t)} ⋅ − 13/3 + 1 = 2 + 3
p p p
Γ (2 / 3) 1 2
or L { F (t)} ⋅ = 2 + 3
p 2 /3 p p
1  1 2 
or L { F (t)} = ⋅  4 /3 + 7 /3  ⋅
Γ (2 / 3)  p p 
1   1   1  
∴ F (t) = ⋅  L−1  4 /3  +2 L
−1
 7 /3 
Γ (2 / 3)   p   p  
1  t (4 /3) − 1 t (7 /3) − 1 
= ⋅ +2⋅ 
Γ (2 / 3)  Γ (4 / 3) Γ (7 / 3)
 
1  t 1 /3 t 4 /3 
= ⋅ +2⋅ 
1
Γ (2 / 3)  ⋅ Γ (1/ 3) 4 1
⋅ ⋅ Γ (1/ 3)
3 3 3 
3 t 1 /3  3 
= 1+ t
Γ (1 − 13 ) Γ (13 )  2 
3 t 1 /3  3   π 
= 1 + t  ∵ Γ (n) Γ (1 − n) = 
1 sin nπ 
(π / sin π)  2  
3
3 √ 3 1 /3
= ⋅ t (2 + 3 t).

M-104

t
Example 24: Solve F ′ (t) = t + ∫0 F (t − u) cos u du, F (0 ) = 4.

Solution: The given equation may be expressed as


F ′ (t) = t + F (t) * cos t.
Taking Laplace transform of both the sides, we have
L { F ′ (t)} = L { t} + L { F (t) * cos t}
1
or pL { F (t)} − F (0 ) = 2 + L { F (t)} . L {cos t}
p
1 p
or pL { F (t)} − 4 = + L { F (t)} ⋅
p2 p2 +1
p3 1
or L { F (t)} ⋅ = +4
p2 + 1 p2
p2 + 1 4 ( p2 + 1)
or L { F (t)} = 5
+
p p3
1 1 4 4 4 5 1
= + + + = + 3 + 5 ⋅
p3 p5 p p3 p p p

 4 5 1  5 t2 t4
∴ F (t) = L−1  + 3 + 5  = 4 + +
 p p p  Γ (3) Γ (5)
5 t2 t4 5 t4
=4+ + = 4 + t2 + ⋅
2 4! 2 24
Example 25: Express
2 F ′ ′ (t) − 3 F ′ (t) − 2 F (t) = 4 e − t + 2 cos t, F (0 ) = 4, F ′ (0 ) = − 1
into an integral equation.
Solution: We have
2 F ′ ′ (t) − 3 F ′ (t) − 2 F (t) = 4 e − t + 2 cos t, …(1)
F (0 ) = 4, F ′ (0 ) = − 1 …(2)
Method 1: Let F ′ ′ (t) = G (t). …(3)
Integrating (3), we get
t
F ′ (t) = ∫0 G (u) du + C1. …(4)

For t = 0, (4) gives


− 1 = F ′ (0 ) = 0 + C1 or C1 = − 1.
t
∴ F ′ (t) = ∫0 G (u) du − 1. …(5)

Again integrating, we get


t
F (t) = ∫0 (t − u) G (u) du − t + C2 .

For t = 0, this gives


4 = F (0 ) = 0 − 0 + C2 or C2 = 4.
M-105
t
∴ F (t) = ∫0 (t − u) G (u) du − t + 4 …(6)

Putting the values of F ′ ′ (t), F ′ (t) and F (t) from (3), (5) and (6) in (1), we get
t t
2 G (t) − 3 ∫0 G (u) du + 3 − 2 ∫
0
(t − u) G (u) du + 2 t − 8

= 4 e − t + 2 cos t
t
or 2 G (t) + ∫0 (− 2 t + 2 u − 3) G (u) du = 4 e − t + 2 cos t − 2 t + 5 ,

which is the required integral equation.


Method 2: Integrating (1) between the limits 0 to t, we get
t
2 [ F ′ (u)] 0 − 3 [ F ′ (u)] 0 − 2 F (u) du = − 4 [e − u] 0t + 2 [sin u] 0t
t t
∫0
t
or 2 F ′ (t) − 2 F ′ (0 ) − 3 F (t) + 3 F (0 ) − 2 ∫0 F (u) du

= 4 (1 − e − t ) + 2 sin t.
t
or 2 F ′ (t) − 3 F (t) − 2 ∫0 F (u) du = − 4 e − t + 2 sin t − 10 . [Using (2)]

Again integrating, between the limits 0 to t, we get


t t
2 [ F (u)] 0 − 3
t
∫0 F (u) du − 2 ∫0 (t − u) F (u) du

= 4 [e − u] 0 − 2 [cos u] 0 − 10 t.
t t

t
or 2 [ F (t) − F (0 )] + ∫0 (− 2 t + 2 u − 3) F (u) du

= 4 (e − t − 1) − 2 (cos t − 1) − 10 t.
t
or 2 F (t) + ∫0 (− 2 t + 2 u − 3) F (u) du = 4 e − t − 2 cos t − 10 t + 6

which is the required integral equation.


Example 26: Express the given differential equation
F ′ ′ (t) + a F (t) = 0 , F (0 ) = 0 = F (1)
as an integral equation.
Solution: We have
F ′ ′ (t) + aF (t) = 0 …(1)
F (0 ) = 0 = F (1). …(2)
Method 1: Let F ′ ′ (t) = G (t). …(3)
Integrating (3) between the limits 0 to t, we get
t
F ′ (t) = ∫ 0 G (u) du + C1 …(4)

Again integrating between the limits 0 to t, we get


t
F (t) = ∫0 (t − u) G (u) du + C1 t + C2 . …(5)
M-106

For t = 0, (5) gives


0 = F (0 ) = 0 + C2 or C2 = 0 .
Putting this value in (5), we get
t
F (t) = ∫ 0 (t − u) G (u) du + C1 t …(6)

For t = 1, (6) gives


1
0 = F (1) = ∫0 (1 − u) G (u) du + C1

1
or C1 = − ∫0 (1 − u) G (u) du.

Putting this value of C1 in (6), we get


t 1
F (t) = ∫ 0 (t − u) G (u) du − ∫ 0 t (1 − u) G (u) du

t 1
= ∫0 (t − u) G (u) du + ∫0 (ut − t) G (u) du

t t 1
= ∫0 (t − u) G (u) du + ∫ 0 (ut − t) G (u) du + ∫ t (ut − t) G (u) du
t 1
= ∫0 (t − 1) u G (u) du + ∫t t (u − 1) G (u) du

1 (t − 1) u , if u < t
or F (t) = ∫0 K (t, u) G (u) du, where K (t, u) = 
 t (u − 1) , if u > t.
Hence from (1), we get
t (t − 1) u , u < t
G (t) + a ∫0 K (t, u) G (u) du = 0 , where K (t, u) = 
(u − 1) t , u > t.
This is the required integral equation of the given differential equation.
Method 2: Integrating (1) between the limits 0 to t, we get
t t
∫0 F ′ ′ (u) du + a ∫0 F (u) du = 0
t
or [ F ′ (u)] 0t a + ∫ 0 F (u) du = 0
t
or F ′ (t) − F ′ (0 ) + a ∫ 0 F (u) du = 0.
Again integrating between the same limits, we get
t
[ F (u)] 0t − F ′ (0) [u]0t + a ∫ 0 (t − u) F (u) du = 0
t
or F (t) − F (0 ) − tF ′ (0 ) + a ∫ 0 (t − u) F (u) du = 0 …(1)
t
or F (t) − t F ′ (0 ) + a ∫ 0 (t − u) F (u) du = 0
For t = 1, this gives
1
F (1) − F ′ (0 ) + a ∫ 0 (1 − u) F (u) du = 0 [Using (2)]
M-107
1
or 0 − F ′ (0 ) + a ∫0 (1 − u) F (u) du = 0
1
or F ′ (0 ) = a ∫ 0 (1 − u) F (u) du.
Putting this value in (I), we get
1 t
F (t) − ta ∫0 (1 − u) F (u) du + a ∫0 (t − u) F (u) du = 0
1 t
or F (t) + a ∫ 0 t (u − 1) F (u) du + a ∫ 0 (t − u) F (u) du = 0
t 1
or F (t) + a ∫ 0 t (u − 1) F (u) du + a ∫ t t (u − 1) F (u) du
t
+a ∫0 (t − u) F (u) du = 0
t 1
or F (t) + a ∫ 0 u (t − 1) F (u) du + a ∫ t t (u − 1) F (u) du = 0
1  u (t − 1) , u< t
or F (t) + a ∫0 K (u, t) F (u) du = 0 , where K (u, t) = 
 t (u − 1) , u > t.
Example 27: Convert the given integral equation
t
F (t) − ∫0 (t − u) sec t ⋅ F (u) du = t

into differential equation and associated conditions.


Solution: The given integral equation is
t
F (t) − ∫ 0 (t − u) sec t ⋅ F (u) du = t …(1)

We know that,
d b (t) b (t) ∂K db da
dt ∫ a (t) K (u, t) du = ∫ a (t) ∂t
du + K (b, t)
dt
− K (a, t)
dt
⋅ …(2)

Differentiating (1) using the fact (2), we get


t
F ′ (t) − ∫ 0 [sec t + (t − u) sec t tan t] F (u) du = 1
t t
or F ′ (t) − ∫ 0 sec t ⋅ F (u) du − tan t ∫ 0 (t − u) sec t F (u) du = 1
t
or F ′ (t) − ∫ 0 sec t ⋅ F (u) du + tan t ⋅ [t − F (t)] = 1, [Using (1)] …(3)

Again differentiating (3), we get


F ′ ′ (t) + sec2 t ⋅ [t − F (t)] + tan t ⋅ [1 − F ′ (t)]
t
−sec t ⋅ tan t ∫ F (u) du − sec t ⋅ F (t) = 0 .
0

or F ′ ′ (t) + sec2 t . [t − F (t)] + tan t . [1 − F ′ (t)]


− sec t . F (t) + tan t . [1 − F ′ (t) − t tan t + tan tF (t)] = 0 ,
[Using (2)]
or F ′ ′ − F ′ . [tan t + tan t] + [− sec 2 t + tan2 t − sec t] F
+ [t sec2 t + tan t − t . tan2 t + tan t] = 0
M-108

or F ′ ′ (t) − 2 tan t . F ′ (t) − (1 + sec t) F (t) + (t + 2 tan t) = 0 . …(4)


Putting t = 0 in (1) and (3) respectively, we get
F (0 ) = 0 , F ′ (0 ) = 1. …(5)
Hence, (4) is the required differential equation and its associated conditions are
given by (5).

Comprehensive Exercise 5

1. Solve the integral equation


t
F (t) = a sin t − 2 ∫ 0 F (u) ⋅ cos (t − u) du.
t
2. Solve y (t) = t − 1 + ∫0 y (Γ) ⋅ sin (t − Γ) dΓ.

3. Show that the solution of the integral equation


t
F (t) = 4 t − 3 ∫0 F (u) sin (t − u) du
3
is F (t) = t + sin 2 t.
2
t
4. Solve the integral equation F (t) = 1 + 2 ∫0 F (t − u) ⋅ e −2 u du.

1 t
5. Solve y (t) = t +
6 ∫0 (t − u)3 ⋅ y (u) du.

1 2 t
6. Solve the integral equation F (t) =
2
t − ∫0 (t − u) F (u) du. (Agra 2003)

t
7. Solve 2 F (t) = 2 − t + ∫0 F (t − u) F (u) du.
t F (u) du
∫ 0 √ (t − u) = 1 + t + t
2
8. Solve .

9. Convert the given differential equation


F ′ ′ (t) + 2 F ′ (t) − 8 F (t) = 5 t2 − 3 t, F (0 ) = − 2, F ′ (0 ) = 3
into an integral equation.
10. Convert the given integral equation
t
F (t) = t2 − 3 t + 4 − 3 ∫0 (t − u)2 F (u) du

into differential equation and associated conditions.

A nswers 5
1 2 1 3
1. F (t) = at e − t 2. y (t) = − 1 + t − t + t
2 6
1
4. F (t) = 1 + 2 t 5. y (t) = (sinh t + sin t)
2
M-109

6. F (t) = 1 − cos t 7. F (t) = 1, − 1


1 8 
8. F (t) = t − 1 /2 + 2 t 1 /2 + t 3 /2 
π 3 
t
∫ 0 [2 − 8t + 8u] G (u) du − 5t
2
9. G (t) + − 21t + 22 = 0

t 5 t4 t3
or F (t) + ∫0 (− 8 t + 8 u + 2) F (u) du =
12

2
− t−2

10. F ′ ′ ′ (t) + 6 F (t) = 0 with F (0 ) = 4, F ′ (0 ) = − 3, F ′ ′ (0 ) = 2

Objective Type Questions

Multiple Choice Questions


Indicate the correct answer for each question by writing the corresponding letter from (a),
(b), (c) and (d).
1. The solution of ( D + 1) y = 0 , t > 0 , given that y = y0 when t = 0 is
(a) y0 e − t (b) y0 e t
(c) 2 y0 e − t (d) 2 y0 e t
∂ y
2. If y ( x, t) is a function of x and t, then L   is
∂ x
∂ y dy
(a) (b)
∂x dx
∂2 y d2 y
(c) (d)
∂x2 dx 2
where L { y ( x, t)} = y ( x, p).
3. An integral equation of the form
t
F (t) = y (t) + ∫0 K (t − u) F (u) du is called

(a) an integral equation of convolution type


(b) Abel’s integral equation
(c) integro-differential equation
(d) Cauchy’s integral equation

Fill in the Blank(s)


Fill in the Blanks ‘……’ so that the following statements are complete and correct.
1. On taking the Laplace transform of both sides of the given differential equation
and using the given conditions, we obtain an algebraic equation known as ……
from which y ( p) = L { y (t)} is determined.
2. The solution of the given differential equation is obtained by finding the ……
transform of y ( p), where y ( p) = L { y ( x, t)}.
M-110

 ∂ y
3. If y ( x, t) is a function of x and t, then L   = …… .
 ∂t 
 ∂2 y 
4. If y ( x, t) is a function of x and t, then L  2  = ……
 ∂x 
b
5. An equation of the form F (t) = y (t) + ∫a K (u, t) F (u) du is called …… .

equation where y (t) and K (u, t) are known, a and b are either constants or
functions of t.

True or False
Write ‘T’ for true and ‘F’ for false statement.
1. If y ( x, t) is a function of x and t then
 ∂2 y 
L  2  = y ( x, p) − py ( x, 0 ) − yt ( x, 0 ) where L { y ( x, t)} = y ( x, p).
 ∂t 

∂y ∂2 y
2. The differential equation = 2 2 with the condition y ( x, 0 ) = 10 sin 4 π x is
∂t ∂x
d2 y p
takes the form 2
− y = − 5 sin 4 π x, where L { y ( x, t)} = y ( x, p).
dx 2
t F (u)
3. An equation of the form G (t) = ∫0 (t − u)n
du is called Abel’s integral equation

where F (u) is unknown and G (t) is known and n is a constant between 0 and 1,
i. e., 0 < n < 1.
4. An integral equation in which various derivatives of the unknown function F (t)
can also be present is called an Integro transform equation.

A nswers
Multiple Choice Questions
1. (a) 2. (b) 3. (a)

Fill in the Blank(s)


1. subsidiary equation 2. inverse Laplace
3. p y ( x, p) − y ( x, 0 ), where L { y ( x, t)} = y ( x, p)
d2 y
4. , where L { y ( x, t)} = y ( x, p) 5. an integral
dx2

True or False
1. F 2. T 3. T 4. F

¨
M-111

4
F ourier T ransforms

1 Dirichlet’s Conditions
function f ( x) is said to satisfy Dirichlet's conditions in the interval (a, b), if
A (i) f ( x) is defined and is single-valued except possibly at a finite number of points
in the interval (a, b), and
(ii) f ( x) and f ′ ( x) are piecewise continuous in the interval (a, b).
These conditions play an important role in the study of Fourier series and Fourier
Transforms.

2 Fourier Series
(Meerut 2013, 13B; Rohilkhand 14)
If f ( x) is a periodic function with period 2 l i. e. f ( x + 2 l ) = f ( x) and satisfies
Dirichlet's conditions in the interval (− l, l ), then at every point of continuity we have

1  nπx nπx 
f ( x) = a0 + ∑ an cos + bn sin …(1)
2 n =1  l l 
M-112

1 l nπx
where an =
l ∫ −l f ( x) cos
l
dx …(2)

1 l nπx
and bn =
l ∫ − l f ( x) sin l dx. …(3)

The series (1) with coefficients an and bn given by (2) and (3) respectively is called the
Fourier series of f (x), and the coefficients an and bn are called the Fourier coefficients
corresponding to f ( x).
At a point of discontinuity
1
f ( x) = [ f ( x + 0 ) + f ( x − 0 )].
2
If the function f ( x) defined in the interval (− l, l ) be an even function of x i.e. if
f (− x) = f ( x), then
1 l nπx 2 l nπx
an = ∫ f ( x) cos dx = ∫ f ( x) cos dx
l − l l l 0 l
1 l nπx
and bn = ∫ f ( x) sin dx = 0 .
l − l l
Therefore in this case we get Fourier cosine series.
Again if f ( x) be an odd function of x i.e. if f (− x) = − f ( x), then
1 l nπx
an = ∫ f ( x) cos dx = 0
l −l l
1 l nπx 2 l nπx
bn = ∫
l ∫0
and f ( x) sin dx = f ( x) sin dx
l −l l l
and thus in this case we get Fourier sine series.
Note: If f ( x) is a function of period 2l but is defined only in (0 , l ), we can extend it to
(− l, 0 ) so as to be an even or an odd function of x in the interval (− l , l )

3 Fourier’s Integral Formula (Fourier’s Integral Theorem)


Let f ( x) be a function satisfying Dirichlet's conditions in every finite interval − l ≤ x ≤ l
1
and defined as [ f ( x + 0 ) + f ( x − 0 )] at every point of discontinuity.
2

Further let ∫ − ∞ | f ( x)| dx converge i. e., f ( x)is absolutely integrable in − ∞ < x < ∞,then by
Fourier’s integral formula
1 ∞  ∞ 
f ( x) =
2π ∫ −∞ f (v)  ∫
 −∞
cos w ( x − v) dw dv

…(1)

1 ∞ ∞
or f ( x) =
2π ∫ −∞ dw ∫ −∞ f (v) cos w ( x − v) dv.

The representation (1) of f ( x) is known as Fourier’s integral formula.


The proof is out of scope of this book.
M-113

Other Forms of Fourier Integral Formula (Fourier Integral Theorem):


1. Complex or exponential Form of Fourier Integral Formula:
We have
1 ∞  ∞ 
0=
2π ∫ −∞ f (v)  ∫
 −∞
sin w ( x − v) dw dv

…(2)

Multiplying both sides of (2) by i and then adding to (1), we get


1 ∞  ∞ iw ( x − v ) 
2 π ∫ −∞
f ( x) = f (v)  ∫ e dw dv
 −∞ 
1 ∞ ∞
or f ( x) = ∫ −∞ e iw x dw ∫ f (v) e − iwv dv.
2π −∞

2. General Form of Fourier Integral Formula:


1 ∞  ∞ 
(i) f ( x) = ∫
π −∞
f (v)  ∫
 0
cos w( x − v) dw dv.


(ii) f ( x) = ∫0 { P (w)cos wx + Q (w)sin wx} dw

1 ∞
π ∫− ∞
where P (w) = f (v)cos wv dv

1 ∞
π ∫− ∞
and Q (w) = f (v)sin wv dv.

3. Fourier Sine Integral Formula:


If f ( x) is an odd function, then
2 ∞  ∞ 
f ( x) =
π ∫0
sin wx  ∫
 0
f (v)sin wv dv dw.

4. Fourier Cosine Integral Formula:
If f ( x) is an even function, then
2 ∞  ∞ 
π ∫0
f ( x) = cos wx  ∫ f (v)cos wv dv dw.
 0 

Example 1: Using Fourier integral formula prove that


2 ∞ (u2 + 2)cos ux
e − x cos x =
π ∫0
du.
(u4 + 4) (Gorakhpur 2007)

Solution: We know that the Fourier cosine integral formula is


2 ∞  ∞ 
f ( x) = ∫ cos ux ∫ f (v)cos uv dv du. ...(1)
π 0  0 
Now let f ( x) = e − x cos x
then f (v) = e − v cos v.
M-114

∴ form (1), we get


2 ∞  ∞ 
e − x cos x = ∫ cos ux ∫ e − v cos v cos uv dv du
π 0  0 
1 ∞  ∞ 
= ∫ cos ux ∫ e − v (2 cos uv cos v) dv du
π 0  0 
1 ∞  ∞ 
cos ux ∫ e − v {cos (u + 1) v + cos (u − 1) v } dv du
π ∫0
=
 0 
1 ∞  ∞ ∞ 
cos ux ∫ e − v cos(u + 1) vdv + ∫ e − v cos (u − 1)v dv du
π ∫0
=
 0 0 
1 ∞  1 1 
π ∫0
= cos ux  +  du
2
1 + (u + 1) 1 + (u − 1)2 
 ∞ −a x a 
∵ ∫0 e cos bx dx = 
 a + b2 
2

1 ∞ (u2 + 2 − 2 u) + (u2 + 2 + 2 u)
π ∫0
= cos ux du
(u2 + 2 + 2 u)(u2 + 2 − 2 u)

1 ∞ (2 u2 + 4)cos ux 2 ∞ (u2 + 2)cos ux


π ∫0 (u2 + 2)2 − (2 u)2 π ∫0
= du = du.
u4 + 4
Example 2: Using Fourier integral method, prove that
2 (b2 − a2 ) ∞ u sin ux du
e − ax − e − bx = ∫ ⋅
π 0 (u + a2 )(u2 + b2 )
2

(Gorakhpur 2008, 11, 13)


Solution: We know that the Fourier sine integral formula is
2 ∞  ∞ 
f ( x) = ∫ sin ux ∫ f (v)sin uv dv du. ...(1)
π 0  0 
Let f ( x) = e − ax − e − bx then f (v) = e − av − e − bv .
∴ From (1), we get
2 ∞  ∞ 
e − ax − e − bx = ∫ sin ux ∫ (e − av − e − bv ) cos uv dv du
π 0  0 
2 ∞  ∞ ∞ 
= ∫ sin ux ∫ e − av sin uv dv − ∫ e − bv sin uv dv du
π 0  0 0 
2 ∞  u u 
π ∫0
= sin ux  −  du
 a2 + u2 b2 + u2 
 ∞ − ax u 
∵ ∫0 e cos bx dx = 
 a + b2 
2

2 ∞ u (b2 + u2 − a2 − u2 )
π ∫0
= sin ux du
(a2 + u2 )(b2 + u2 )
2 ∞ u (b2 − a2 ) sin ux
π ∫0 (a2 + u2 )(b2 + u2 )
= du
M-115

2 (b2 − a2 ) ∞ u sin ux du
Hence, e − ax − e − bx = ∫ ⋅
π 0 (a + u2 )(b2 + u2 )
2

 0, when x ≤ 0 or x ≥ π
Example 3: Suppose a function f ( x) is given by f ( x) = 
 sin x, when 0 ≤ x ≤ π.
1 ∞ cos u (π − x) + cos ux
Using Fourier integral formula, show that f ( x) = ∫ du.
π 0 1 − u2
Solution: The Fourier integral formula is
1 ∞ ∞
f ( x) = ∫ [ f (v) ∫ cos u (v − x) dv] du, − ∞ < x < ∞. ...(1)
π −∞ 0

 0, when x ≤ 0 or x ≥ π
Given that f ( x) = 
sin x, when 0 ≤ x ≤ π.
 0, when v ≤ 0 or v ≥ π
∴ f (v) =  ...(2)
sin v, when 0 ≤ v ≤ π .
∞ 0
Now ∫− ∞ f (v)cos u (v − x) dv = ∫− ∞ f (v)cos u (v − x) dv

π ∞
+∫ f (v)cos u (v − x) dv + ∫π f (v)cos u (v − x) dv
0
π
= ∫0 sin v cos u (v − x) dv [From (2)]

1 π
2 ∫0
= [sin{(u + 1) v − ux } − sin { (u − 1) v − ux } ] dv

[∵ 2 cos A sin B = sin ( A + B) − sin ( A − B)]


π
1  − cos {(u + 1) v − ux} cos { u − 1} v − ux 
= +
2  u +1 u −1 
0
π
1 (u + 1)cos X − (u − 1)cos Y 
=   , where X = (u − 1) v − ux
2 (u − 1)(u + 1) 0 and Y = (u + 1) v − ux
π
 u (cos X − cos Y ) + (cos X + cos Y )
= 
 2 (u2 − 1) 0
X +Y Y − X  X +Y Y − X 
2 u sin   sin   + 2 cos   cos  
 2   2   2   2 
=
2 (u2 − 1)
1 π
= [u sin (v − x) u sin v + cos (v − x) u cos v]
2 0
(u − 1)
[ Putting the values of X and Y ]
1
= [cos (π − x) u cos π − cos (− xu)]
(u2 − 1)
1
= [− cos (π − x) u − cos xu]
(u2 − 1)
[∵ cos π = −1 and cos (− x) = cos x ]
M-116

1
= [cos (π − x) u + cos xu].
(1 − u2 )
∞ cos u (π − x) + cos ux
Therefore, ∫− ∞ f (v)cos u (v − x) dv =
1 − u2
⋅ ...(3)

Hence from equation (1), we get


1 ∞ cos u (π − x) + cos ux
f ( x) = ∫ du.
π 0 (1 − u2 )

Comprehensive Exercise 1

1. Using Fourier cosine integral formula, prove that


∞ cos λx π −x
∫0 λ2 + 1 dλ = 2 e , x ≥ 0. (Gorakhpur 2016)

2. Using Fourier cosine integral formula, show that


2 a ∞ cos ux
e − ax =
π ∫ 0 u2 + a2
dx, a > 0 , x ≥ 0 .

 1, when | x|≤ 1
3. Express the function f ( x) =  as a Fourier integral.
0 , when | x|> 1
∞ sin λ cos λ x
Hence evaluate ∫ dλ .
0 λ
 1, when 0 ≤ x ≤ π
4. Express f ( x) =  as a Fourier sine integral.
0 , when x > π
∞ 1 − cos πλ
Hence evaluate ∫ sin xλ dλ .
0 λ
5. Using Fourier sine integral formula, show that
6 ∞ u sin ux du
e x − e2 x = ∫ , x ≥ 0.
π 0 (u2 + 1)(u2 + 4)

6. Using Fourier integral formula, show that


1 ∞ u [sin ux − sin u (π − x)]
f ( x) = ∫ du
π 0 u2 − 1
0 , when x ≤ 0 and x ≥ π
where f ( x) is given by f ( x) = 
cos x, when 0 ≤ x ≤ π .

A nswers 1

 π / 2, | x|< 1  π / 2, 0 ≤ x ≤ π π
3.  4.  ;
 0, | x|> 1  0, x > π 4
M-117

4 Fourier Transform or Complex Fourier Transform


(Gorakhpur 2005, 07; Purvanchal 14)
Let f ( x) be a function defined on (− ∞, ∞) and be piecewise continuous in each finite
partial interval and absolutely integrable in (− ∞, ∞), then
1 ∞
F { f ( x)} =
√ (2π) ∫ − ∞
e ipx f ( x) dx

~
is called the Fourier Transform of f ( x) and is denoted by F { f ( x)} or f ( p).
~
The function f ( x) is called the inverse Fourier transform of f ( p) i.e.,
~
f ( x) = F −1 { f ( p)}.

5 Inversion Theorem for Complex Fourier Transform


~
If f ( p) is the Fourier transform of f ( x) and if f ( x) satisfies the Dirichlet's conditions in every

finite interval (− l, l ) and further if ∫ − ∞ | f ( x)| dx is convergent, then at every point of
continuity of f ( x),
1 ∞ ~
f ( x) = ∫ −∞ f ( p) e − ipx dp.
√ (2π) (Gorakhpur 2007)
Proof: From Fourier integral formula, we have
1 ∞  ∞ iw ( x − v ) 
f ( x) =
2π ∫ − ∞
f (v)  ∫
 −∞
e dw dv

1 ∞ ∞
f (v) e − iwv dv
2 π ∫ −∞
= e iwx dw ∫
−∞
1 ∞ 1 ∞
= ∫ −∞ e − ipx dp ∫ −∞ f ( x) e ipx dx,
√ (2 π) √ (2 π)
putting w = − p so that dw = − dp
1 ∞ ~
= ∫ −∞ e − ipx f ( p) dp.
√ (2 π)
Note: Some authors also define the Fourier transform in the following forms :
~ ∞
(1) f ( p) = ∫ e − ipx f ( x) dx
−∞

1 ∞ ~
and f ( x) =
2π ∫ −∞ f ( p) ⋅ e ipx dp.

~ ∞
(2) f ( p) = ∫ e ipx f ( x) dx
−∞

1 ∞ ~
and f ( x) = ∫ −∞ e − ipx f ( p) dp.

M-118

~ 1 ∞
(3) f ( p) = ∫ −∞ e − ipx f ( x) dx
√ (2π)

1 ∞ ~
and f ( x) =
√ (2π) ∫ −∞ f ( p) ⋅ e ipx dp.

6 Fourier Sine Transform


~
The infinite Fourier sine transform of f ( x), 0 < x < ∞, is denoted by Fs { f ( x)} or f s ( p) , and is
defined as
~  2 ∞
Fs { f ( x)} = f s ( p) =  
 π ∫0 f ( x) sin px dx .

~
The function f ( x) is called the inverse Fourier sine transform of f s ( p)
~
i. e., f ( x) = Fs −1 { f s ( p)}.

Note: Some authors also define


~ ∞
f s ( p) = ∫ f ( x) sin px dx.
0

7 Inversion Formula for Fourier Sine Transform


~
If f s ( p) is the Fourier sine transform of the function f ( x) which satisfies the Dirichlet's conditions

in every finite interval (0 , l ) and is such that ∫0 | f ( x)| dx exists, then

 2 ∞ ~
f ( x) =  
 π ∫0 f s ( p) sin px dp

at every point of continuity of f ( x).


This is an inversion formula for infinite Fourier sine transform.
Proof: From Fourier integral formula, we have
1 ∞ ∞
f ( x) =
π ∫0 dw ∫ −∞ f (v) cos w ( x − v) dv

1 ∞ ∞
= ∫
π 0
dp ∫
−∞
{ f (v) cos px cos pv + f (v) sin px sin pv} dv,
where w = p
1 ∞ ∞
=
π ∫0 cos px dp ∫ −∞ f (v) cos pv dv

1 ∞ ∞
+
π ∫0 sin px dp ∫ −∞ f (v) sin pv dv
M-119

1 ∞ ∞
or f ( x) =
π ∫0 cos px dp ∫ −∞ f ( x) cos px dx

1 ∞ ∞
+
π ∫0 sin px dp ∫ −∞ f ( x) sin px dx …(1)

Now define f ( x) in (− ∞, 0 ) such that f ( x) is an odd function of x in (− ∞, ∞). Then


obviously f ( x) cos px is an odd function of x and f ( x) sin px is an even function of x in
(− ∞, ∞).
∴ From (1), we have
 2 ∞  2 ∞
f ( x) =  
 π ∫0 sin px dp ×  
 π ∫0 f ( x) sin px dx

 2 ∞ ~
or f ( x) =  
 π ∫0 f s ( p) sin px dp .

Note: According to the authors who define


~ ∞
f s ( p) = ∫0 f ( x) sin px dx,

2 ∞ ~
we have f ( x) =
π ∫0 f s ( p) sin px dp.

8 Fourier Cosine Transform


The infinite Fourier cosine transform of f ( x), 0 < x < ∞, is denoted by Fc { f ( x)} or
~
f c ( p), and is defined as
~  2 ∞
Fc { f ( x)} = f c ( p) =  
 π ∫0 f ( x) cos px dx.

~
The function f ( x) is called the inverse Fourier Cosine transform of f c ( p)
~
i.e., f ( x) = Fc −1 { f c ( p)}
~ ∞
Note: Some authors also define f c ( p) =
0 ∫ f ( x) cos px dx .

9 Inversion Formula for Fourier Cosine Transform


~
If f c ( p) is the Fourier cosine transform of the function f ( x) which satisfies the Dirichlet's

conditions in every finite interval (0 , l ) and is such that ∫0 | f ( x)| dx exists, then

 2 ∞ ~
f ( x) =  
 π ∫0 f c ( p) cos px dp

at every point of continuity of f ( x).


M-120

This is an inversion formula for infinite Fourier cosine transform.


Proof: Proceeding as in article 7, we have
1 ∞ ∞
f ( x) =
π ∫0 cos px dp ∫ −∞ f ( x) cos px dx

1 ∞ ∞
+
π ∫0 sin px dp ∫ −∞ f ( x) sin px dx. …(1)

Now define f ( x) in (−∞, 0 ) such that f ( x) is an even function of x in (− ∞, ∞). Then


obviously f ( x) cos px is an even function of x and f ( x) sin px is an odd function of x in
(− ∞, ∞).
∴ from (1), we have
 2 ∞  2 ∞
f ( x) =  
 π ∫0 cos px dp ×  
 π ∫0 f ( x) cos px dx

 2 ∞ ~
or f ( x) =  
 π ∫0 f c ( p) cos px dp.

Note: According to the authors who define


~ ∞
f c ( p) = ∫ f ( x) cos px dx,
0

2 ∞ ~
we have f ( x) =
π ∫0 f c ( p) cos px dx.

10 Linearity Property of Fourier Transform


~ ~
If f ( p) and g ( p) are Fourier transforms of f ( x) and g ( x) respectively, then
~ ~
F { a f ( x) + bg ( x)} = a f ( p) + b g ( p),
where a and b are constants. (Gorakhpur 2014)

Proof: We have
~ 1 ∞
F { f ( x)} = f ( p) =
√ (2π) ∫−∞ e ipx f ( x) dx

~ 1 ∞
and F { g ( x)} = g ( p) =
√ (2π) ∫−∞ e ipx g( x) dx.

1 ∞
∴ F { a f ( x) + b g ( x)} =
√ (2 π) ∫−∞ e ipx { af ( x) + bg ( x)} dx

a ∞ b ∞
=
√ (2 π) ∫−∞ e ipx f ( x) dx +
√ (2 π) ∫−∞ e ipx g ( x) dx

~ ~
= a f ( p) + b g ( p).
M-121

11 Change of Scale Property


~
Theorem 1: (For Complex Fourier Transform). If f ( p) is the complex Fourier
1 ~  p
transform of f ( x), the complex Fourier transform of f (ax) is f   ⋅
a  a
(Gorakhpur 2008, 10)
Proof: We have
~ 1 ∞
f ( p) = F { f ( x)} =
√ (2π) ∫−∞ e ipx f ( x) dx. …(1)

1 ∞
Now F { f (ax)} =
√ (2π) ∫ − ∞
e ipx f (ax) dx

1 1 ∞
= ∫ e ip( t / a ) f (t) dt,
a √ (2 π) − ∞
1
putting ax = t so that dx = dt
a
1 1 ∞ i( p / a ) t
a √ (2 π) ∫ − ∞
= ⋅ e f (t) dt

1 ~  p
= f   , from (1).
a  a
~
Theorem 2: (For Fourier Sine Transform). If f s ( p) is the Fourier sine transform of f ( x),
1 ~  p
then the Fourier sine transform of f (ax) is f s   ⋅
a  a
Proof: We have
~
f s ( p) = Fs { f ( x)}
 2 ∞
=  
 π ∫0 f ( x) sin px dx …(1)

 2 ∞
Now Fs { f (ax)} =  
 π ∫0 f (ax) sin px dx

1  2 ∞ p 
a  π  ∫0
= ⋅   f (t) . sin  t dt,
a 

putting ax = t so that dx = (1 / a) dt
1 ~  p
= f s   , from (1).
a  a
~
Theorem 3: (For Fourier Cosine Transform). If f c ( p) is the Fourier Cosine
1 ~  p
Transform of f ( x) , then the Fourier Cosine transform of f (ax) is f c   .
a  a
Proof: We have
~
f c ( p) = Fc { f ( x)}

 2 ∞
=  
 π ∫0 f ( x) cos px dx …(1)
M-122

 2 ∞
Now Fc { f (ax)} =  
 π ∫0 f (ax) cos px dx

1  2 ∞ p 
= ⋅  
a  π ∫0 f (t) cos  t dt ,
a 
putting ax = t so that dx = (1 / a) dt
1 ~  p
= f c   , from (1).
a  a

12 Shifting Property
~
If f ( p) is the complex Fourier transform of f ( x), then the complex Fourier transform of f ( x − a)
~
is e ipa f ( p). (Gorakhpur 2006, 08; Purvanchal 14)
Proof: We have
~ 1 ∞
f ( p) = F { f ( x)} =
√ (2π) ∫−∞ e ipx f ( x) dx …(1)

1 ∞
Now F { f ( x − a)} =
√ (2π) ∫ −∞ e ipx f ( x − a) dx

1 ∞
= ∫ e ip( a + t ) f (t) dt,
√ (2 π) − ∞
putting x − a = t so that dx = dt
1 ∞
= e ipa ⋅
√ (2 π) ∫ − ∞
e ipt f (t) dt

~
= e ipa f ( p) , from (1).

13 Modulation Theorem
~
If f ( p) is the Complex Fourier transform of f ( x), then the Fourier transform of f ( x) cos ax is
1 ~ ~
[ f ( p − a) + f ( p + a)].
2 (Gorakhpur 2007, 08, 12)
Proof: We have
~ 1 ∞
f ( p) = F { f ( x)} =
√ (2π) ∫ −∞ e ipx f ( x) dx …(1)

Now F { f ( x) cos ax}


1 ∞ e iax + e − iax
=
√ (2 π) ∫ −∞ e ipx . f ( x) ⋅
2
dx

1 1 ∞ 1 ∞ 
=  ∫ −∞ e i( p + a ) x f ( x) dx + ∫ e i ( p − a ) x f ( x) dx
2  √ (2 π) √ (2 π) − ∞

1 ~ ~
= [ f ( p + a) + f ( p − a)].
2
M-123

14 Important Results
~ ~
Theorem: If f s ( p) and f c ( p) are Fourier sine and cosine transforms of f ( x) respectively, then
1 ~ ~
(i) Fs { f ( x) cos ax} = [ f s ( p + a) + f s ( p − a)]
2
1 ~ ~
(ii) Fc { f ( x) sin ax} = [ f s ( p + a) − f s ( p − a)]
2
1 ~ ~
(iii) Fs { f ( x) sin ax} = [ f c ( p − a) − f c ( p + a)].
2
Proof: (i) We have
 2 ∞
Fs { f ( x) cos ax} =  
 π ∫0 f ( x) cos ax sin px dx

 2 1 ∞
=   ⋅
 π 2 ∫0 f ( x) ⋅ [sin ( p + a) x + sin ( p − a) x] dx

1   2 ∞
2   π ∫ 0
=    f ( x) ⋅ sin ( p + a) x dx

 2 ∞ 
+  
 π ∫0 f ( x) ⋅ sin ( p − a) x dx

1 ~ ~
= [ f s ( p + a) + f s ( p − a)].
2
(ii) We have
 2 ∞
Fc { f ( x) sin ax} =  
 π ∫0 f ( x) sin ax cos px dx

 2 1 ∞
=   ⋅ ∫ f ( x) [sin ( p + a) x − sin ( p − a) x] dx
 π 2 0

1   2 ∞
=   
2   π ∫0 f ( x) sin ( p + a) x dx

 2 ∞ 
−  
 π ∫0 f ( x) ⋅ sin ( p − a) x dx

1 ~ ~
= [ f s ( p + a) − f s ( p − a)].
2
(iii) We have
 2 ∞
Fs f { ( x) sin ax} =   ∫ f ( x) sin ax sin px dx
 π 0

 2 1 ∞
=   ⋅ ∫ f ( x) [cos ( p − a) x − cos ( p + a) x] dx
 π 2 0
M-124

1   2 ∞
2   π  ∫0
=    f ( x) cos ( p − a) x dx

 2 ∞ 
−   ∫ f ( x) cos ( p + a) x dx
 π 0 
1 ~ ~
= [ f c ( p − a) − f c ( p + a)].
2

15 Theorem
If φ ( p) is the Fourier sine transform of f ( x) for p > 0, then
Fs { f ( x)} = − φ (− p) for p < 0 .
Proof: We have
 2 ∞
Fs { f ( x)} =   ∫ F ( x) sin px dx
 π 0
= φ ( p), for p > 0. …(1)
For p < 0, let p = − s where s > 0.
 2 ∞
∴ Fs { f ( x)} =  
 π ∫0 f ( x) sin (− sx) dx

 2 ∞
=−  
 π ∫0 f ( x) sin sx dx = − φ (s)

= − φ (− p ), for p < 0.
Hence in general
 φ (| p |), p > 0
Fs { f ( x)} = 
 − φ (| p |), p < 0
or Fs { f ( x)} = φ (| p |). Sgn p,
 + 1, p > 0
where the symbol Sgn p = 
 − 1, p < 0 .

16 Multiple Fourier Transforms


Let f ( x, y) be a function of two variables x and y. Regarding f ( x, y), temporarily, as a
function of x, its Fourier transform is
~ 1 ∞
f ( p, y) = ∫
√ (2π) − ∞
f ( x, y) e ipx dx .

~
Now regarding f ( p, y) as a function of y, its Fourier transform is
~ 1 ∞ ~
F ( p, q) =
√ (2π) ∫ −∞ f ( p, y) e ipy dy
M-125

~ 1 ∞ ∞
f ( x, y) e i ( px + qy )dx dy
2 π ∫ −∞ ∫ −∞
or F ( p, q) =

which is Fourier transform of f ( x, y).


Inversion formula: Using inversion formula for Fourier transforms, we have
1 ∞ ~
f ( x, y) = ∫ f ( p, y) e − ipx dp
√ (2π) − ∞
~ 1 ∞ ~
and f ( p, y) = ∫ −∞ F ( p, q) e − iqy dq.
√ (2π)
1 ∞ ~
Hence f ( x, y) = ∫ −∞ F ( p, q) e − i( px + qy )dp dq,

which is the inversion formula for the Fourier transform of f ( x, y).

17 Convolution
The function
1 ∞
H ( x) = F * G =
√ (2π) ∫ −∞ F (u). G ( x − u) du

is called the convolution or Falting of two integrable functions F and G over the
interval (− ∞, ∞).
Note: Some authors also define

F*G= ∫ −∞ F (u). G ( x − u) du.

18 The Convolution or Falting Theorem for Fourier


Transforms
If F { f ( x)} and F { g ( x)} are the Fourier transforms of the functions f ( x) and g ( x) respectively,
then the Fourier transform of the convolution of f ( x) and g ( x) is the product of their Fourier
transforms
i.e. F { f ( x) * g ( x)} = F { f ( x)} . F { g ( x)}.
Proof: We have F { f ( x) * g ( x)}
 1 ∞ 
= F
 √ (2 π)
∫ −∞ f (u) ⋅ g ( x − u) du

1 ∞  1 ∞ 
= ∫  ∫
√ (2 π) − ∞  √ (2 π) − ∞
f (u) g ( x − u) du e ipx dx

1 ∞  ∞ 
2π ∫ − ∞
= f (u)  ∫ g ( x − u) e ipx dx du
 − ∞ 
M-126

1 ∞  ∞ 
g ( y) e ip ( u + y )dy du,
2π ∫ − ∞
= f (u)  ∫
 −∞ 
putting x − u = y so that dx = dy,

1 ∞  ∞ ipu 
=
2π ∫ −∞ f (u)  ∫
 − ∞
e g ( y) e ipy dy du

1 ∞  ∞ 
=
2π ∫ −∞ f (u)  e ipu ∫
 − ∞
g ( y) e ipy dy du

1 ∞  1 ∞ 
=
√ (2 π) ∫ − ∞
f (u) e ipu
 √ (2 π) ∫ − ∞
g ( x) e ipx dx du

1 ∞
= ∫
√ (2 π) ∞−
f (u) [e ipu F { g ( x)}] du

 1 ∞ 
= ∫
 √ (2 π) − ∞
f (u) e ipu du F { g ( x)}

 1 ∞ 
= ∫
 √ (2 π) − ∞
f ( x) e ipx dx F { g( x)}

= F { f ( x)} ⋅ F { g ( x)} .

19 Relationship Between Fourier and Laplace


Transforms
(Gorakhpur 2011)
Let us consider the function
 e − x t g(t), t > 0
f (t) =  …(1)
0 , t < 0.

∴ The Fourier transform of f (t) is given by



F { f (t)} = ∫ −∞ e ipt . f (t) dt, (Taking non-symmetrical form of F. T.)

0 ∞
=∫ 0 ⋅ e ipt dt + ∫0 e − xt g(t) ⋅ e ipt dt
−∞

= ∫0 e( ip− x ) t g (t) dt


= ∫0 e − st g (t) dt , putting x − ip = s

= L { g (t)}.
Hence the Fourier transform of the function f (t) defined by (1) is the Laplace transform of
the function g (t).
M-127

20 Fourier Transform of The Derivatives of a Function


~ ~
(a) The Fourier transform of f ′ ( x) , the derivative of f ( x) is − ip f ( p), where f ( p) is the
Fourier transform of f ( x). (Gorakhpur 2015)
Proof: By definition
1 ∞
F { f ′ ( x)} =
√ (2π) ∫ −∞ f ′ ( x) ⋅ e ipx dx

1 ∞ f ( x + h) − f ( x) ipx
= ∫ lim
√ (2 π) − ∞ h→ 0 h
⋅ e dx

1 ∞ f ( x + h) ipx 1 ∞ f ( x) ipx
= lim ∫
h→ 0 √ (2 π) − ∞ h
⋅ e dx − lim
h→ 0 √ (2 π)
∫ −∞ h
⋅ e dx

~
1 ∞ f ( x + h) ip( x + h ) − iph f ( p)
= lim ∫
h→ 0 √ (2π) − ∞ h
⋅e e d ( x + h) − lim
h→ 0 h
~
e − iph ∞ 1 f ( p)
= lim ∫
h→ 0 √ (2 π) − ∞ h
⋅ f ( y) e ipy dy − lim
h→ 0 h

~ ~
e − iph f ( p) f ( p)
= lim − lim
h→ 0 h h→ 0 h
~ e − iph − 1 ~
= f ( p) ⋅ lim = (− ip) f ( p).
h→ 0 h

(b) The Fourier transform of f n( x), the nth derivative of f ( x) is (− ip)n times the Fourier
transform of f ( x) provided that the first (n − 1) derivatives of f ( x) vanish as x → ± ∞ .
Proof: By definition
1 ∞
F { f n ( x)} = ∫ −∞ f n ( x) . e ipx dx.
√ (2π)
Integrating by parts, we have
1 1 ∞
F { f n ( x)} = [ f n−1( x) ⋅ e ipx ] ∞
−∞ − ∫ f n−1( x) ipe ipx dx
√ (2 π) √ (2 π) − ∞
(− ip) ∞
= ∫ −∞ f n−1 ( x) e ipx dx, since lim f n−1 ( x) = 0 .
√ (2 π) x →± ∞

Repeating the same process of integration by parts (n − 1) times more, we have


1 ∞
F { f n ( x)} = (− ip)n ∫ f ( x) e ipx dx
√ (2π) − ∞
~ ~
or f n ( p) = (− ip)n f ( p).
M-128

(c) The Fourier cosine and sine transforms of the derivatives of f ( x) are given by
~ n −1 ~
 2
f c2 n ( p) = −  
 π ∑
(− 1)r α2 n−2 r −1 p2 r + (−1)n p2 n f c ( p);
r=0
~ n ~
 2
f c2 n+1 ( p) = −   ∑ (− 1)r α2 n−2 r p2 r + (−1)n p2 n+1 f s ( p);
 π
r =1
~ n ~
 2
f s2 n ( p) = −  
 π ∑ (− 1)r α2 n−2 r p2 r −1 + (−1)n+1 p2 n f s ( p);
r =1
~ n ~
 2
and f c2 n+1 ( p) = −   ∑ (− 1)r α2 n−2 r −1 p2 r −1 + (−1)n+1 p2 n+1 f c ( p);
 π
r =1

provided that first (n − 1) derivatives of f ( x) vanish as x → ∞ and


d n−1 f
→ α n−1 etc . as x → 0 .
dx n−1
Proof: By definition, we have
~ ∞
 2
f cn ( p) =   ∫0 f n ( x) cos px dx …(1)
 π
~ ∞
 2
and f cn ( p) =   ∫0 f n ( x) sin px dx . …(2)
 π
Integrating R.H.S. of (1) by parts, we have
~ ∞
 2 n−1  2
f cn ( p) =  [f ( x)cos px]0∞ + p   ∫0 f n−1( x)sin px dx
 π  π
~ ~
 2
or f cn ( p) = −   α n−1 + p f sn−1( p). …(3)
 π
Similarly integrating R.H.S. of (2), we have
~ ~
f sn ( p) = − p f cn−1( p). …(4)
From (3) and (4), we have
~ ~
 2
f cn ( p) = −   α n−1 − p2 f nn−2 ( p). …(5)
 π
~ ~
By repeated application of these rules f c n ( p) is obtained as a sum of α’s and f c ′ ( p)
~
or f c ( p).
~
It is clear that f c ′ ( p) will occur when n is odd and in that case it may be replaced
by
 2 ~
−   α0 + p f s ( p).
 π
M-129

Thus, we have
~ n −1 ~
 2

2n
fc ( p) = −   (− 1)r α2 n−2 r −1 p2 r + (−1)n p2 n f c ( p)
 π
r=0
~ n
2n + 1  2 ~
and fc ( p) = −  
 π ∑ (− 1)r α2 n−2 r p2 r + (−1)n p2 n+1 f s ( p)
r=0

Also from (3) and (4), we have


~  ~ 
 2
f sn( p) = − p −   α n−2 + p f sn−2 ( p)
  π 
 
~ ~
 2
or f sn( p) = p   α n−2 − p2 f sn−2 ( p).
 π
~ ~
By repeated application of these results f s n ( p) is obtained as a sum of α’s and f s ′ ( p) or
~
f s ( p).
~ ~
It is clear that f s ′ ( p) will occur when n is odd and it may be replaced by − p f c ( p).

Thus, we have
~ n ~
 2
f s2 n( p) = −  
 π ∑ (− 1)r α2 n−2 r p2 r −1 + (−1)n+1 p2 n f s ( p)
r =1
~ n
 2
and f c2 n + 1( p) = −  
 π ∑ (− 1)r α2 n−2 r +1 p2 r −1
r =1
~
+ (− 1)n+1 p2 n+1 f c ( p).
Note: The infinite sine and cosine transforms can be applied when the range of the
variable selected for exclusion is 0 to ∞.

Example 4: Find the Fourier complex transform of f ( x), if


iωx
 e a< x< b
f ( x) = 
0 x < a, x > b. (Meerut 2013B; Avadh 14)

Solution: We have
1 ∞
F { f ( x)} =
√ (2π) ∫ −∞ e ipx f ( x) dx

1  a b ipx ∞ 
⋅ e iωx dx + ∫
√ (2 π)  ∫ − ∞
= 0 ⋅ e ipx dx + ∫a e 0 ⋅ e ipx dx
b 
M-130

1 b i ( p + ω)x
= ∫
√ (2 π) a
e dx

b
1  e i( p + ω ) x 
= ⋅ 
√ (2 π)  i ( p + ω) 
a

1  ei ( p + ω ) a − ei ( p + ω ) b 
=  ⋅
√ (2 π)  p+ω 

Example 5: Find the Fourier transform of F ( x) defined by

 1, | x | < a
F ( x) = 
 0 ,| x | > a.
(Gorakhpur 2005, 07; Kanpur 08; Meerut 13; Bundelkhand 13; Avadh 13)
and hence evaluate
∞ sin pa cos px
(a) ∫ dp,
−∞ p (Gorakhpur 2009, 11)
∞ sin p
(b) ∫0 p
dp.
(Gorakhpur 2008)

Solution: We have
~ 1 ∞
F ( p) =
√ 2π) ∫ −∞ e ipx F ( x) dx

a
1 a 1  e ipx 
∫ −a e
ipx
= dx =  
√ (2 π) √ (2 π)  ip 
−a

 e ipa − ipa 
1 e
=  − 

√ (2 π)  ip ip 
2 i sin pa 2 sin pa
= = , p ≠ 0.
ip √ (2 π) p √ (2 π)
~
For p = 0 , F ( p) = 2 a / √ (2 π).

(a) We know that if


~ 1 ∞
∫ − ∞ F ( x) e
ipx
F ( p) = dx
√ (2π)

1 ∞ ~
then F ( x) = ∫ −∞ F ( p) e − ipx dp .
√ (2π)

1 ∞ 2 sin pa  1, | x | < a
∴ ∫ −∞ ⋅ e − ipx dp = 
√ (2 π) p √ (2 π) 0 , | x | > a.
M-131

1 ∞ sin pa cos px i ∞ sin pa sin px


But L.H.S. =
π ∫ −∞ p
dp −
π ∫ −∞ p
dp

1 ∞ sin pa cos px
=
π ∫ −∞ p
dp,

since the integrand in the other integral is an odd function of p.

∞ sin pa cos px  π, | x | < a


∴ ∫ −∞ p
dp = 
0 , | x | > a.
(b) If x = 0 and a = 1 in (a), then
∞ sin p
∫ − ∞ p dp = π
∞ sin p ∞ sin p π
or 2 ∫0 p
dp = π or ∫0 p
dp =
2
⋅ …(1)

Note: Putting x = 0 in (a) and on simplification, we get


∞ sin ap π
∫0 p
dp =
2

…(2)
The results (1) and (2) can be used as standard formulae.

Example 6: Find Fourier sine and cosine transforms of e − x and using the inversion formulae
recover the original functions, in both the cases. (Gorakhpur 2006, 09, 13; Kanpur 09)
Solution: Let f ( x) = e − x .
~  2 ∞  2 ∞ −x
Then f s ( p) =  
 π ∫0 f ( x) sin px dx =  
 π ∫0 e sin px dx


 2  e
−x 
=    (− sin px − p cos px)
 π  1 + p2 0

p  2
=  
1 + p2  π
~  2 ∞  2 ∞
and f c ( p) =   ∫0 f ( x) cos px dx =   ∫0 e − x cos px dx
 π  π

 2  e
−x 
=    (− cos px + p sin px)
 π  1 + p 2
0

1  2
= ⋅
2  π 
1+ p
Applying inversion to the sine transform, we have
 2 ∞ ~
f ( x) =   ∫ f s ( p) ⋅ sin px dp
 π 0
2 ∞ p sin px
=
π ∫0 1 + p2
dp …(1)
M-132

and applying inversion to the cosine transform, we have


 2 ∞ ~
f ( x) =  
 π ∫0 f c ( p) cos px dp

2 ∞ cos px
=
π ∫0 1 + p2
dp. …(2)

Now from Fourier integral theorem, we have


1 ∞ ∞
π ∫0
f ( x) = dp ∫ f (v) cos p ( x − v) dv
−∞
1 ∞ ∞
or f ( x) = ∫
π 0
cos px dp ∫
−∞
f (v) cos pv dv

1 ∞ ∞
+
π ∫ 0
sin px dp ∫
− ∞
f (v) sin pv dv. …(3)

Case I: Defining f ( x) in (− ∞, 0 ) such that f ( x) is an even function of x, from (3), we


have
2 ∞ ∞
f ( x) =
π ∫0 cos px dp ∫0 f (v) cos pv dv.

Taking f ( x) = e − x , we have
2 ∞ ∞
e− x = ∫0 cos px dp ∫0 e − v cos pv dv
π

2 ∞  e− v 
= ∫ cos px  (− cos pv + p sin pv) dp
π 0 1 + p
2
0

2 ∞ cos px
π ∫ 0 1 + p2
= dp.

∞ cos px π −x
∴ ∫0 1+ p 2
dp =
2
e .

2 π −x
∴ from (2) we have f ( x) = ⋅ e = e− x .
π 2
Case II: Again defining f ( x) in (− ∞, 0 ) such that f ( x) is an odd function of x, from
(2), we have
2 ∞ ∞
f ( x) =
π ∫0 sin px dp ∫0 f (v) sin pv dv.

Taking f ( x) = e − x and simplifying, we have


∞ p sin px π −x
∫0 1+ p 2
dp =
2
e .
(Gorakhpur 2013)
2 π
∴ from (1), f ( x) = ⋅ e − x = e − x .
π 2
M-133

1
Example 7: Find Fourier cosine transform of f ( x) = and hence find Fourier sine
1 + x2
x
transform of F ( x) = ⋅
1 + x2 (Kanpur 2007, 10; Meerut 13; Rohilkhand 14)

Solution: We have
~  2 ∞
f c ( p) =  
 π ∫0 f ( x) cos px dx

 2 ∞ cos px
=  
 π ∫0 2
1+ x
dx .

Differentiating both sides w.r.t. p, we have


d ~  2 ∞ x sin px
dp
f c ( p) = −  
 π ∫0 2
1+ x
dx

 2 ∞ ( x2 + 1 − 1) sin px
=−  
 π ∫ 0 2
x (1 + x )
dx

 2 ∞ sin px  2 ∞ sin px
=−  
 π ∫0 x
dx +  
 π ∫0 x (1 + x2 )
dx

 2 π  2 ∞ sin px  ∞ sin px π
=−   ⋅ +  
 π 2  π ∫0 2
x (1 + x )
dx . ∵ ∫0 x
dx =
2 

Differentiating again w.r.t. p, we have


d2 ~  2 ∞ cos px ~

dp2
f c ( p) =  
 π ∫0 2
1+ x
dx = f c ( p)

~
or ( D2 − 1) f c ( p) = 0
whose general solution is
~
f c ( p) = Ae p + Be − p . …(1)
Now when p = 0 ,

~  2 ∞ dx  2  −1 
f c ( p) =  
 π ∫ 0 1 + x2 =  π   tan x 
0
π  2  π
=   =  
2  π  2

d ~  π
and f c ( p) = −   .
dp  2

 π
∴ from (1), we have   = A + B
 2

 π
and −   = A − B.
 2
M-134

 π
Solving, A = 0 , B =   .
 2
~  π
∴ from (1), we have f c ( p) =   e − p.
 2

Second part: We have


~  2 ∞ cos px  π − p
f c ( p) =  
 π ∫0 2
dx =
1+ x
  ⋅e .
 2
Now differentiating both sides w.r.t. p, we have
∞ x sin px π
− ∫ dx = − ⋅ e − p .
0 2
1+ x 2
~  2 ∞ x  π
∴ Fs ( p) =   ∫0 sin px dx =   e − p .
 π 1+ x 2  2

Example 8: Find the sine and cosine transforms of x ne − ax .


Solution: Let f ( x) = x ne − ax .
~  2 ∞
∴ f s ( p) =  
 π ∫0 f ( x) sin px dx

 2 ∞ n − ax
=  
 π ∫0 x e sin px dx …(1)


∞ − ax
 e − ax 
We have ∫0 e sin px dx = 
2
 a + p
2
(− a sin px − p cos px)
0

p 1 1 1 
= =  − ⋅
a +p2 2 2 i  a − ip a + ip

Differentiating both sides w.r.t. a, n times, we have



(−1)n ∫0 x n e − ax sin px dx

1  dn −1 dn −1
=  n (a − ip) − n (a + ip) 
2 i  da da 
1
= (− 1)n (n !) [(a − ip)−( n+1) − (a + ip)−( n+1)]
2i
1
= (− 1)n (n !) [2 i r −( n+1) sin (n + 1) θ], putting a = r cos θ, p = r sin θ
2i
= (− 1)n n !(1 / r)n+1 sin (n + 1) θ.

∴ ∫0 x n e − ax sin px dx

= (n !) . [1 / (a2 + p2 )( n+1)/2 ] sin {(n + 1) tan−1 ( p / a)}.


[ ∵ r = (a2 + p2 )1 /2 and θ = tan−1 ( p / a)]
M-135

Hence, from (1),


−1
~  2  n !sin {(n + 1) tan ( p / a)}
f s ( p) =   ⋅
 π (a2 + p2 )( n + 1) /2
~  2 ∞
Also f c ( p) =  
 π ∫0 f ( x) cos px dx

 2 ∞ n − ax
=  
 π ∫0 x e cos px dx . …(2)


∞ − ax
 e − ax 
We have, ∫0 e cos px dx = 
2
 a + p
2
(− a cos px + p sin px)
0
a
=
a + p2
2

1 1 1 
=  + ⋅
2  a − ip a + ip

Differentiating both sides w.r.t. a, n times, we have



(− 1)n ∫ x n e − ax cos px dx
0

1
= (− 1)n (n !) [(a − ip)−( n+1) + (a + ip)−( n+1)]
2
= (− 1)n (n !) (1 / r)n+1 cos (n + 1) θ,

putting a = r cos θ, p = r sin θ and on simplification.

∞ cos { (n + 1) tan−1 ( p / a)}


∴ ∫0 x n e − ax cos px dx = (n !) . ⋅
(a2 + p2 )( n + 1)/2

Hence from (2), we have


−1
~  2  n !cos {(n + 1) tan ( p / a)}
f c ( p) =   ⋅ .
 π (a2 + p2 )( n + 1) /2

Alternative method to evaluate the integrals


∞ − ax ∞
∫0 e x n sin px dx and ∫0 e − ax x n cos px dx.

∞ ∞
We have ∫0 e − ax x n cos px dx + i ∫0 e − ax x n sin px dx


=∫ e − ax x n (cos px + i sin px) dx
0
∞ ∞
= ∫0 e − ax x n e ipx dx = ∫0 e −( a − ip ) x x ( n + 1) −1 dx

Γ (n + 1)  ∞ Γ (n)
= ∵ ∫0 e − az z n − 1 dz = 
n+1  an 
(a − ip )
M-136

Γ (n + 1)
= , putting a = r cos θ and p = r sin θ
n+1
r (cos θ − i sin θ) n + 1

n!  1 
= (cos θ + i sin θ) n + 1 ∵ = cos θ + i sin θ
n +1
r  cos θ − i sin θ 
n!
= [cos (n + 1) θ + i sin (n + 1) θ]. ...(1)
r n +1
Equating real and imaginary parts on both sides of (1), we have
∞ n!
∫0 e − ax x n cos px dx = cos (n + 1) θ
r n +1
∞ n!
and ∫0 e − ax x n sin px dx = sin (n + 1) θ ,
r n +1
where r2 = a2 + p2 i. e., r = (a2 + p2 )1 /2 and θ = tan−1 ( p / a).
∞ n!
Hence ∫0 e − ax x n cos px dx = cos { (n + 1) tan−1 ( p / a)}
2 2 ( n+1)/2
(a + p )
∞ n!
and ∫0 e − ax x n sin px dx = sin {(n + 1) tan−1 ( p / a)}.
2 2 ( n+1)/2
(a + p )

Example 9: Find the sine transform of


e ax + e − ax

e πx − e − πx (Meerut 2013B; Purvanchal 14; Kanpur 14)
ax − ax
e +e
Solution: If f ( x) = πx , then we have
e − e − πx
~  2 ∞
f s ( p) =  
 π ∫0 f ( x) sin px dx

 2 ∞ e ax + e − ax
=  
 π ∫ 0 πx
e − πx
−e
sin px dx

 2 ∞ e ax + e − ax e ipx − e − ipx
=  
 π ∫0 e πx − e − πx

2i
dx

( a + ip) x
 2  1 ∞ e − e −( a + ip) x 1 ∞ e( a − ip) x − e −( a − ip) x 
=    ∫
2i ∫ 0
dx − dx
 π  2 i 0
 e π x − e − πx e π x − e− π x 

 2 1 1 a + ip 1 1 a − ip
=   2 i ⋅ 2 tan 2 − 2 i ⋅ 2 tan 2 
 π

 ∞ e az − e − az 1 a
∵ From definite integrals, ∫ dz = tan 
 0 e π z − e− π z 2 2 
M-137

 a + ip a − ip 
 2  1 sin 2 1
sin
2 
=    −
 π a + ip 4 i a − ip 
4 i cos cos 
 2 2 
a + ip a − ip a − ip a + ip
sin cos − sin cos
 2 2 2 2 2
=  
 π a + ip a − ip
4 i cos cos
2 2
 2  sin a + sin ip − (sin a − sin ip)
=  
 π 2 ⋅ 2 i [cos ip + cos a]
sin ip
=
√ (2π) i [cos ip + cos a]
sinh p
=
√ (2π) (cosh p + cos a)
e p − e− p
= ⋅
√ (2 π) (e p + e − p + 2 cos a)

Example 10: Find the Fourier sine transform of


1
f ( x) = ⋅
x (a + x2 )
2

Solution: We have
~  2 ∞ 1
f s ( p) =  
 π ∫0 x (a2 + x2 )
sin px dx . …(1)

∞ 1
Let I = ∫0 x (a2 + x2 )
sin px dx. …(2)

dI d ∞ sin px
Then =
dp dp ∫0 x (a2 + x2 )
dx

∞∂ 
 sin px  
=∫    dx
0  ∂p  x (a2 + x2 ) 
   
∞ cos px
= ∫0 a2 + x2
dx. …(3)

d2 I ∞ x sin px

dp 2
=− ∫0 a2 + x2
dx

∞ x2 sin px
=− ∫0 x (a2 + x2 )
dx

∞ ( x2 + a2 ) − a2
=− ∫0 x (a2 + x2 )
sin px dx
M-138

∞ sin px ∞ sin px
=− ∫0 dx + a2 ∫0 dx
x x (a2 + x2 )
π  ∞ sin px π
=− + a2 I. ∵ ∫0 dx =
2 x 2 

d2 I π
∴ − a2 I = −
2 2
dp
π d
or ( D2 − a2 ) I = − , where D ≡ ⋅
2 dp
The solution of the above differential equation is
π
I = Ae − ap + Be ap + ⋅ …(4)
2 a2
dI
∴ = − Aae − ap + Bae ap. …(5)
dp

Now from (2), when p = 0, we have I = 0 and from (3), when p = 0, we have

dI ∞ 1 1  −1 x∞ π
dp
= ∫0 a2 + x2
dx =
a tan  =
a 0 2 a

So putting p = 0 in (4) and (5), we get


π
A+ B= −
2 a2 …(6)
π π
and a (− A + B) = i. e., − A + B = ⋅ …(7)
2a 2 a2
Solving (6) and (7), we get
π
B = 0, A = − ⋅
2 a2
Putting the values of A and B in (4), we get

∞ sin px
I = ∫0 x (a2 + x2 )
dx

π π π
=− e − ap + = (1 − e − ap).
2 2
2a 2a 2 a2

Now putting the value of I in (1), we get


~  2 π − ap
f s ( p) =   ⋅ 2 (1 − e )
 π  2a

1  π − ap
=   ⋅ (1 − e ).
a 2  2
M-139

2
Example 11: Find the Fourier cosine transform of e − x .

Solution: We have
~ 2  2 ∞ 2
Fc { e − x } =   ∫0 e − x cos px dx = I …(1)
 π

Differentiating w.r.t.‘p’ we have


dI  2 ∞ 2
=−   ∫0 xe − x sin px dx
dp  π

1  2 ∞ 2
=   ∫0 (− 2 xe − x ) ⋅ sin px dx
2  π

1  2   − x2 ∞ − x2 
=   (e sin px)0∞ − p ∫ e cos px dx
2  
π  0 
(Integrating by parts taking sin px as first function)
p
=− I.
2
dI p
∴ =− dp.
I 2
Integrating, we have
p2
log I = − + log A
4
2
or I = Ae − p /4 . …(2)
But when p = 0, from (1)
 2 ∞ − x2 1
I =  
 π ∫0 e dx =
√2

∴ from (2), A = 1 / √ 2.
2 2
Hence, I = Fc { e − x } = (1 / √ 2) e − p /4 .

Comprehensive Exercise 2

1. (i) Find the Fourier transform of f ( x), if


 √ (2 π)
 , | x| ≤ ε
f ( x) =  2 ε
 0 , | x| > ε.

(ii) Find the Fourier transform of
 x, | x| ≤ a
f ( x) =  (Gorakhpur 2004; Kanpur 08)
 0 , | x| > a.
M-140

2 2
2. (i) Show that the Fourier transform of f ( x) = e − x /2 is e − p /2 .(Kanpur 2011)

(ii) Find the Fourier transform of the function


 x
1 + a , for − a < x < 0

 x
f ( x) = 1 − , for 0 < x < a
 a
 0 , other wise.

1 − x2 , | x | ≤ 1
3. (i) Find the Fourier transform of F ( x) = 
 0, | x | > 1
(Gorakhpur 2004, 12; Kanpur 10; Purvanchal 14)

∞  x cos x − sin x  x
and hence evaluate ∫0 
 x3
 cos dx.
 2
cos x, 0 < x < a
(ii) Find the cosine transform of the function f ( x) = 
 0, x > a.

sin x, 0 < x < a


(iii) Find the sine transform of the function f ( x) = 
 0, x > a.
(Gorakhpur 2013)
4. (i) Find the Fourier sine and cosine transform of f ( x), if
 x, 0 < x <1

f ( x) = 2 − x, 1 < x < 2 (Kanpur 2007, 11, 14; Gorakhpur 16)
0 , x > 2.

(ii) Find the Fourier sine and cosine transform of the function
f ( x) = x m − 1.
 ∞ ∞
Hint. ∫0 x m − 1 cos px dx − i ∫0 x m − 1 sin px dx

∞ Γ (m) Γ (m)  mπ mπ  
= ∫0 e ipx x m − 1 dx = = cos − i sin 
(ip) m pm  2 2  

5. (i) Find the Fourier sine transform of x /(1 + x2 ).

e ax + e − ax
(ii) Find the cosine transform of ⋅
e πx + e − πx (Purvanchal 2014; Kanpur 14)

1
6. Find the sine transform of πx
and deduce that
e − e − πx
1
Fs (cosech πx) = tanh ( p / 2).
√ (2 π)
M-141

7. Find the Fourier sine transform of f ( x) , if


0 , 0 < x < a

f ( x) =  x, a ≤ x ≤ b
0 , x > b.

8. Find f ( x) if its sine transform is π / 2.
9. Find f ( x) if (i) its sine transform is e − ap, (Kanpur 2012)
− ap
(ii) its cosine transform is e .
~
10. Find the inverse Fourier transform of f ( p) = e −| p | y .
~ e − ap
11. Find f ( x) if f s ( p) = . Hence deduce Fs −1 {1/ p}.
p (Gorakhpur 2009)

12. Solve the integral equation


∞ (1 − λ ), for 0 ≤ λ ≤ 1
∫0 f ( x) cos λ x dx = 
 0 , for λ > 1.
(Gorakhpur 2007, 09, 11, 13)

A nswers 2
sin pε i  2
1. (i) (ii) − ⋅   (ap cos ap − sin ap)
pε 2
p  π

1 2
2. (ii) ⋅ [1 − cos pa]
2
ap π
 p cos p − sin p
3.
 2
(i) −2   ⋅  ; − 3π
 π  p3  16
 
1 sin (1 + p) a sin (1 − p) a 
(ii)  + 
√ (2 π)  1 + p 1− p 

1 sin (1 − p) a sin (1 + p) a 
(iii)  − 
√ (2 π)  1 − p 1+ p 

 2  sin p  2  cos p
4. (i) 2   ⋅ (1 − cos p); 2   ⋅ (1 − cos p).
 π  p2  π p2
Γ(m)  2  mπ Γ(m)  2  mπ
(ii)   sin ; m   cos
pm  π  2 p  π  2
 2  cos (a / 2) ⋅ (e
p /2
+ e − p /2 )
5. (i) √ (π / 2) . e − p (ii)  
 π 2 cos a + e p + e − p
1 ep − 1
6.
2 √ (2 π) ep + 1
M-142

 2  − b cos pb + a cos pa sin pb − sin pa 


7.    + 
 π  p p2 

8. √ (π / 2).(1 / x)
 2 x  2 a y √2
9. (i)   ⋅ (ii)   ⋅ 10.
 π  a2 + x2  π  a2 + x2 √ π( y2 + x2 )
2 ∞ 1 − cos x
√ (2 / π) tan−1 ( x / a) ; √ (π / 2)
π ∫0
11. 12. cos λ x dx
x2

Objective Type Questions

Multiple Choice Questions


Indicate the correct answer for each question by writing the corresponding letter from (a),
(b), (c) and (d).
~
1. If f ( p) is the complex Fourier transform of f ( x), then the complex Fourier
transform of f (ax) is
1 ~  p ~  p
(a) f   (b) f  
a  a  a
1 ~
(c) f ( p) (d) None of these.
a
~ ~
2. If f s ( p) and f c ( p) are infinite Fourier sine and cosine transforms of f ( x)
respectively, then Fc { f ( x) sin ax} is
~ ~ 1 ~ ~
(a) [ f s ( p + a) − f s ( p − a)] (b) [ f s ( p + a) − f s ( p − a)]
2
~ ~ 1 ~ ~
(c) 2 [ f s ( p + a) − f s ( p − a)] (d) [ f s ( p − a) − f s ( p + a)]
2
3. The infinite Fourier sine transform of e − x is
1  2 p  2
2  π  2  π 
(a) (b)
1+ p 1+ p
1  2 1  2
(c)   (d)  
p  π p2  π 
e − ap
4. What is f ( x) if its infinite Fourier sine transform is ?
p
 2 −1 x  2 −1 a
(a)   tan (b)   tan
 π a  π x
 2 −1 2 x  2 −1 2 a
(c)   tan (d)   tan ⋅
 π a  x x
M-143

Fill in the Blank(s)


Fill in the blanks “……” so that the following statements are complete and correct.
1. If the function f ( x) is defined on (− ∞ , ∞) and is piecewise continuous in each
finite partial interval and absolutely integrable in (− ∞ , ∞), then the integral
1 ∞

√ (2 π) − ∞
e ipx f ( x) dx is called the ........... transform of f ( x).

~
2. The infinite Fourier sine transform f s ( p) of f ( x) in 0 < x < ∞ , is ............ .
~
3. If f ( p) is the complex Fourier transform of f ( x), then the complex Fourier
transform of f ( x − a) is ........... .
~ ~
4. If f s ( p) and f c ( p) are infinite Fourier sine and cosine transforms of f ( x)
respectively, then Fs { f ( x) cos ax} = ........... .
 1, | x|< a
5. The Fourier transform of F ( x) =  is ......... .
0 , | x|> a

True or False
Write ‘T’ for true and ‘F’ for false statement.
1. The Fourier transform is a linear transformation.
~
2. If f c ( p) is the infinite Fourier cosine transform of f ( x), then the infinite
~  p
Fourier cosine transform of f (ax) is f c   ⋅
 a
 2 x
3.   ⋅ 2 is the inverse Fourier sine transform of e − ap.
 π  a + x2
~ ~
4. If f s ( p) and f c ( p) are infinite Fourier sine and cosine transforms of f ( x)
1 ~ ~
respectively, then Fs { f ( x) sin ax} = [ f c ( p − a) + f c ( p + a).
2
~ ~
5. The Fourier transform of f ′ ( x), the derivative of f ( x) is (− ip) f ( p), where f ( p)

is the Fourier transform of f ( x).


6. The Fourier transform of the function
 e − xt g(t), t > 0
f (t) =  is the Laplace transform of the function g (t).
 0 , t<0

A nswers

Multiple Choice Questions


1. (a) 2. (b) 3. (b) 4. (a)
M-144

Fill in the Blank(s)


 2 ∞
1. Fourier 2.   ∫ f ( x) sin px dx
 π 0
~ 1 ~ ~
3. e ipa f ( p) 4. [ f s ( p + a) + f s ( p − a)]
2
2 sin pa
5. , p≠0
p √ (2 π)

True or False
1. T 2. F 3. T 4. F
5. T 6. T

¨
M-145

5
F inite F ourier T ransforms

1 Finite Fourier Sine Transforms


et f ( x) denote a function that is sectionally continuous over some finite interval
L (0 , l ) of the variable x. The finite Fourier sine transform of f ( x) on this interval is
defined as
~ l pπx
f s ( p) = ∫ f ( x) sin dx ,
0 l
where p is an integer.
By the proper choice of the origin and the unit of length, if the end points of the interval
become x = 0 and x = π, then
~ π

f s ( p) =
0
f ( x) sin px dx.
~
Note that f s ( p) is always zero when p = 0.
~
The function f ( x) is called the inverse finite Fourier sine transform of f s ( p).
~
i. e., f ( x) = Fs −1 { f s( p)}.
M-146

2 Inversion Formula For Sine Transform


~
If f s ( p) is the finite Fourier sine transform of f ( x) over the interval (0 , l ) then the inversion
formula for sine transform is given by
2 ∞ ~ pπx 2 ∞ ~
f ( x) = ∑ f s ( p)sin l
l p=1
or f ( x) = ∑ f s ( p)sin px
π p=1
~
if (0 , π) is the interval considered for f s ( p).

3 Finite Fourier Cosine Transforms


Let f ( x) denote a function that is sectionally continuous over some finite interval (0 , l)
of the variable x. The finite Fourier cosine transform of f ( x) on this interval is defined
as
~ l pπx
f c ( p) = ∫ f ( x) cos dx
0 l
~
where p is an integer. On the interval (0 , π), f c ( p) is defined as
~ π
f c ( p) = ∫0 f ( x) cos px dx.
~
The function f ( x) is called the inverse finite Fourier cosine transform of f c ( p) i. e.,
~
f ( x) = Fc −1 { f c ( p)}.

4 Inversion Formula for Cosine Transform


~
If f c ( p) is the finite Fourier cosine transform of f ( x) over the interval (0 , l ) then the inversion
formula for cosine transform is given by
1~ 2 ∞ ~ pπx
f ( x) = f c (0 ) +
l

l p=1
f c ( p) cos
l
,

~ l
where f c (0 ) = ∫0 f ( x) dx .

If π is taken as the upper limit for the finite Fourier cosine transform then the inversion formula is
given by
1 ~ 2 ∞ ~
f ( x) =
π
f c (0 ) + ∑ f c ( p) cos px ,
π p=1
~ π
where f c (0 ) = ∫0 f ( x) dx .

Note: The upper limit for Fourier sine or cosine transforms will be taken as x = π, if not
given in the problem.
M-147

5 Multiple Finite Fourier Transforms


Let f ( x, y) be a function of two variables x and y, defined in the square 0 ≤ x ≤ π and
0 ≤ y ≤ π.
Considering it, temporarily, as a function of x, the finite sine transform is given by
~ π
f s ( p, y) = ∫0 f ( x, y) sin px dx
~
and now the finite sine transform of f s ( p, y) which is a function of y is given by
~ π ~
Fs ( p, q) = ∫ f ( p, y) sin qy dy.
0 s
~ π π
∴ Fs ( p, q) = ∫0 ∫0 f ( x, y) sin px sin qy dx dy,

which is the double finite sine transform of f ( x, y) in the region 0 ≤ x ≤ π, 0 ≤ y ≤ π.


Similarly
~ π π
f c ( p, q) = ∫0 ∫0 f ( x, y) cos px cos qy dx dy,

which is the double finite cosine transform of f ( x, y) in the region 0 ≤ x ≤ π, 0 ≤ y ≤ π.


Using inversion formula, we have
~ ∞ ~
2
f s ( p, y) =
π q =1
∑ Fs ( p, q) sin qy

2 ∞ ~
and f ( x, y) = ∑ f s ( p, y) sin px.
π p=1
∞ ∞ ~
4
Hence f ( x, y) = ∑ ∑
π2 p = 1 q = 1
Fs ( p, q) sin px sin qy.

Similarly we can find the inversion formula for double finite cosine transform of
f ( x, y).

6 Operational Properties of Finite Fourier Sine Transform


Theorem 1: The finite Fourier sine transformation resolves the differential form f ′ ′ ( x) into a
~
linear algebraic form in the transform f s ( p) and the boundary values f (0 ) and f (π) in the
manner
~
Fs { f ′ ′ ( x)} = − p2 f s ( p) + p { f (0 ) − (− 1) p f (π)}
whenever f ( x) and f ′ ( x) are continuous and f ′ ′ ( x) is sectionally continuous, on the interval,
0 ≤ x ≤ π.
M-148

Note: If f ′ ′ ( x) and f ′ ′ ′ ( x) are continuous and f (4 ) ( x) is sectionally continuous, on


the interval, 0 ≤ x ≤ π then replacing f ( x) by f ′ ′ ( x), in the result of the above theorem
we have
Fs { f 4 ( x)} = − p2 Fs { f ′ ′ ( x)} + p [ f ′ ′ (0 ) − (−1) p f ′ ′ (π)]
~
= − p2 [− p2 f s ( p) + p { f (0 ) − (−1) p f (π)}] + p { f ′ ′ (0 ) − (−1) p f ′ ′ (π)}
~
= p4 f s ( p) − p3 [ f (0 ) − (− 1) p f (π)] + p [ f ′ ′ (0 ) − (−1) p f ′ ′ (0 )].
In the similar way finite Fourier sine transforms of other derivatives of even order can
be found.
~
Theorem 2: If f s ( p) is the sine transform of a sectionally continuous function f ( x), 0 ≤ x ≤ π,
then
~ 
 f ( p) x π t x t
Fs−1  s  = ∫0 ∫0 f (r) dr dt − ∫0 ∫0 f (r) dr dt
 p  π
2
 
x π π
=
π ∫0 (π − r) f (r) dr − ∫0 ( x − r) f (r) dr.

7 Operational Properties of Finite Fourier Cosine


Transforms
Theorem 1: If f ( x) and f ′ ( x) are continuous and if f ′ ′ ( x) is sectionally continuous, the
finite Fourier cosine transformation resolves the differential form f ′ ′ ( x) into an algebraic form in
~
f c ( p) and the boundary values f ′ (0 ) and f ′ (π), in the manner
~
Fc { f ′ ′ ( x)} = − p2 f c ( p) − f ′ (0 ) + (− 1) p f ′ (π).
Note: If f ′ ′ ( x) and f ′ ′ ′ ( x) are continuous and if f (4 ) ( x) is sectionally continuous,
then replacing f ( x) by f ′ ′ ( x) in (1), we have
Fc { f 4 ( x)} = − p2 Fc { f ′ ′ ( x)} − f ′ ′ ′ (0 ) + (−1) p f ′ ′ ′ (π)
~
= − p2 [− p2 f c ( p) − f ′ (0 ) + (− 1) p f ′ (π)] − f ′ ′ ′ (0 ) + (− 1) p f ′ ′ ′ (π)
~
= p 4 f c ( p) + p2 [ f ′ (0 ) − (− 1) p f ′ (π)] − f ′ ′ ′ (0 ) + (−1) p f ′ ′ ′ (π).
Similarly finite Fourier cosine transforms of other derivatives of even order can be
found.
~
Theorem 2: If f c ( p) is the cosine transform of a sectionally continuous function f ( x), 0 ≤ x ≤ π,
then
~  ~
−1  f c ( p) x π f c (0 )
Fc 
 p 
2  ∫0= ∫t f (r) dr dt +

( x − π)2 + A,
 
where A is an arbitrary constant.
M-149

8 Combined Properties of Finite Fourier Sine and Cosine


Transforms
When f ( x) is continuous and f ′ ( x) is sectionally continuous then
(a) Fs { f ′ ( x)} = − pFc { f ( x)}, ( p = 1, 2, 3,...)
and (b) Fc { f ′ ( x)} = pFs { f ( x)} − f (0 ) + (−1) p f (π), ( p = 0 , 1, 2, ...).
Note: If H ( x) is any sectionally continuous function, the above properties may be
written as
 x 
Fs { H ( x)} = − p Fc  ∫ H (r) dr , ( p = 1, 2, ......)
 0 
 1 ~   π x ~ 
and Fc  H ( x) − Hc (0 ) = p Fs  ∫ H (r) dr − Hc (0 ) ,
 π   0 π 
( p = 0 , 1, 2, ......)

9 Convolution
Let F ( x) and G ( x) be two functions defined on the interval −2 π < x < 2 π, then the function
π
F ( x) * G ( x) = ∫−π F ( x − y) G ( y) dy

is called the convolution of F ( x) and G ( x) on the interval − π < x < π.

Example 1: Find the finite Fourier sine and cosine transforms of the function
f ( x) = 2 x, 0 < x < 4. (Agra 2003)

Solution: We have
~ l
f s ( p) = ∫0 f ( x) sin ( pπx / l ) dx]

4
= ∫ 2 x sin ( pπx / 4) dx, as l = 4
0 (Given)
4
 − 2 x cos ( pπx / 4) 4 cos ( pπx / 4)
=  + 2 ∫0 dx
 pπ / 4 0 pπ / 4
4
32 8 sin ( pπx / 4) 32
=− cos pπ +   =− cos pπ.
pπ pπ  pπ / 4 0 pπ
~ l
Also f c ( p) = ∫0 f ( x) cos ( pπx / l) dx

4
= ∫ 2 x cos ( pπx / 4) dx, as l = 4
0
M-150

4
2 x sin ( pπx / 4) 4 sin ( pπx / 4)
=  − 2 ∫0 dx
 p π / 4 0 pπ / 4
4
8 cos ( pπx / 4) 32
=   = (cos pπ − 1) , if p > 0
pπ  pπ / 4 0 p2 π2
~ 4
and if p = 0, then f c ( p) = ∫0 2 x ⋅ 1 dx = 16.

Example 2: Find the finite Fourier cosine transform of f ( x) if


π x2
(i ) f ( x) = −x+ ⋅ (ii) f ( x) = sin nx.
3 2π
~ π
Solution: (i) We have f c ( p) = ∫0 f ( x) cos px dx .

~ π π x2 
∴ f c ( p) = ∫0  −x+
3

 cos px dx
2 π 
π
 π x2  1  1 π  x
p ∫0
=  − x +  sin px −
 p  −1 +  sin px dx
 3 2 π  0  π
π
1  x 1  1 π1
= − −  −1 +  cos px −
p  
π p 0
∫ cos px dx
p2 0 π
1 1 π 1
3 [
= − sin px]0 = , if p > 0
2
p p π p2
and when p = 0,
~ π π x2 
f c ( p) = ∫ 0  3
 −x+  dx = 0
2 π 
~ π
(ii) f c ( p) = ∫0 sin nx cos px dx

1 π
=
2 ∫0 [sin (n + p) x + sin (n − p) x] dx
π
1  cos (n + p) x cos (n − p) x 
= − −  , if p ≠ n
2 n+ p n− p 0
~ 1  cos (n + p) π cos (n − p) π 1 1 
∴ if p ≠ n, f c ( p) = − − + + ⋅
2 n+ p n− p n+ p n− p
If n − p is even, then n + p is also even and so
~ 1 1 1 1 1 
f c ( p) = − − + +  =0.
2  n+ p n− p n+ p n− p
If n − p is odd, then n + p is also odd and so
~ 1 2 2  2n
f c ( p) =  + = ⋅
2 n + p n− p n2 − p2
M-151

If p = n, then
~ π 1 π
f c ( p) = ∫
0
sin nx cos nx dx =
2 0 ∫
sin 2 nx dx

π
1  cos 2 nx 
=  − =0.
2 2 n 0
~ 2n
∴ f c ( p) = 0 or according as n − p is even or odd.
n − p2
2

cos k (π − x)
Example 3: Find the finite cosine transform of f ( x) if f ( x) = − ⋅
k sin kπ

Solution: We have
~ π cos { k (π − x)}
f c ( p) = −
0 ∫ k sin k π
cos px dx

1 π
=−
2 k sin kπ ∫0 [cos { k (π − x) + px } + cos { k (π − x) − px }] dx

π
1 sin (kπ − kx + px) sin (kπ − kx − px)
=−  − 
2 k sin kπ  p− k p+ k 0
1 sin pπ sin (− pπ) sin kπ sin kπ 
=−  − − + 
2 k sin kπ  p − k p+ k p− k p+ k 

1  1 1  1
=  −  = , k ≠ 0 , 1, 2, 3, ......
2 k  p − k p + k  p2 − k 2

Example 4: Find f ( x) if
 pπ 
6 sin − cos pπ
~  2  2
f c ( p) = for p = 1, 2, 3,...... and for p = 0,
(2 p + 1) π π
where 0 < x < 4.
Solution: We have
1~ 2 ∞ ~ pπx
f ( x) =
l
f c (0 ) + ∑ f c ( p) cos l
l p=1

 pπ 
6 sin − cos pπ
1 2 2 ∞  2  pπx
= ⋅ + ⋅ ∑ cos
4 π 4 p=1 (2 p + 1) π 4

 pπ 
sin − cos pπ
1 3 ∞  2   pπx 
= + ∑
2π π p = 1 2p + 1
cos 
 4 
⋅

Example 5: Find f ( x) if its finite sine transform is given by


~ 1 − cos pπ
f s ( p) = , where 0 < x < π.
2 2
p π (Kanpur 2008)
M-152

Solution: We have
2 ∞ ~
f ( x) = ∑ f s ( p) sin px
π p=1

2 ∞ 1 − cos pπ 
= ∑
π p = 1  p2 π2 
sin px

∞ 
2 1 − cos pπ 
=
π 3 ∑ 
 p2
 sin px.

p =1 

Comprehensive Exercise 1

1. (i) Find the finite Fourier sine and cosine transforms of f ( x) = 1.


(Gorakhpur 2014)
(ii) Find the finite Fourier sine and cosine transforms of f ( x) = x.
2. (i) Find the finite Fourier sine transforms of
 x x
1 −  and ⋅
 π 4π
(ii) Find the finite Fourier cosine transform of
 x x
1 −  and ⋅
 π 4π
3. Find the finite Fourier sine transform of f ( x) if
 x, 0 ≤ x≤ π/2
(i) f ( x) = 
 π − x, π / 2 ≤ x ≤ π .
 − x, x< c
(ii) f ( x) = 
 π − x, x > c , where 0 ≤ c ≤ π .
4. (i) Find the finite Fourier cosine transform of f ( x) if
 1, 0 < x < π / 2
f ( x) = 
 −1, π / 2 < x < π. (Kanpur 2012)
2
(ii) Find the finite cosine transform of (1 − x / π) . (Avadh 2013)
5. Find the finite Fourier sine transforms of
(i) x (π − x) (ii) x (π2 − x2 ).
6. Find the finite Fourier sine transform of f ( x) if
π x2
(i) f ( x) = sin nx (ii) f ( x) =
−x+ ⋅
3 2π
7. Find the finite sine transform of f ( x), if
(i) f ( x) = cos kx (ii) f ( x) = x3 (iii) f ( x) = e cx .

cosh { c (π − x)}
8. Find finite Fourier cosine transform of f ( x) if f ( x) = ⋅
sinh (πc )
M-153

sin k (π − x)
9. Find finite Fourier sine transform of f ( x) , if f ( x) = ⋅
sin (kπ)
10. Find the finite Fourier sine and cosine transforms of
f ( x) = x2 , 0 < x < 4. (Gorakhpur 2008, 14, 15)
~ cos(2 pπ / 3)
11. Find f ( x) if f c ( p) = , if 0 < x < 1.
2
(2 p + 1)
12. When f ( x) = sin mx, where m is a positive integer, show that
~ ~
f s ( p) = 0 if p ≠ m and f s ( p) = π / 2 if p = m.

A nswers 1
1
1. (i) [1 − (− 1) p]; 0
p
π (−1) p + 1 (−1) p − 1 π2
(ii) ; , if p = 1, 2, 3,... and , if p = 0 ⋅
p p2 2
1 (−1) p + 1
2. (i) ;
p 4p
1 1
(ii) [1 − (−1) p] ; [(−1) p − 1]
2
πp 4 πp2
3. (i) (2 / p2 ) sin ( pπ / 2) , (ii) (π / p) cos pc .
4. (i) (2 / p) sin ( pπ / 2), p > 0 and 0, if p = 0.
2 π
(ii) , if p > 0 and , if p = 0
2
πp 3
2 6π
5. (i) p
[1 − (− 1) ], (ii) (−1) p + 1
3 3
p p
6. (i) 0, if p ≠ n , and π / 2, if p = n
π 1
(ii) {(− 1) p + 2} + {(− 1) p − 1}
3
6p πp
p 6 π2 
7. (i) [1 − (−1) p cos kπ], (ii) π (− 1) p  − ,
p2 − k 2  p3 p 

p
(iii) [1 − (−1) p e cπ ]
c + p2
2

c p
8. 9., k ≠ 0 , 1, 2,... .
2 2
c +p p − k2 2

64 128 128 64
10. − cos pπ + (cos pπ − 1) ; cos p π, if p > 0; , if p = 0
pπ 3
p π 3 2
p π 2 3

cos (2 pπ / 3)
11. 1 + 2 ∑ (2 p + 1)2
cos pπx
p=1
M-154

Objective Type Questions

Multiple Choice Questions


Indicate the correct answer for each question by writing the corresponding letter from (a),
(b), (c) and (d).
1. If f ( x) is continuous and f ′ ( x) is sectionally continuous then Fs { f ′ ( x)} is
(a) − Fc { f ( x)} (b) Fc { f ( x)}
(c) − p Fc { f ( x) } (d) − p2 Fc { f ( x) }
2. The finite Fourier sine transform of f ( x) = x is
π π
(a) (− 1) p + 1 (b) (− 1) p
p p
π π
(c) (− 1) p + 1 (d) (− 1) p
2
p p2
~ 2 π (− 1) p − 1
3. What is f ( x) if its finite sine transform f s ( p) is , where 0 < x < π ?
p3
2 ∞ (− 1) p − 1 ∞
(− 1) p
(a) ∑
π p=1 p3
sin px (b) 4 ∑ p3
sin px
p =1

(− 1) p − 1 4 ∞ (− 1) p − 1
(c) 4 ∑ p3
sin px (d) ∑
π p=1 p3
sin px
p=1
x
4. The finite Fourier sine transform of is
π
(− 1) p (− 1) p + 1
(a) (b)
p2 p2
(− 1) p − 1 (− 1) p + 1
(c) (d)
p p

Fill in the Blank(s)


Fill in the blanks “……” so that the following statements are complete and correct.
~
1. The finite Fourier sine transform f s ( p) of f ( x) on the interval (0 , π) is ........
~ π
2. On the interval (0 , π) , f c ( p) = ∫0 f ( x) cos px dx, then the function f ( x) is
~
called ................. Fourier cosine transform of f c ( p).
3. If F ( x) and G ( x) are two functions defined on the interval − 2 π < x < 2 π ,then the
π
function F ( x) * G ( x) = ∫−π F ( x − y) G ( y) dy is called the ......... of F ( x) and

G ( x) on the interval − π < x < π.


4. The finite Fourier sine transform of the function f ( x) = 2 x , 0 < x < 4 is .............
M-155

True or False
Write ‘T’ for true and ‘F’ for false statement.
~
1. If f s ( p) is the finite Fourier sine transform of f ( x) on the interval (0 , π) then
2 ∞ ~
the inversion formula for sine transform is f ( x) = ∑ f s ( p)sin px
π p=1

2. If f ( x) is continuous and f ′ ( x) is sectionally continuous then


Fc { f ′ ( x)} = p Fs { f ( x)} − f (0 ) + f (π).
3. The finite Fourier cosine transform of the function f ( x) = 1 is 0.
~ 1 − cos pπ
4. If the finite sine transform is given by f s ( p) = , where 0 < x < π , then
p2 π2
∞ 1 − cos pπ 
2
the value of f ( x) is
π3
∑ 
 p2
 sin px.

p =1  

A nswers
Multiple Choice Questions
1. (c) 2. (a) 3. (c) 4. (d)

Fill in the Blank(s)


π
1. ∫0 f ( x) sin px dx 2. inverse finite

− 32
3. Convolution 4. cos pπ

True or False
1. T 2. F 3. T 4. T

You might also like